The Flat Earth Society

Flat Earth Discussion Boards => Flat Earth Investigations => Topic started by: Tom Bishop on July 16, 2018, 11:43:56 PM

Title: Round Earth Celestial Mechanics Cannot Predict the Solar System
Post by: Tom Bishop on July 16, 2018, 11:43:56 PM
I have made a new article on the wiki: Celestial Mechanics Cannot Predict the Solar System (https://wiki.tfes.org/Celestial_Mechanics_Cannot_Predict_The_Solar_System)

This is a companion to my previous article, the NOAA Solar Calculator (https://wiki.tfes.org/NOAA_Solar_Calculator), and should both be read.

It is often alleged that we need only download an astronomy software to see that the Round Earth system predicts celestial events, and that this is a demonstration of the superiority and fact of the Round Earth model. Users on this forum have linked us to astronomy software as their evidence and refuse to entertain the idea that they are just linking to pattern-based methods rather than anything to do with the heliocentric system.

It has also been alleged that the seemingly simple math used in Astronomical calculation textbooks are "really" based on keplerian or newtonian orbital dynamics. This article addresses that.

I have collected a number of resources showing that the pervasive myth that the Round Earth Theory has been validated, to be false. I ask that any challenger in opposition demonstrates with real evidence that astronomy can predict the motions of the planets as they are described in the Round Earth Theory. More evidence than a link to an obscure pdf or unverified model. It will need to be demonstrated that a model, according to the geometry of the heliocentric system, can predict any positions of the planets at all!
Title: Re: Round Earth Celestial Mechanics Cannot Predict the Solar System
Post by: Tumeni on July 17, 2018, 12:04:25 AM
What counts as "real evidence"?
Title: Re: Round Earth Celestial Mechanics Cannot Predict the Solar System
Post by: Tom Bishop on July 17, 2018, 12:09:39 AM
What counts as "real evidence"?

Not evidence: "Go and download Stellarium, it's a Round Earth model."

Evidence: "Here is evidence that Stellarium is using a Round Earth model. Look here, we can see that the equations are using the Round Earth distances as starting points."

Evidence: "Here is a study showing a successful astronomy model."
Title: Re: Round Earth Celestial Mechanics Cannot Predict the Solar System
Post by: ICanScienceThat on July 17, 2018, 12:10:42 AM
I have made a new article on the wiki: Celestial Mechanics Cannot Predict the Solar System (https://wiki.tfes.org/Celestial_Mechanics_Cannot_Predict_The_Solar_System)

This is a companion to my previous article, the NOAA Solar Calculator (https://wiki.tfes.org/NOAA_Solar_Calculator), and should both be read.

It is often alleged that we need only download an astronomy software to see that the Round Earth system predicts celestial events, and that this is a demonstration of the superiority and fact of the Round Earth model. Users on this forum have linked us to astronomy software as their evidence and refuse to entertain the idea that they are just linking to pattern-based methods rather than anything to do with the heliocentric system.

It has also been alleged that the seemingly simple math used in Astronomical calculation textbooks are "really" based high level keplerian orbital dynamics. This article addresses that.

I have collected a number of resources showing that the perverted myth that the Round Earth Theory has been validated to be false. I ask that any challenger in opposition demonstrates with real evidence that astronomy can predict the motions of the planets as they are described in the Round Earth Theory. More evidence than a link to an obscure pdf or unverified model. It will need to be demonstrated that the math, according to the geometry of the heliocentric system, can predict any positions of the planets at all!
You are aware that Kepler and Newton used empirical observations of the astronomical bodies to derive their models right? You're probably aware that Newton was left perplexed with a tiny deviation in the behavior of Mercury that didn't match his model. And you may have heard the story of how Einstein finally fixed the model for Mercury with GR.

I think you are saying that because we store all these observations in charts, that invalidates the model that is able to predict them. Is that right?

Let me ask you this... in what format do you think these numbers are stored? When you use one of these many planetarium types of software, how does it tell you where the moon will be at your lat/long at any given time? Do you think we have charts of the future location of the moon for every possible time at every possible location on the globe? For the stars, they mostly don't move, so just narrow it down to the location. If we have the position of every star in a big chart, how do they know where to draw that on your screen? They need to factor in your lat/long in some way right? How do you think they do that?
Title: Re: Round Earth Celestial Mechanics Cannot Predict the Solar System
Post by: Tom Bishop on July 17, 2018, 12:18:10 AM
You are aware that Kepler and Newton used empirical observations of the astronomical bodies to derive their models right?

You know how to Google. Show that Kepler or Newton had any successful models based on orbital mechanics that predicted reality.

Quote
You're probably aware that Newton was left perplexed with a tiny deviation in the behavior of Mercury that didn't match his model. And you may have heard the story of how Einstein finally fixed the model for Mercury with GR.

Google. Demonstrate the orbital mechanics. Einstein didn't solve the multiple body problems or the major problems in celestial mechanics.

Quote
I think you are saying that because we store all these observations in charts, that invalidates the model that is able to predict them. Is that right?

Let me ask you this... in what format do you think these numbers are stored? When you use one of these many planetarium types of software, how does it tell you where the moon will be at your lat/long at any given time? Do you think we have charts of the future location of the moon for every possible time at every possible location on the globe? For the stars, they mostly don't move, so just narrow it down to the location. If we have the position of every star in a big chart, how do they know where to draw that on your screen? They need to factor in your lat/long in some way right? How do you think they do that?

Astronomical events come in patterns. The ancients were able to predict events such as eclipses because they occur in a pattern that can be expressed with an equation. Astronomy as a whole is based on patterns, not "celestial mechanics."
Title: Re: Round Earth Celestial Mechanics Cannot Predict the Solar System
Post by: ICanScienceThat on July 17, 2018, 12:31:35 AM
You are aware that Kepler and Newton used empirical observations of the astronomical bodies to derive their models right?

You know how to Google. Show that Kepler or Newton had any successful models based on orbital mechanics that predicted reality.

Quote
You're probably aware that Newton was left perplexed with a tiny deviation in the behavior of Mercury that didn't match his model. And you may have heard the story of how Einstein finally fixed the model for Mercury with GR.

Google. Demonstrate the orbital mechanics. Einstein didn't solve the multiple body problems or the major problems in celestial mechanics.

Quote
I think you are saying that because we store all these observations in charts, that invalidates the model that is able to predict them. Is that right?

Let me ask you this... in what format do you think these numbers are stored? When you use one of these many planetarium types of software, how does it tell you where the moon will be at your lat/long at any given time? Do you think we have charts of the future location of the moon for every possible time at every possible location on the globe? For the stars, they mostly don't move, so just narrow it down to the location. If we have the position of every star in a big chart, how do they know where to draw that on your screen? They need to factor in your lat/long in some way right? How do you think they do that?

Astronomical events come in patterns. The ancients were able to predict events such as eclipses because they occur in a pattern that can be expressed with an equation. Astronomy as a whole is based on patterns, not "celestial mechanics."

I'm already busy with the moon tilt illusion I'm working on for you, so I don't have much time for this. Here are a few googles you might find interesting:
http://www.stjarnhimlen.se/comp/ppcomp.html
https://io9.gizmodo.com/the-200-year-old-mystery-of-mercurys-orbit-solved-1458642219

I do not refute that all of this information comes from patterns. Look at how the calculations are done, and you'll see a whole list of parameters that were achieved through pure observation. And in that first link, you'll see that they are only presenting things down to a certain level of precision.

But the fact remains, you cannot project a star onto a screen in the correct position unless you first assume the world is a globe.
Title: Re: Round Earth Celestial Mechanics Cannot Predict the Solar System
Post by: Tom Bishop on July 17, 2018, 12:47:54 AM
I'm already busy with the moon tilt illusion I'm working on for you, so I don't have much time for this. Here are a few googles you might find interesting:
http://www.stjarnhimlen.se/comp/ppcomp.html

It is apparent that Celestial Mechanics can attempt to describe orbital motions, and "positions" can be "computed." The problem is that they are unable to predict anything.

Quote
https://io9.gizmodo.com/the-200-year-old-mystery-of-mercurys-orbit-solved-1458642219

That's an article. I can imagine that a different version of gravity would probably better explain why mercury moves faster or differently in the sky compared to other planets too. It is not a demonstration of celestial mechanics, or showcase its predictive power. You would need to demonstrate that this has anything to do with what we are talking about.

We can take the current three body problem models of Celestial Mechanics and change around the gravity of the sun to show that bodies generally can behave quicker when near it depending on our theory of gravity. None of this is a demonstration that the model can actually predict the positions of bodies into the future. That is far from prediction of the planets in the sky.
Title: Re: Round Earth Celestial Mechanics Cannot Predict the Solar System
Post by: BillO on July 17, 2018, 01:49:47 AM
This is a companion to my previous article, the NOAA Solar Calculator (https://wiki.tfes.org/NOAA_Solar_Calculator), and should both be read.
Well, your article on the NOAA calculator was wrong and I explained why.  I'll give your new article a read though and see what it says.
Title: Re: Round Earth Celestial Mechanics Cannot Predict the Solar System
Post by: ICanScienceThat on July 17, 2018, 01:59:44 AM
I'm already busy with the moon tilt illusion I'm working on for you, so I don't have much time for this. Here are a few googles you might find interesting:
http://www.stjarnhimlen.se/comp/ppcomp.html

It is apparent that Celestial Mechanics can attempt to describe orbital motions, and "positions" can be "computed." The problem is that they are unable to predict anything.

Quote
https://io9.gizmodo.com/the-200-year-old-mystery-of-mercurys-orbit-solved-1458642219

That's an article. I can imagine that a different version of gravity would probably better explain why mercury moves faster or differently in the sky compared to other planets too. It is not a demonstration of celestial mechanics, or showcase its predictive power. You would need to demonstrate that this has anything to do with what we are talking about.

We can take the current three body problem models of Celestial Mechanics and change around the gravity of the sun to show that bodies generally can behave quicker when near it depending on our theory of gravity. None of this is a demonstration that the model can actually predict the positions of bodies into the future. That is far from prediction of the planets in the sky.
I'm sitting here scratching my head. I think we mean different things by the word "prediction." That's got to be the explanation. I'm not sure what you mean by "prediction," but I can tell you what I mean. What I mean is that if you tell me where a body is in it's orbit around the Sun with it's position and current velocity, I can predict for you where it's likely to go in the future. The accuracy of the prediction is going to vary based on how accurate the initial information was, and it will also depend highly on how accurate my model of the other objects in the solar system is. As the prediction goes forward in time, it is likely to drift farther away from accuracy. So, yes we can make predictions. We know when objects are coming our way. We know of many large asteroids, but we're pretty sure that none of them will endanger us any time soon. How do we know? Because we've used these formulas to predict the future trajectory of every large object we are aware of. That's a "prediction." You must mean something else.
Title: Re: Round Earth Celestial Mechanics Cannot Predict the Solar System
Post by: Tom Bishop on July 17, 2018, 03:16:31 AM
I'm already busy with the moon tilt illusion I'm working on for you, so I don't have much time for this. Here are a few googles you might find interesting:
http://www.stjarnhimlen.se/comp/ppcomp.html

It is apparent that Celestial Mechanics can attempt to describe orbital motions, and "positions" can be "computed." The problem is that they are unable to predict anything.

Quote
https://io9.gizmodo.com/the-200-year-old-mystery-of-mercurys-orbit-solved-1458642219

That's an article. I can imagine that a different version of gravity would probably better explain why mercury moves faster or differently in the sky compared to other planets too. It is not a demonstration of celestial mechanics, or showcase its predictive power. You would need to demonstrate that this has anything to do with what we are talking about.

We can take the current three body problem models of Celestial Mechanics and change around the gravity of the sun to show that bodies generally can behave quicker when near it depending on our theory of gravity. None of this is a demonstration that the model can actually predict the positions of bodies into the future. That is far from prediction of the planets in the sky.
I'm sitting here scratching my head. I think we mean different things by the word "prediction." That's got to be the explanation. I'm not sure what you mean by "prediction," but I can tell you what I mean. What I mean is that if you tell me where a body is in it's orbit around the Sun with it's position and current velocity, I can predict for you where it's likely to go in the future. The accuracy of the prediction is going to vary based on how accurate the initial information was, and it will also depend highly on how accurate my model of the other objects in the solar system is. As the prediction goes forward in time, it is likely to drift farther away from accuracy. So, yes we can make predictions. We know when objects are coming our way. We know of many large asteroids, but we're pretty sure that none of them will endanger us any time soon. How do we know? Because we've used these formulas to predict the future trajectory of every large object we are aware of. That's a "prediction." You must mean something else.

Just look at what you linked us to:

Quote
The Precession of Mercury

The orbits of the planets are slight ovals, with the sun located toward one end of the oval. The point at which the planet comes closest to the sun is the perihelion, and the farthest point of the oval is the aphelion. The oval orbits themselves move. As if the sun were a pin stuck into them, the orbits slowly rotate around it, in a motion called precession.

It's talking about about its assumed orbit, and why the planet Mercury's "motions in the sky" are odd compared to the other planets.

Here is a basic gravity simulator: https://www.testtubegames.com/gravity.html

We can change the elements and see that if we change gravity, we can get the little elements to move differently. Since Mercury seems to move too fast in its changing positions of the sky, another version of gravity may seem to work a little better. We are just changing gravity on an incredibly basic model.

Explaining why the apparent changing position Mercury moves a little more off from its pattern repeating patterns (assumed orbits) might work better with a different type of gravity, is something that is vastly different than a model of orbital mechanics that can predict the position of mercury in the sky.

Read what it it talking about. It is not talking about the prediction of the position of mercury in the sky. Not at all. It is talking about some elements of its assumed orbit when the patterns in the sky repeat themselves that would work better if gravity were different.

Placing little balls on an online gravity simulator to 'orbit' each other, and trying to find the right version of gravity that makes the closest ball move a little more in its repeating patterns, because in reality it moves more in its repeating patterns, and under your interpretation on what is happening when seen from earth, is not what we are talking about here. It is possible to analyze the heliocentric model on a very simple level like this, and change the reach of gravity to maybe "explain" something a little better on a high theoretical level about the assumed system, but using the motions of those bodies to actually predict the future position of a body is, due to the 3 body/n-body problem, impossible.

The whole of astronomy is based on fictitious explanations to "explain" phenomena. Einstein's fictitious phenomena might be better than Newton's fictitious phenomena to explain something about a fictitious system. But until real prediction of bodies can be performed to validate the system, the fictitious system and the fictitious explanations that are a part of it remain just that -- fiction.
Title: Re: Round Earth Celestial Mechanics Cannot Predict the Solar System
Post by: ICanScienceThat on July 17, 2018, 04:12:00 AM
I'd really like to help you to understand this, but it's going to take some time. I'm going back to the tilt illusion for now. As my last note on the subject, I'm convinced more than ever that we're not using the same vocabulary. We must be seeing the same things, but they mean something different to each of us. I'll once again back up until I think we're on common ground...

The formulas and charts to predict the future locations of the planets ARE based on past observations. They break those observations down into patterns and extend those patterns forward in time. We agree so far.

Extending the pattern forward in time like this is a prediction, and it works in a geocentric model or a heliocentric model. Yeah I'm agreeing that these predictions worked under the geocentric model. We're on common ground. The geocentric model was every bit as accurate as the later heliocentric model. The only real difference was that the heliocentric model was a lot simpler. The math was simpler, and the diagrams were simpler. So the heliocentric model gained favor rapidly.

Here we're going to start to diverge, but I'm still talking history here...
It wasn't until Newton's insight about the inverse-square law that the elliptical orbits of the planets suddenly made sense. The heliocentric model already had the elliptical model based on pure observation, and Newton used that to deduce his inverse square law of gravity. Disagree with his conclusions if you like, but that's how he came up with it, and hopefully we agree on that part.

Edit: Still talking history here, I'd like to note that both the geocentric and the heliocentric models at this point had a round Earth. That was not in dispute.

Here's where our ideas are about to diverge...
We can use either the geocentric or the heliocentric models to predict the future locations of the planets. I think you'll find these charts all use the heliocentric model these days because that model is a lot simpler. (That simply means assuming the planets move in ellipses around the Sun and adding precession if they are accurate enough. No they don't generally re-simulate the paths of the orbits based on the mass of the Sun or any of that... just the historical patterns of the planets.) But then once we have that, how do we tell where they will be in our sky at any given place on the Earth? In that NOAA spreadsheet you have mentioned, I would like to draw your attention to cells B3 and B4. These are the observer's latitude and longitude. The locations of the bodies is presented given the observer's lat/long under the assumption that the Earth is a globe. Take a look at cell W2:
=DEGREES(ACOS(COS(RADIANS(90.833))/(COS(RADIANS($B$3))*COS(RADIANS(T2)))-TAN(RADIANS($B$3))*TAN(RADIANS(T2))))
Notice that the latitude is going into some trig there. That isn't projecting the sun onto a flat earth. That's RE math there.

Edit: One last note. You seem really fascinated with the N-body problem. We have no analytical solution for this. Meaning, there does not exist a mathematical formula to solve this in general. All that means is we have to do it with computer simulations instead. Not really a flaw in RE in any way.
Title: Re: Round Earth Celestial Mechanics Cannot Predict the Solar System
Post by: Tom Bishop on July 17, 2018, 04:57:14 AM
As my last note on the subject, I'm convinced more than ever that we're not using the same vocabulary. We must be seeing the same things, but they mean something different to each of us. I'll once again back up until I think we're on common ground...

All I can point out is, again, that the article you posted is not talking about the position of mercury in the sky. It's talking about a particular observation mercury is seen to do when the pattern of its motion on the sky repeats itself. Einstein's version of gravity is a little different, and so on a high level "basic gravity simulator," or otherwise simple model, the bodies closer to the center would move a little differently.

Since the Flat Earth theory also has the planets moving around the sun, with Mercury at the closest position, I don't find it odd that a different version of the phenomena that keeps the planets moving around the sun could cause the most interior body to move a little differently in its repeating patterns in the sky.

This is a basic modification to a high level and theoretical system. But we are just talking about basic orbit types on a basic and high level model, and slight adjustments to make some interior planet move faster or slower based on your imagined central pulling phenomena. It is an imagined explanation.

To prove the model there would need to be a prediction of a body in the sky and expression of the celestial mechanics. If the article you had posted claimed that Einstein predicted the position of Mercury in the sky this would be a very different conversation. He did not do that, and the articles surrounding this event admit as such.

Quote
But then once we have that, how do we tell where they will be in our sky at any given place on the Earth? In that NOAA spreadsheet you have mentioned, I would like to draw your attention to cells B3 and B4.

These are the observer's latitude and longitude. The locations of the bodies is presented given the observer's lat/long under the assumption that the Earth is a globe. Take a look at cell W2:
=DEGREES(ACOS(COS(RADIANS(90.833))/(COS(RADIANS($B$3))*COS(RADIANS(T2)))-TAN(RADIANS($B$3))*TAN(RADIANS(T2))))
Notice that the latitude is going into some trig there. That isn't projecting the sun onto a flat earth. That's RE math there.

Why do you think it has anything to do with a Round Earth? Cell W2 is the sunset column. The time of sunset must have some sort of relationship to do with your longitude, even in the Flat Earth model. Using trig manipulations to find the relationship is entirely possible.

Quote
Edit: One last note. You seem really fascinated with the N-body problem. We have no analytical solution for this. Meaning, there does not exist a mathematical formula to solve this in general. All that means is we have to do it with computer simulations instead. Not really a flaw in RE in any way.

The impossibility for an analytical solution of the Three Body Problem means that we can't create formulas to turn the positions and movements of bodies into equations that will predict future occurrences under the heliocentric model. It means that it is impossible to create a program like Stellarium that predicts things under a Heliocentric or Round Earth model. The idea that such programs are possible is the crux of many arguments that the Round Earth model is true.

If such programs are impossible, and the prediction of bodies is really made on patterns and trends from past occurrences then it means that the Heliocentric Theory is much weaker.

It is more of a blow and punch to the nose. It shows that the model is much closer to a hypothesis than its supporters believe.

The inability to predict suggests that it is either the science or the model that is incorrect, or both. The inability to predict the positions of bodies does not lend support to the heliocentric model, and only subtracts from it. It detracts from the reputation of classical mechanics, mathematics, astronomy, et all.
Title: Re: Round Earth Celestial Mechanics Cannot Predict the Solar System
Post by: sandokhan on July 17, 2018, 05:14:24 AM
https://www.theflatearthsociety.org/forum/index.php?topic=30499.msg1774581#msg1774581 (three body problem paradox, dependence on the sensitivity of the initial conditions of the set of differential equations, Hamiltonian formulation of the equations of celestial mechanics, homoclinic tangles)

https://www.theflatearthsociety.org/forum/index.php?topic=30499.msg1935048#msg1935048 (orbital stability of the heliocentrical solar system, 300 year limit for the RE calculations, Nekhoroshev's theorem, Saari's theorem)
Title: Re: Round Earth Celestial Mechanics Cannot Predict the Solar System
Post by: SphericalEarther on July 17, 2018, 07:14:09 AM
So basically your claim is, that because NOAA doesn't seem to use orbital mechanics for their calculations, the RE is false.

First up, the linked spreadsheet:
The Sun RAD Vector isn't used for the other calculations, it is an output to show the current distance to the sun.
All formulas in the sheet are simplified to the extreme, so that they are faster to work with, thereby they are also nearly impossible to apprehend.

Second:
You seem to assume that we would use orbital mechanics for our calculations, while I would expect that we would not.
Orbital mechanics are used for simulations, not as much for calculations. This is because orbital mechanics are extremely hard to calculate over time, where all planets and moons affect each other always, and affect each other more or less dependent on their current distance.
If you want orbital mechanics based on Newtonian physics: http://universesandbox.com/ it is actually pretty fun to use, and you can play with it all you wish. It is also very accurate (as long as you don't run the simulation too fast).

Third:
Stellarium is an excellent tool, it precisely shows all observations we see in the sky, and shows all predictions. Even if Stellarium is pattern based, it still shows all observations anywhere on earth which can all be verified. Now that you've brought it up, I would like to know if you acknowledge Stellarium as a valid source of observations, or if you have a problem with it?
Is there any observation in Stellarium which you know cannot be seen in reality?
Title: Re: Round Earth Celestial Mechanics Cannot Predict the Solar System
Post by: Tumeni on July 17, 2018, 08:34:18 AM
Astronomical events come in patterns. The ancients were able to predict events such as eclipses because they occur in a pattern that can be expressed with an equation.

You do realise that eclipses can only be seen from certain areas of the world, don't you?

And that the area from which an eclipse can be observed is predictable, along with its duration, path of totality, etc.?
Title: Re: Round Earth Celestial Mechanics Cannot Predict the Solar System
Post by: totallackey on July 17, 2018, 12:03:33 PM
I have made a new article on the wiki: Celestial Mechanics Cannot Predict the Solar System (https://wiki.tfes.org/Celestial_Mechanics_Cannot_Predict_The_Solar_System)

This is a companion to my previous article, the NOAA Solar Calculator (https://wiki.tfes.org/NOAA_Solar_Calculator), and should both be read.

It is often alleged that we need only download an astronomy software to see that the Round Earth system predicts celestial events, and that this is a demonstration of the superiority and fact of the Round Earth model. Users on this forum have linked us to astronomy software as their evidence and refuse to entertain the idea that they are just linking to pattern-based methods rather than anything to do with the heliocentric system.

It has also been alleged that the seemingly simple math used in Astronomical calculation textbooks are "really" based high level keplerian orbital dynamics. This article addresses that.

I have collected a number of resources showing that the perverted myth that the Round Earth Theory has been validated to be false. I ask that any challenger in opposition demonstrates with real evidence that astronomy can predict the motions of the planets as they are described in the Round Earth Theory. More evidence than a link to an obscure pdf or unverified model. It will need to be demonstrated that the math, according to the geometry of the heliocentric system, can predict any positions of the planets at all!
You are aware that Kepler and Newton used empirical observations of the astronomical bodies to derive their models right? You're probably aware that Newton was left perplexed with a tiny deviation in the behavior of Mercury that didn't match his model. And you may have heard the story of how Einstein finally fixed the model for Mercury with GR.

I think you are saying that because we store all these observations in charts, that invalidates the model that is able to predict them. Is that right?

Let me ask you this... in what format do you think these numbers are stored? When you use one of these many planetarium types of software, how does it tell you where the moon will be at your lat/long at any given time? Do you think we have charts of the future location of the moon for every possible time at every possible location on the globe? For the stars, they mostly don't move, so just narrow it down to the location. If we have the position of every star in a big chart, how do they know where to draw that on your screen? They need to factor in your lat/long in some way right? How do you think they do that?
And you are aware that no CGI representation (utilizing KEPLER and NEWTON formulas and GR) of the movement of the EARTH, SOLAR SYSTEM, AND GALAXIES, exists?

How hard could it actually be to produce such a representation given it is strictly math and science claims to have the correct numbers?
Title: Re: Round Earth Celestial Mechanics Cannot Predict the Solar System
Post by: BillO on July 17, 2018, 12:18:28 PM
So basically your claim is, that because NOAA doesn't seem to use orbital mechanics for their calculations, the RE is false.

First up, the linked spreadsheet:
The Sun RAD Vector isn't used for the other calculations, it is an output to show the current distance to the sun.
All formulas in the sheet are simplified to the extreme, so that they are faster to work with, thereby they are also nearly impossible to apprehend.

Second:
You seem to assume that we would use orbital mechanics for our calculations, while I would expect that we would not.
Orbital mechanics are used for simulations, not as much for calculations. This is because orbital mechanics are extremely hard to calculate over time, where all planets and moons affect each other always, and affect each other more or less dependent on their current distance.
If you want orbital mechanics based on Newtonian physics: http://universesandbox.com/ it is actually pretty fun to use, and you can play with it all you wish. It is also very accurate (as long as you don't run the simulation too fast).

Third:
Stellarium is an excellent tool, it precisely shows all observations we see in the sky, and shows all predictions. Even if Stellarium is pattern based, it still shows all observations anywhere on earth which can all be verified. Now that you've brought it up, I would like to know if you acknowledge Stellarium as a valid source of observations, or if you have a problem with it?
Is there any observation in Stellarium which you know cannot be seen in reality?
Well said.  However, I made your first 2 points (plus a few more) already in a reply to Mr. Bishop and he ignored it.  However, I feel these objections need to be made at least as often as the erroneous 'analyses' are presented.
Title: Re: Round Earth Celestial Mechanics Cannot Predict the Solar System
Post by: Tumeni on July 17, 2018, 12:24:24 PM
you are aware that no CGI representation (utilizing KEPLER and NEWTON formulas and GR) of the movement of the EARTH, SOLAR SYSTEM, AND GALAXIES, exists?

Disagree. I've got one on my iPad for Earth and Solar System.
Title: Re: Round Earth Celestial Mechanics Cannot Predict the Solar System
Post by: totallackey on July 17, 2018, 12:48:46 PM
you are aware that no CGI representation (utilizing KEPLER and NEWTON formulas and GR) of the movement of the EARTH, SOLAR SYSTEM, AND GALAXIES, exists?

Disagree. I've got one on my iPad for Earth and Solar System.
No, you do not.

Provide the program code that provides clear, unadulterated evidence of the use of Kepler, Newton, and GR formulas in the source material.

Otherwise, apologize to the forum for making such a claim and immediately cease posting here on this forum.
Title: Re: Round Earth Celestial Mechanics Cannot Predict the Solar System
Post by: SphericalEarther on July 17, 2018, 01:14:46 PM
you are aware that no CGI representation (utilizing KEPLER and NEWTON formulas and GR) of the movement of the EARTH, SOLAR SYSTEM, AND GALAXIES, exists?

Disagree. I've got one on my iPad for Earth and Solar System.
No, you do not.

Provide the program code that provides clear, unadulterated evidence of the use of Kepler, Newton, and GR formulas in the source material.

Otherwise, apologize to the forum for making such a claim and immediately cease posting here on this forum.
You got him there, ay.

It would be impossible to make a simulation using all the methods simultaneously, as they overlap.

There are plenty of simulators that use the Newtonian physics to simulate the solar system.

Quick search finds a Java project simulating 24 solar systems using newtonian and GR:
https://github.com/nicokuijpers/SolarSystemSimulator

You wont find source code for the more advanced simulators, as people generally want to make money on the advanced stuff they create.

Besides, simulating our entire galaxy where we have very limited information about the makeup of planets is as good as impossible.
Simulating our solar system is easy, and really only requires Newtonian physics.

Scientists finally created a simulation recently, which could actually simulate an entire universe being created, forming galaxies similar to the forms we observe in our universe, though this simulation still can't form all the planets, it is a huge step forward.
Title: Re: Round Earth Celestial Mechanics Cannot Predict the Solar System
Post by: totallackey on July 17, 2018, 01:24:17 PM
you are aware that no CGI representation (utilizing KEPLER and NEWTON formulas and GR) of the movement of the EARTH, SOLAR SYSTEM, AND GALAXIES, exists?

Disagree. I've got one on my iPad for Earth and Solar System.
No, you do not.

Provide the program code that provides clear, unadulterated evidence of the use of Kepler, Newton, and GR formulas in the source material.

Otherwise, apologize to the forum for making such a claim and immediately cease posting here on this forum.
You got him there, ay.

It would be impossible to make a simulation using all the methods simultaneously, as they overlap.

There are plenty of simulators that use the Newtonian physics to simulate the solar system.

Quick search finds a Java project simulating 24 solar systems using newtonian and GR:
https://github.com/nicokuijpers/SolarSystemSimulator

You wont find source code for the more advanced simulators, as people generally want to make money on the advanced stuff they create.

Besides, simulating an entire galaxy where we have very limited information about the makeup of planets is as good as impossible.
Simulating our solar system is easy, and really only requires Newtonian physics.
And that is where all of RE gets the complete kabosh.

You guys are promoting a complete and utter pile of unadulterated HOGWASH, plain, pure, and simple!

All this mystical and accurate math crapola you constantly put forth, stating: "THE MATH IS CLEAR AND IN FULL SUPPORT OF A SPHERICAL EARTH!"

Yet, when pressed to provide a COMPUTER PROGRAM (you know computers, right? the machine that PROCESSES AND RENDERS IMAGES CORRECTLY ACCORDING TO MATH INPUTS!) UTILIZING your sacred gods of RE in the rendering, what do we have?

"You gotta be satisfied with this BS..."

YOUR OWN SOURCE STATES IT USES: "...Newton mechanics or General Relativity...", NOT BOTH!

LMAO!!! 
Title: Re: Round Earth Celestial Mechanics Cannot Predict the Solar System
Post by: SphericalEarther on July 17, 2018, 02:08:58 PM
And that is where all of RE gets the complete kabosh.

You guys are promoting a complete and utter pile of unadulterated HOGWASH, plain, pure, and simple!

All this mystical and accurate math crapola you constantly put forth, stating: "THE MATH IS CLEAR AND IN FULL SUPPORT OF A SPHERICAL EARTH!"

Yet, when pressed to provide a COMPUTER PROGRAM (you know computers, right? the machine that PROCESSES AND RENDERS IMAGES CORRECTLY ACCORDING TO MATH INPUTS!) UTILIZING your sacred gods of RE in the rendering, what do we have?

"You gotta be satisfied with this BS..."

YOUR OWN SOURCE STATES IT USES: "...Newton mechanics or General Relativity..., NOT BOTH!

LMAO!!!

I'm sure I just stated that they overlap:
Quote
It would be impossible to make a simulation using all the methods simultaneously, as they overlap.
Yeah I did.

The source probably uses newtonian physics for solar systems and general relativity for binding them together in a small galaxy cluster, it doesn't use both for the same things as that would be an overlap.
I'm sure I also stated it would be near impossible to get the source code for more advanced simulations.
I'm also sure I stated that for our own solar system, it is quite easy to simulate only using Newtonian physics, hell I can even create that myself.

Do you even know anything about programming?
Well I do, I know how 3d graphics work, I've created my own raytracing camera once which perfectly showed a perspective image, I've created a solar system with planets orbiting a sun, with moons orbiting the planets (not our own solar system though), using the very simple concept of gravity that FEers have an extremely hard time to grasp.
You demand simulations, you demand source code, you probably won't even understand the source code even if you got exactly what you asked for anyways.
Title: Re: Round Earth Celestial Mechanics Cannot Predict the Solar System
Post by: garygreen on July 17, 2018, 02:24:25 PM
It has also been alleged that the seemingly simple math used in Astronomical calculation textbooks are "really" based on keplerian orbital dynamics. This article addresses that.

I have collected a number of resources showing that the perverted myth that the Round Earth Theory has been validated, to be false. I ask that any challenger in opposition demonstrates with real evidence that astronomy can predict the motions of the planets as they are described in the Round Earth Theory. More evidence than a link to an obscure pdf or unverified model. It will need to be demonstrated that a model, according to the geometry of the heliocentric system, can predict any positions of the planets at all!

an ephemeris is a table of positions of celestial objects like stars and planets and such.  these tables are derived from models of planetary positions.  one of the most widely used is a model called VSOP87.  btw this is the model from which NASA's eclipse tables are derived.

Planetary theories in rectangular and spherical variables - VSOP 87 solutions: http://adsabs.harvard.edu/abs/1988A&A...202..309B
Numerical expressions for precession formulae and mean elements for the Moon and the planets: http://adsabs.harvard.edu/abs/1994A&A...282..663S

here's a more readable overview that includes source code and instructions for using VSOP87 output positions: http://neoprogrammics.com/vsop87/index.html

Quote
In the simplest terms, the VSOP87 theory can be described as a long-term mathematical model of the solar system, specifically, the orbits of the major planets from Mercury to Neptune.
...
To compute the apparent positions of the planets as viewed from Earth, we first need to know where the planets and the Earth are in their orbits at the same moment.  This is the first and most laborious step in computing the apparent geocentric positions of the planets.
...
All the VSOP87 series do essentially the same thing.  They compute the co-ordinates of the planets for a given moment.
...
These are the raw, instantaneous, geometric, heliocentric co-ordinates from which the apparent geocentric ecliptical and equatorial co-ordinates and distance are subsequently derived.  By geometric co-ordinates we mean that no corrections for the location of the observer have yet been applied, such as for parallax, light-time, aberration, etc.  This is the actual location of the planet in space, not its apparent location to the eye.
Title: Re: Round Earth Celestial Mechanics Cannot Predict the Solar System
Post by: totallackey on July 17, 2018, 02:40:34 PM
And that is where all of RE gets the complete kabosh.

You guys are promoting a complete and utter pile of unadulterated HOGWASH, plain, pure, and simple!

All this mystical and accurate math crapola you constantly put forth, stating: "THE MATH IS CLEAR AND IN FULL SUPPORT OF A SPHERICAL EARTH!"

Yet, when pressed to provide a COMPUTER PROGRAM (you know computers, right? the machine that PROCESSES AND RENDERS IMAGES CORRECTLY ACCORDING TO MATH INPUTS!) UTILIZING your sacred gods of RE in the rendering, what do we have?

"You gotta be satisfied with this BS..."

YOUR OWN SOURCE STATES IT USES: "...Newton mechanics or General Relativity..., NOT BOTH!

LMAO!!!

I'm sure I just stated that they overlap:
Quote
It would be impossible to make a simulation using all the methods simultaneously, as they overlap.
Yeah I did.

The source probably uses newtonian physics for solar systems and general relativity for binding them together in a small galaxy cluster, it doesn't use both for the same things as that would be an overlap.
I'm sure I also stated it would be near impossible to get the source code for more advanced simulations.
I'm also sure I stated that for our own solar system, it is quite easy to simulate only using Newtonian physics, hell I can even create that myself.

Do you even know anything about programming?
Well I do, I know how 3d graphics work, I've created my own raytracing camera once which perfectly showed a perspective image, I've created a solar system with planets orbiting a sun, with moons orbiting the planets (not our own solar system though), using the very simple concept of gravity that FEers have an extremely hard time to grasp.
You demand simulations, you demand source code, you probably won't even understand the source code even if you got exactly what you asked for anyways.
Listen, I understand enough to know this...

Your mad hatter scientists of your holy RE temples claim that Einstein GR, Kepler, and Newton math are in agreement and not "overlapping," as you deceivingly phrase it.

Your own source states it uses one or the other and not all in concert, as your own scientists state it must be.

It is patently evident you do not even understand your own writing and hence your claim you have created your own CGI models of the solar system based only on Newtonian physics is also a bunch of BS and you need to stop making such provably false claims on the internet.
Title: Re: Round Earth Celestial Mechanics Cannot Predict the Solar System
Post by: sandokhan on July 17, 2018, 02:55:51 PM
The orbital celestial equations of motion are a set of nonlinear ordinary differential equations with initial values.

Nonlinear ordinary differential equations are analyzed using bifurcation theory.

The hallmark of this analysis is the sensitive dependence on initial conditions.

'As Poincare experimented, he was relieved to discover that in most of
the situations, the possible orbits varied only slightly from the initial
2-body orbit, and were still stable, but what occurred during further
experimentation was a shock. Poincare discovered that even in some of the
smallest approximations some orbits behaved in an erratic unstable manner. His
calculations showed that even a minute gravitational pull from a third body
might cause a planet to wobble and fly out of orbit all together.'

Even measuring initial conditions of the system to an arbitrarily high, but finite accuracy, we will not be able to describe the system dynamics "at any time in the past or future". To predict the future of a chaotic system for arbitrarily long times, one would need to know the initial conditions with infinite accuracy, and this is by no means possible.

The Hamiltonian formulation of this set of nonlinear ordinary differential equations (mechanical system without friction) leads to the famous KAM theory.

Two of the greatest Soviet mathematicians of the 20th century, A.N. Kolmogorov and V.I. Arnold asked the following question: to what extent the geometric structure of the quasi-periodic dynamics of a Hamiltonian system persists under small perturbations that destroy the toroidal symmetry?

This led to the famous KAM theory (Kolmogorov-Arnold-Moser); however, it is valid for "sufficiently" small perturbations.

In reality, the perturbations in the solar system are far too large to apply KAM theory: the conditions of the KAM theorem are too strict to apply to a realistic solar system.

So, the mathematicians have to rely on computing Lyapunov exponents, in order to try to predict any region of instability/chaos.

Even in this case, the measured Lyapunov exponent may have no relation to the true exponent: great care has to be taken in computing such quantities.

In 1989, J. Laskar proudly announced that the exponential divergence time for the solar system is 5 million years.

However, again, this calculation DOES NOT take into account the sensitivity of the results due to uncertainties of the knowledge of true masses and the INITIAL CONDITIONS of the planets.


Jack Wisdom (MIT): It is not possible to exclude the possibility that the orbit of the Earth will suddenly exhibit similar wild excursions in eccentricity.

The exponential divergence of chaotic trajectories precludes long-term prediction given the limited knowledge of the state of our solar system.

Lyapunov exponents and symplectic integration.


Let d(t) be the distance between two solutions, with d(0) being their initial separation. Then d(t) increases approximately as d(0)eλt in a chaotic system, where λ is the Lyapunov exponent. The inverse of the Lyapunov exponent, 1/λ, is called the Lyapunov time, and measures how long it takes two nearby solutions to diverge by a factor of e.

Since the solar system is not integrable, and experiences unpredictable small perturbations, it cannot lie permanently on a KAM torus, and is thus chaotic.


Sussman and Wisdom's 1992 integration of the entire solar system displayed a disturbing dependence on the timestep of the integration (measurement of the Lyapunov time).


Thus, different researchers who draw their initial conditions from the same ephemeris at different times can find vastly different Lyapunov timescales.

Wayne Hayes, UC Irvine


To show the importance and the dependence on the sensitivity of the initial conditions of the set of differential equations, an error as small as 15 meters in measuring the position of the Earth today would make it impossible to predict where the Earth would be in its orbit in just over 100 million years' time.

A difference in the initial position of 1 cm grows to ∼1 AU (= 1.496 x 10^11 m) after 90–150 million years.


Let us take a closer look the chaotic dynamics of planetary formation; thus, a clear indication that the initial conditions cannot be predicted with accuracy (as we have seen, a mere 15 meters difference in the data will have catastrophic consequences upon the calculations).

OFFICIAL SCIENCE INFORMATION

Four stages of planetary formation

Initial stage: condensation and growth of grains in the hot nebular disk

Early stage: growth of grains to kilometer-sized planetesimals

Middle stage: agglomeration of planetesimals

Late stage: protoplanets


For the crucial stages, the initial and early stages, prediction becomes practically impossible.

As if this wasn't enough, we have absolute proof that in the age of modern man planet Earth underwent sudden pole shifts (heliocentrical version), thus making null and void any integration of the solar system/Lyapunov exponents calculations which do not take into account such variations of the system's parameters:

http://www.theflatearthsociety.org/forum/index.php?topic=30499.msg1635693#msg1635693

http://www.theflatearthsociety.org/forum/index.php?topic=30499.msg1546053#msg1546053


Let me show what sensitive dependence on initial conditions means, using one of the most famous examples: the Lorenz attractor butterfly effect.

In 1961, Lorenz was running a numerical computer model to redo a weather prediction from the middle of the previous run as a shortcut. He entered the initial condition 0.506 from the printout instead of entering the full precision 0.506127 value. The result was a completely different weather scenario.

Here is the set of Lorentz equations:

(https://upload.wikimedia.org/math/b/0/e/b0ea9119f6aaa31302164c4212cce72d.png)

Now, the set of differential equations which describes the planetary orbits is much more complicated than this.



Numerical expressions for precession formulae and mean elements for the Moon and the planets: http://adsabs.harvard.edu/abs/1994A&A...282..663S

Laskar's calculations are useless without specifying the initial conditions and parameters of the equations:

https://www.theflatearthsociety.org/forum/index.php?topic=30499.msg1774581#msg1774581


The best that modern astronomy can claim is a 300 year limit on the stability of the solar system, using Newcomb's series:

https://www.theflatearthsociety.org/forum/index.php?topic=30499.msg1935048#msg1935048

And that hypothesis does not take into account the fact that the initial conditions cannot be specified at all for the orbital equations in question.

Title: Re: Round Earth Celestial Mechanics Cannot Predict the Solar System
Post by: SphericalEarther on July 17, 2018, 03:03:01 PM
And that is where all of RE gets the complete kabosh.

You guys are promoting a complete and utter pile of unadulterated HOGWASH, plain, pure, and simple!

All this mystical and accurate math crapola you constantly put forth, stating: "THE MATH IS CLEAR AND IN FULL SUPPORT OF A SPHERICAL EARTH!"

Yet, when pressed to provide a COMPUTER PROGRAM (you know computers, right? the machine that PROCESSES AND RENDERS IMAGES CORRECTLY ACCORDING TO MATH INPUTS!) UTILIZING your sacred gods of RE in the rendering, what do we have?

"You gotta be satisfied with this BS..."

YOUR OWN SOURCE STATES IT USES: "...Newton mechanics or General Relativity..., NOT BOTH!

LMAO!!!

I'm sure I just stated that they overlap:
Quote
It would be impossible to make a simulation using all the methods simultaneously, as they overlap.
Yeah I did.

The source probably uses newtonian physics for solar systems and general relativity for binding them together in a small galaxy cluster, it doesn't use both for the same things as that would be an overlap.
I'm sure I also stated it would be near impossible to get the source code for more advanced simulations.
I'm also sure I stated that for our own solar system, it is quite easy to simulate only using Newtonian physics, hell I can even create that myself.

Do you even know anything about programming?
Well I do, I know how 3d graphics work, I've created my own raytracing camera once which perfectly showed a perspective image, I've created a solar system with planets orbiting a sun, with moons orbiting the planets (not our own solar system though), using the very simple concept of gravity that FEers have an extremely hard time to grasp.
You demand simulations, you demand source code, you probably won't even understand the source code even if you got exactly what you asked for anyways.
Listen, I understand enough to know this...

Your mad hatter scientists of your holy RE temples claim that Einstein GR, Kepler, and Newton math are in agreement and not "overlapping," as you deceivingly phrase it.

Your own source states it uses one or the other and not all in concert, as your own scientists state it must be.

It is patently evident you do not even understand your own writing and hence your claim you have created your own CGI models of the solar system based only on Newtonian physics is also a bunch of BS and you need to stop making such provably false claims on the internet.

All I needed to use was this:
Quote
The formula is F = G*((m sub 1*m sub 2)/r^2), where F is the force of attraction between the two bodies, G is the universal gravitational constant, m sub 1 is the mass of the first object, m sub 2 is the mass of the second object and r is the distance between the centers of each object.
Newtons law of gravity, simple as that.

As stated, I didn't simulate our own solar system, but if I had placed the planets correctly, given them correct mass, and given them their initial speed, I would have created a model of our solar system.
Then I would need to apply spin to the objects aswell as light from the sun and perhaps a static star background, and I would then be able to see all our observations in the sky.

That is all it takes, and there are many simulations out there of our solar system, since it is so easy to make.
Title: Re: Round Earth Celestial Mechanics Cannot Predict the Solar System
Post by: totallackey on July 17, 2018, 03:15:41 PM
And that is where all of RE gets the complete kabosh.

You guys are promoting a complete and utter pile of unadulterated HOGWASH, plain, pure, and simple!

All this mystical and accurate math crapola you constantly put forth, stating: "THE MATH IS CLEAR AND IN FULL SUPPORT OF A SPHERICAL EARTH!"

Yet, when pressed to provide a COMPUTER PROGRAM (you know computers, right? the machine that PROCESSES AND RENDERS IMAGES CORRECTLY ACCORDING TO MATH INPUTS!) UTILIZING your sacred gods of RE in the rendering, what do we have?

"You gotta be satisfied with this BS..."

YOUR OWN SOURCE STATES IT USES: "...Newton mechanics or General Relativity..., NOT BOTH!

LMAO!!!

I'm sure I just stated that they overlap:
Quote
It would be impossible to make a simulation using all the methods simultaneously, as they overlap.
Yeah I did.

The source probably uses newtonian physics for solar systems and general relativity for binding them together in a small galaxy cluster, it doesn't use both for the same things as that would be an overlap.
I'm sure I also stated it would be near impossible to get the source code for more advanced simulations.
I'm also sure I stated that for our own solar system, it is quite easy to simulate only using Newtonian physics, hell I can even create that myself.

Do you even know anything about programming?
Well I do, I know how 3d graphics work, I've created my own raytracing camera once which perfectly showed a perspective image, I've created a solar system with planets orbiting a sun, with moons orbiting the planets (not our own solar system though), using the very simple concept of gravity that FEers have an extremely hard time to grasp.
You demand simulations, you demand source code, you probably won't even understand the source code even if you got exactly what you asked for anyways.
Listen, I understand enough to know this...

Your mad hatter scientists of your holy RE temples claim that Einstein GR, Kepler, and Newton math are in agreement and not "overlapping," as you deceivingly phrase it.

Your own source states it uses one or the other and not all in concert, as your own scientists state it must be.

It is patently evident you do not even understand your own writing and hence your claim you have created your own CGI models of the solar system based only on Newtonian physics is also a bunch of BS and you need to stop making such provably false claims on the internet.

All I needed to use was this:
Quote
The formula is F = G*((m sub 1*m sub 2)/r^2), where F is the force of attraction between the two bodies, G is the universal gravitational constant, m sub 1 is the mass of the first object, m sub 2 is the mass of the second object and r is the distance between the centers of each object.
Newtons law of gravity, simple as that.

As stated, I didn't simulate our own solar system, but if I had placed the planets correctly, given them correct mass, and given them their initial speed, I would have created a model of our solar system.
Then I would need to apply spin to the objects aswell as light from the sun and perhaps a static star background, and I would then be able to see all our observations in the sky.

That is all it takes, and there are many simulations out there of our solar system, since it is so easy to make.
Blah, blah, blah...

Take notice, you have produced NOTHING!

You can write about it all you want...you can clamor on and on and on...

In the end, even if you could do as you claim, your purty model would only be based on Newtonian physics (per your own claim) and include nothing of Kepler or Einstein.

If it was easy to make, we would already have it.

We do not.

Therefore, the science you proudly support and claim as gospel is INCORRECT and a bald-faced lie!
Title: Re: Round Earth Celestial Mechanics Cannot Predict the Solar System
Post by: sandokhan on July 17, 2018, 03:17:41 PM
Newtons law of gravity, simple as that.

The greatest mathematician and physicist of the 18th century, Leonhard Euler, dismissed the attractive law of gravity:

“Now, in whatever way we imagine the cause of gravity, as it is the effect of the pressure of a fluid, the force with which each molecule is pushed will always be proportional to the extension or the volume of that molecule. Indeed it is a general rule of hydrostatics that fluids act according to the volumes: a body immersed in water is always pushed by a force equal to the weight of an equal volume of water, but in an opposite direction.”

“the matter which constitutes the subtle fluid, cause of the gravity, is of an utterly different nature from the matter, of which all sensible bodies are composed. There will hence be two kinds of matter, one which provides the stuff to all sensible bodies, and of which all particles have the same [high] density [...]; the other kind of matter will be that of which the subtle fluid, which causes gravity, and which we name ether, is composed of. It is probable that this matter has always the same degree of density, but that this degree is incomparably smaller than that of the first kind.”

L. Euler, “Recherches physiques sur la nature des moindres parties de la matiere,” in Leonhardi Euleri Opera Omnia, Series Tertia, Pars Prima (B. G. Teubner, Leipzig and Bern 1911), pp. 3–15

“Those who attribute gravity to an attractive force of the Earth base their opinion mainly on the fact that otherwise no origin could be displayed for this force. But since we proved that all bodies are surrounded with ether and are pressed by the elastic force of the latter, we do not need to search elsewhere the origin of gravity. Only if the pressure of the ether would be everywhere the same, which assignment is indistinguishable from that of its equilibrium, would the bodies be equally pressed from every side, and thus would not be induced in any motion. But if we assume that the ether around the Earth is not in equilibrium, and that instead its pressure becomes smaller as one comes closer to the Earth, then any given body must experience a stronger pressure downwards on its superior surface that it does upwards on its inferior surface; it follows that the downwards pressure will have the advantage and hence that the body will really be pushed downwards, which effect we call gravity, and the downwards-pushing force the weight of the body.”

L. Euler, “Von der Schwere und den Kraften so auf die himmlischen Korper wirken,” in Leonhardi Euleri Opera Omnia, Series Tertia, Pars Prima (B. G. Teubner, Leipzig and Bern 1911), pp. 149–156

(translation by Dr. M. Arminjon)
Title: Re: Round Earth Celestial Mechanics Cannot Predict the Solar System
Post by: SphericalEarther on July 17, 2018, 06:29:26 PM
Blah, blah, blah...

Take notice, you have produced NOTHING!

You can write about it all you want...you can clamor on and on and on...

In the end, even if you could do as you claim, your purty model would only be based on Newtonian physics (per your own claim) and include nothing of Kepler or Einstein.

If it was easy to make, we would already have it.

We do not.

Therefore, the science you proudly support and claim as gospel is INCORRECT and a bald-faced lie!
Why do you claim that I need all Kepler, Newtonian and Einstein physics? Why should we use all together? Explain what Kepler provides that Newtonian does not, explain why we need Einstein to simulate our tiny solar system?

You demand as much as you possibly can, because your only goal is to proclaim it is impossible. You demand without putting in any of the effort you require of the opposition. I've met the demands of several FEers, provided 3d models and all, but I would never do it for a demanding person as you.
Title: Re: Round Earth Celestial Mechanics Cannot Predict the Solar System
Post by: totallackey on July 17, 2018, 07:47:41 PM
Blah, blah, blah...

Take notice, you have produced NOTHING!

You can write about it all you want...you can clamor on and on and on...

In the end, even if you could do as you claim, your purty model would only be based on Newtonian physics (per your own claim) and include nothing of Kepler or Einstein.

If it was easy to make, we would already have it.

We do not.

Therefore, the science you proudly support and claim as gospel is INCORRECT and a bald-faced lie!
Why do you claim that I need all Kepler, Newtonian and Einstein physics? Why should we use all together? Explain what Kepler provides that Newtonian does not, explain why we need Einstein to simulate our tiny solar system?

You demand as much as you possibly can, because your only goal is to proclaim it is impossible. You demand without putting in any of the effort you require of the opposition. I've met the demands of several FEers, provided 3d models and all, but I would never do it for a demanding person as you.
Do it for your fellow RE adherent then...

You are aware that Kepler and Newton used empirical observations of the astronomical bodies to derive their models right? You're probably aware that Newton was left perplexed with a tiny deviation in the behavior of Mercury that didn't match his model. And you may have heard the story of how Einstein finally fixed the model for Mercury with GR.
As you can see, Newton by his lonesome, Kepler by his lonesome, and Einstein by his lonesome does not cut the mustard...

Like I wrote earlier, you got bupkus...
Title: Re: Round Earth Celestial Mechanics Cannot Predict the Solar System
Post by: Rama Set on July 17, 2018, 09:07:10 PM
Blah, blah, blah...

Take notice, you have produced NOTHING!

You can write about it all you want...you can clamor on and on and on...

In the end, even if you could do as you claim, your purty model would only be based on Newtonian physics (per your own claim) and include nothing of Kepler or Einstein.

If it was easy to make, we would already have it.

We do not.

Therefore, the science you proudly support and claim as gospel is INCORRECT and a bald-faced lie!
Why do you claim that I need all Kepler, Newtonian and Einstein physics? Why should we use all together? Explain what Kepler provides that Newtonian does not, explain why we need Einstein to simulate our tiny solar system?

You demand as much as you possibly can, because your only goal is to proclaim it is impossible. You demand without putting in any of the effort you require of the opposition. I've met the demands of several FEers, provided 3d models and all, but I would never do it for a demanding person as you.
Do it for your fellow RE adherent then...

You are aware that Kepler and Newton used empirical observations of the astronomical bodies to derive their models right? You're probably aware that Newton was left perplexed with a tiny deviation in the behavior of Mercury that didn't match his model. And you may have heard the story of how Einstein finally fixed the model for Mercury with GR.
As you can see, Newton by his lonesome, Kepler by his lonesome, and Einstein by his lonesome does not cut the mustard...

Like I wrote earlier, you got bupkus...

You apparently misunderstand the content of these theories.  Kepler's laws can be derived from Newton's laws, they are essentially equivalent.  Any calculation you wish to produce with either Newton's or Kepler's laws can be produced with GR.  GR is the only theory that you need request.  If you go back through the forum history, you will see that I provided with documentation on a model of the solar system that used GR, so perhaps go have a look at that and then come back and discuss?
Title: Re: Round Earth Celestial Mechanics Cannot Predict the Solar System
Post by: Tom Bishop on July 17, 2018, 10:03:13 PM
It has also been alleged that the seemingly simple math used in Astronomical calculation textbooks are "really" based on keplerian orbital dynamics. This article addresses that.

I have collected a number of resources showing that the perverted myth that the Round Earth Theory has been validated, to be false. I ask that any challenger in opposition demonstrates with real evidence that astronomy can predict the motions of the planets as they are described in the Round Earth Theory. More evidence than a link to an obscure pdf or unverified model. It will need to be demonstrated that a model, according to the geometry of the heliocentric system, can predict any positions of the planets at all!

an ephemeris is a table of positions of celestial objects like stars and planets and such.  these tables are derived from models of planetary positions.  one of the most widely used is a model called VSOP87.  btw this is the model from which NASA's eclipse tables are derived.

Planetary theories in rectangular and spherical variables - VSOP 87 solutions: http://adsabs.harvard.edu/abs/1988A&A...202..309B
Numerical expressions for precession formulae and mean elements for the Moon and the planets: http://adsabs.harvard.edu/abs/1994A&A...282..663S

here's a more readable overview that includes source code and instructions for using VSOP87 output positions: http://neoprogrammics.com/vsop87/index.html

Quote
In the simplest terms, the VSOP87 theory can be described as a long-term mathematical model of the solar system, specifically, the orbits of the major planets from Mercury to Neptune.
...
To compute the apparent positions of the planets as viewed from Earth, we first need to know where the planets and the Earth are in their orbits at the same moment.  This is the first and most laborious step in computing the apparent geocentric positions of the planets.
...
All the VSOP87 series do essentially the same thing.  They compute the co-ordinates of the planets for a given moment.
...
These are the raw, instantaneous, geometric, heliocentric co-ordinates from which the apparent geocentric ecliptical and equatorial co-ordinates and distance are subsequently derived.  By geometric co-ordinates we mean that no corrections for the location of the observer have yet been applied, such as for parallax, light-time, aberration, etc.  This is the actual location of the planet in space, not its apparent location to the eye.

From what I have read about VSOP the designers have done the following:

Step 1: Take tables of positions of body in the sky, its patterns and trends
 |
 |--Step 2: Convert position in the sky to a presumed heliocentric value
      \_ Step 2a: Try to predict what the heliocentric position of the body will be in the future
            \_Optional Step: Publish the "known" details of the body about the properties of its heliocentric "orbit" in books and teach it to school children.

Step 3: Create a coordinate for the body in the sky to be found on a future date.

The question is, Is Step 3 based on the data from Step 1 or Step 2?

Since the analytical Three Body Problem is impossible, and its limited numerical and restricted solutions make crazy paths (as seen in the article in the OP), let alone a system with dozens of bodies, all of which would cause crazy chaos, and the simplest of motions in the simplest system on paper being beyond the ability of the greatest minds of human history, I would say that Step 3 is likely based on Step 1.

I will continue to look into it, however, and try to point it out.
Title: Re: Round Earth Celestial Mechanics Cannot Predict the Solar System
Post by: Tom Bishop on July 17, 2018, 11:33:23 PM
VSOP may be a NASA thing, so tricks may be involved when analyzing this.

We read the following at http://www.lizard-tail.com/isana/lab/starlitnight/extra/vsop87/vsop87.rb

Quote
The main version of VSOP87 is similar to the previous theory VSOP82.
In the both cases the constants of integration have been determined by
fitting to the numerical integration DE200 of the Jet Propulsion Laboratory.

JPL is a NASA facility. Then, when looking at the DE200 wiki page:

https://en.wikipedia.org/wiki/Jet_Propulsion_Laboratory_Development_Ephemeris

Quote
The method of special perturbations was applied, using numerical integration to solve the n-body problem, in effect putting the entire Solar System into motion in the computer's memory, accounting for all relevant physical laws

Interesting!

The icing on the cake:

Quote
Available documentation is sketchy

Yes, quite curious, indeed, for something that has solved the most vexing problems in mathematics, physics, and astronomy for hundreds of years.

Who was it that won the Nobel Prize for that?

I now go to download this wonder.

ftp://ssd.jpl.nasa.gov/pub/eph/planets/README.txt

Oh, darn it. They are only giving out the planetary position data in the sky. It seems that this wonder of Astronomy, Mathematics, Classical Mechanics, Celestial Mechanics, and the solution to Chaos Theory itself, will have to stay locked away from the world at the JPL.  :(
Title: Re: Round Earth Celestial Mechanics Cannot Predict the Solar System
Post by: Tom Bishop on July 18, 2018, 01:51:20 AM
https://www.newscientist.com/article/2148074-infamous-three-body-problem-has-over-a-thousand-new-solutions/

Quote
Infamous three-body problem has over a thousand new solutions

For more than 300 years, mathematicians have puzzled over the three-body problem – the question of how three objects orbit one another according to Newton’s laws. Now, there are 1223 new solutions to the conundrum, more than doubling the current number of possibilities.

No single equation can predict how three bodies will move in relation to one another and whether their orbits will repeat or devolve into chaos. Mathematicians must test each specific scenario to see if the objects will stay bound in orbit or be flung away.

The new solutions were found when researchers at Shanghai Jiaotong University in China tested 16 million different orbits using a supercomputer.

All the fresh orbits found are periodic. This means that each object, whether it’s a planet or a proton, ends up where it first began its orbit, with their paths forming three intertwined, closed loops.

“It is impressive that they’ve made the list a lot longer,” says Robert Vanderbei at Princeton University in New Jersey – though he adds that there is “basically an unlimited number of orbits”, so it may be overkill if anyone sought to find them all.

Perhaps the most important application of the three-body problem is in astronomy, for helping researchers figure out how three stars, a star with a planet that has a moon, or any other set of three celestial objects can maintain a stable orbit.

But these new orbits rely on conditions that are somewhere between unlikely and impossible for a real system to satisfy. In all of them, for example, two of the three bodies have exactly the same mass and they all remain in the same plane.

Knot-like paths

In addition, the researchers did not test the orbits’ stability. It’s possible that the tiniest disturbance in space or rounding error in the equations could rip the objects away from one another.

“These orbits have nothing to do with astronomy, but you’re solving these equations and you’re getting something beautiful,” says Vanderbei.

Aside from giving us a thousand pretty pictures of knot-like orbital paths, the new three-body solutions also mark a starting point for finding even more possible orbits, and eventually figuring out the whole range of winding paths that three objects can follow around one another.

This is kind of the zeroth step. Then the question becomes, how is the space of all possible positions and velocities filled up by solutions?” says Richard Montgomery at the University of California, Santa Cruz. “These simple orbits are kind of like a skeleton to build the whole system up from.”

Gee, someone had better tell them that NASA already has the Three Body Problem all figured out and can perfectly model the Solar System.
Title: Re: Round Earth Celestial Mechanics Cannot Predict the Solar System
Post by: garygreen on July 18, 2018, 02:57:22 AM
From what I have read about VSOP the designers have done the following:
then you didn't read any of it very carefully.  you don't have to read all of it.  start with the abstracts.  or read the quotes i already posted.

peruse sections 2, 3, and 4.1.  vsop is a semi-analytic model of the solar system that takes as its input the orbital parameters of the planets (semi-major axis, distance from sun, eccentricity, inclination, etc.) and outputs their heliocentric positions in cartesian coordinates.  section 4.1 even explicitly says "We must first solve Kepler's equation in order to get the expressions of the variables X, Y, Z."

stuff about n-body problems
you don't understand the "problem" of the n-body problem in physics.  it's not that these problems are intractable or unsolvable; it's that they don't have analytic solutions.  that's not the same as saying they can't be solved.  it just means that there's no y=f(x) type expression that will tell you exactly how the system will evolve to an arbitrary level of precision.

let's suppose i have a proton and an electron some distance from one another.  there is an analytic solution, coulomb's law, that will tell me the force on each particle.  there's another analytic solution, newton's 420th law, that will tell me the acceleration of each particle, and so on and so on.

now let's suppose i have 69 particles (some protons, some electrons) arbitrarily arranged in space.  there is no analytic solution that will tell me exactly how the system will evolve over time.  but, since i can find the force between two particles, i can solve my analytic expression for every pair of particles and sum them all up.  i let my system evolve over a tiny time-step, then calculate the forces again, then let my system evolve a bit, etc.  that's what a numerical integration is. 

for example:
https://arxiv.org/abs/astro-ph/0111045
http://lnfm1.sai.msu.ru/~rastor/Books/Heggie_&_Hut-Gravitational_Million_Body_Problem.pdf
Title: Re: Round Earth Celestial Mechanics Cannot Predict the Solar System
Post by: Tom Bishop on July 18, 2018, 03:48:39 AM
Gary, did you see those knotted path the moon took around the earth in my Celestial Mechanics article under Best of the Best (https://wiki.tfes.org/Celestial_Mechanics_Cannot_Predict_The_Solar_System#The_Best_of_the_Best)?

Those are numerical solutions to the Three Body Problem in Celestial Mechanics as applied to the Earth-Moon-Sun system. Those are numerical methods with thousands of computed steps. That is the best they can do. Astronomers are unable to make a stable three body problem that looks anything like what the heliocentric system suggests.

The best that can be done are numerically computed orbits like this:

http://fem.um.es/Ejs/CRC_examples/Simulations/ArenstorfOrbit_Arenstorf_Orbits.html

Quote
(http://fem.um.es/Ejs/CRC_examples/Simulations/_examples/CRCDemo/images/Arenstorf.gif)

"Arenstorf orbits are closed trajectories of the restricted three-body problem. ... The computation of these orbits is very sensible to small errors and are a good test for the accuracy of numerical methods for solving Ordinary Differential Equations."

As the article Infamous three-body problem has over a thousand new solutions (https://www.newscientist.com/article/2148074-infamous-three-body-problem-has-over-a-thousand-new-solutions/) states, astronomers and mathematicians are still at the "zeroth step." They cannot make a stable heliocentric system with numerical three body problem solutions.

The stable orbits they compute numerically with a super computer produces nothing but a mess of knotty orbits.

Yet NASA has a secret working model of the Solar System, the heliocentric solution to the n-body problems, which can simulate all physical laws of the Solar System, which they are keeping privately to themselves?
Title: Re: Round Earth Celestial Mechanics Cannot Predict the Solar System
Post by: Tom Bishop on July 18, 2018, 04:15:29 AM
From what I have read about VSOP the designers have done the following:
then you didn't read any of it very carefully.  you don't have to read all of it.  start with the abstracts.  or read the quotes i already posted.

peruse sections 2, 3, and 4.1.  vsop is a semi-analytic model of the solar system that takes as its input the orbital parameters of the planets (semi-major axis, distance from sun, eccentricity, inclination, etc.) and outputs their heliocentric positions in cartesian coordinates.  section 4.1 even explicitly says "We must first solve Kepler's equation in order to get the expressions of the variables X, Y, Z."

VOPS may produce some heliocentric interpretations. But what evidence is there to suggest that they are using those heliocentric concepts to predict the position of bodies in the sky?

There needs to be validation. How do you know that any prediction based on VOPS is done so based on heliocentric coordinates, rather than the geocentric (as seen from the earth) patterns the VOPS application is making those heliocentric coordinates from?

This program seems only good enough to tell us a few things about how a few things should be under the Heliocentric System, which astronomers then proceed to publish as fact about that model.

The numerous n-body problems in celestial mechanics shows that this application cannot be predicting future events based on the motions of a heliocentric system.
Title: Re: Round Earth Celestial Mechanics Cannot Predict the Solar System
Post by: Tumeni on July 18, 2018, 07:42:52 AM
This page was published in advance of the 2017 solar eclipse.

https://eclipses.gsfc.nasa.gov/SEmono/TSE2017/TSE2017.html (https://eclipses.gsfc.nasa.gov/SEmono/TSE2017/TSE2017.html)

It includes this graphic.

(https://eclipses.gsfc.nasa.gov/SEmono/TSE2017/TSE2017fig/TSE2017-1.gif)

Note the times included, how it divides the eclipse up into 10 minute segments, how those segments vary in size across the surface, how the percentage of eclipse varies according to latitude, how this varies between northern and southern hemispheres since the subsolar point is above the equator, and, most notably - how all this is taken into account according to the globe.

The eclipse occurred, exactly as predicted, bang on time, in exactly the predicted place. Nobody saw anything which varied from this.

How would this happen, other than by the mechanics of a globe being taken into account?
Title: Re: Round Earth Celestial Mechanics Cannot Predict the Solar System
Post by: Tumeni on July 18, 2018, 10:17:16 AM
The upcoming 'blood moon' eclipse will be visible to those in the African/Indian region, primarily. I'll get a limited view of it, since I'm in the UK, but America will not see it at all.

(https://cdn.images.express.co.uk/img/dynamic/151/590x/secondary/Eclipse-2018-path-totality-Blood-Moon-map-July-total-lunar-eclipse-1423438.jpg?r=1531824454234)

Tom, why would this be the case, but for the fact that America is on the wrong side of the globe to see it?

If you can explain it any other way, please do.
Title: Re: Round Earth Celestial Mechanics Cannot Predict the Solar System
Post by: totallackey on July 18, 2018, 10:51:10 AM
You apparently misunderstand the content of these theories.  Kepler's laws can be derived from Newton's laws, they are essentially equivalent.  Any calculation you wish to produce with either Newton's or Kepler's laws can be produced with GR.  GR is the only theory that you need request.  If you go back through the forum history, you will see that I provided with documentation on a model of the solar system that used GR, so perhaps go have a look at that and then come back and discuss?
Maybe you can actually produce what you state you have and then come back and discuss.

If GR was all there was to it, then Newton and Kepler go bye-bye...

Did you see anyone waving with a tear in their eye?
Title: Re: Round Earth Celestial Mechanics Cannot Predict the Solar System
Post by: SphericalEarther on July 18, 2018, 11:34:32 AM
You apparently misunderstand the content of these theories.  Kepler's laws can be derived from Newton's laws, they are essentially equivalent.  Any calculation you wish to produce with either Newton's or Kepler's laws can be produced with GR.  GR is the only theory that you need request.  If you go back through the forum history, you will see that I provided with documentation on a model of the solar system that used GR, so perhaps go have a look at that and then come back and discuss?
Maybe you can actually produce what you state you have and then come back and discuss.

If GR was all there was to it, then Newton and Kepler go bye-bye...

Did you see anyone waving with a tear in their eye?
You seriously have no idea what you are talking about.
Newtons law of gravity works.
Einsteins GR generalized Newtons law with his theory on Special Relativity (spacetime), providing answers to issues observed while only using Newtons law of gravity.

We still do not need GR to simulate our solar system, but it easily can and is way more complicated to implement for something which doesn't really change the outcome in our small solar system.

Let me give a similar example:
When we throw objects, if we remove the friction of air, we generally calculate the trajectory via a parabola and this works fine.
However when we throw the ball, the ball actually follows an orbital path, which just so happens to intersect the earth because we didn't throw it very far.
Correctly, we should use math for an orbital path, but we use math for a parabola because there is only a minimal difference which is basically undetectable, and calculating a parabola is way easier than calculating an orbit.
Title: Re: Round Earth Celestial Mechanics Cannot Predict the Solar System
Post by: Rama Set on July 18, 2018, 12:08:56 PM
You apparently misunderstand the content of these theories.  Kepler's laws can be derived from Newton's laws, they are essentially equivalent.  Any calculation you wish to produce with either Newton's or Kepler's laws can be produced with GR.  GR is the only theory that you need request.  If you go back through the forum history, you will see that I provided with documentation on a model of the solar system that used GR, so perhaps go have a look at that and then come back and discuss?
Maybe you can actually produce what you state you have and then come back and discuss.

Why should I do the same thing for you twice because you don’t want to look?

Quote
If GR was all there was to it, then Newton and Kepler go bye-bye...

Did you see anyone waving with a tear in their eye?

No, again you are not displaying even basic levels of knowledge. Kepler and Newton are much easier to use and for most problems is sufficient
Title: Re: Round Earth Celestial Mechanics Cannot Predict the Solar System
Post by: totallackey on July 18, 2018, 03:37:45 PM
You apparently misunderstand the content of these theories.  Kepler's laws can be derived from Newton's laws, they are essentially equivalent.  Any calculation you wish to produce with either Newton's or Kepler's laws can be produced with GR.  GR is the only theory that you need request.  If you go back through the forum history, you will see that I provided with documentation on a model of the solar system that used GR, so perhaps go have a look at that and then come back and discuss?
Maybe you can actually produce what you state you have and then come back and discuss.
Why should I do the same thing for you twice because you don’t want to look?
The record is very clear.

You produced nothing at all.
No, again you are not displaying even basic levels of knowledge. Kepler and Newton are much easier to use and for most problems is sufficient
LOL!

Kepler and Newton needed Einstein (supposedly) to solve the issues with Mercury (in reality, there is no issue with Mercury of course as heliocentricity is an outright lie).
Title: Re: Round Earth Celestial Mechanics Cannot Predict the Solar System
Post by: Rama Set on July 18, 2018, 04:44:58 PM
Why should I do the same thing for you twice because you don’t want to look?
The record is very clear.

You produced nothing at all. [/quote]

You ignoring something does not make it not exist.

Quote
LOL!

Kepler and Newton needed Einstein to solve the issues with Mercury.

Nothing I said disagrees with this.  Did you understand what I said?  It doesn't seem like it.
Title: Re: Round Earth Celestial Mechanics Cannot Predict the Solar System
Post by: garygreen on July 18, 2018, 08:43:32 PM
Yet NASA has a secret working model of the Solar System, the heliocentric solution to the n-body problems, which can simulate all physical laws of the Solar System, which they are keeping privately to themselves?

pretty much everything you say in this post is dead wrong.  also no one is keeping secret models of the solar system.  my first post's last link shows you how to obtain the source code for vosp87 and run it yourself.

VOPS may produce some heliocentric interpretations. But what evidence is there to suggest that they are using those heliocentric concepts to predict the position of bodies in the sky?

There needs to be validation. How do you know that any prediction based on VOPS is done so based on heliocentric coordinates, rather than the geocentric (as seen from the earth) patterns the VOPS application is making those heliocentric coordinates from?

This program seems only good enough to tell us a few things about how a few things should be under the Heliocentric System, which astronomers then proceed to publish as fact about that model.

The numerous n-body problems in celestial mechanics shows that this application cannot be predicting future events based on the motions of a heliocentric system.

this is all completely wrong.  i wish you would bother to read the material i post instead of just making up your own version of things to critique.
Title: Re: Round Earth Celestial Mechanics Cannot Predict the Solar System
Post by: Tom Bishop on July 18, 2018, 09:11:34 PM
Gary, firstly, the full source code isn't found on that website you linked. When searching google, I found the following:

http://www.freevbcode.com/ShowCode.asp?ID=464

Quote
VSOP87 Functions used to Compute Planetary Positions (1.15 MB)

These functions are a VB version of the complete VSOP87 planetary theory designed to be used to in a program to compute the heliocentric ecliptic longitude, latitude and distance of the planets Mercury to Neptune over a period of several thousands of years to about 1 arcsecond of precision. They are intended for use by programmers desiring to make their own astronomical computations programs.

Can it do anything more than compute the presumed heliocentric elements?

I downloaded it and read the following from the about page:

Quote
These Visual BASIC program modules form a nucleus around which to build a
sophisticated planetary ephemeris program.

They compose the essential subroutines to compute the heliocentric, ecliptical
coordinates, L, B and R according to the full VSOP87 theory.

Nothing about running it into the future and then converting to positions in the sky.

VSOP is based on Astronomical Algorithms. I discussed this book in the NOAA Solar Calculator (https://wiki.tfes.org/NOAA_Solar_Calculator) page on the wiki. We know that Astronomical Algorithms is calculating R, the AU, for the planets under the heliocentric model, and some other elements, but then it stops there. The NOAA Solar Calculator Excel Worksheet, that is based on the same Astronomical Algorithms book, shows that the "Sun Rad Vector (AUs)," which is the distance between the the Earth and the Sun and the same as R, doesn't do anything, and is only an output value. It can be deleted from the worksheet entirely and the computations still work -- truly a demonstration of what Astronomical Algorithms is all about.

Where is the part about converting the heliocentric model to what is seen in the sky and accuracy of planet positions?

If the above software package is based on the complete VSOP theory, as it claims to be, then neither the description or about page shows that VSOP does anything more than calculate what the presumed heliocentric elements are.

I have also read that VSOP isn't even really converting to a real heliocentric model, but I will put that aside for now and come back to it later.
Title: Re: Round Earth Celestial Mechanics Cannot Predict the Solar System
Post by: garygreen on July 18, 2018, 11:40:32 PM
Can it do anything more than compute the presumed heliocentric elements?

what? whether or not it can computer the heliocentric coordinates is the thing we're arguing about.  your argument is that astronomers do not use a physical model of the planets to make predictions.  my argument is that vsop87 is exactly one such model.  it is a model of the solar system that predicts the positions of the planets in 3d space.  it's not merely a list of patterns.  the user inputs a time (eg i want to know where mars is in the solar system on april 20th, 2069), and vsop87 outputs heliocentric coordinates.

here's another astronomy site that gives a step-by-step tutorial on how to use vsop87: https://www.caglow.com/info/compute/vsop87
Quote
VSOP87 provides a method for computing the positions of the 8 planets (and the Sun) efficiently and accurately without the major headaches that astronomers from past centuries had to deal with.
...
It consists of a large number of periodic terms that are then added up together in a special way to produce the 3-dimensional heliocentric coordinates of any planet at any moment in time for thousands of years into the future and the past. These coordinates can then be converted through a few transformations into geocentric coordinates which can be used to show their position as seen from Earth.
...
The results of VSOP87C above are heliocentric coordinates, centered on the Sun. Since none of us actually live on the Sun, it might be more useful to obtain geocentric coordinates, centered on the Earth. Using rectangular coordinates, a simple transformation is all that is needed to convert.

Title: Re: Round Earth Celestial Mechanics Cannot Predict the Solar System
Post by: Tom Bishop on July 19, 2018, 06:21:05 AM
I am writing an article about VSOP. I have found some further information on how it works and am putting together an analysis.

I looked at your "it uses kepler" reference and found that the article you quoted is saying that the authors are trying to use kepler equations based on the data from VSOP to do something. It is not saying that VSOP is using kepler equations.

http://articles.adsabs.harvard.edu/cgi-bin/nph-iarticle_query?db_key=AST&bibcode=1988A%26A...202..309B&letter=.&classic=YES&defaultprint=YES&whole_paper=YES&page=309&epage=309&send=Send+PDF&filetype=.pdf

p. 311

Quote
4. The solutions we are going to present in this paragraph have been built from VSOP87
....

4.1 VSOP87A solution

This solution is built for the bodies given in Table 1. It is represented with heliocentric rectangular variable X, Y, Z

We must first solve Kepler's equations in order to...
Title: Re: Round Earth Celestial Mechanics Cannot Predict the Solar System
Post by: ICanScienceThat on July 19, 2018, 06:39:38 AM
It occurs to me that the more we point out how you are misinterpreting the RE model, the more you add the misinformation to the wiki.

Could explaining things actually be making the problem worse instead of better?
Title: Re: Round Earth Celestial Mechanics Cannot Predict the Solar System
Post by: Tumeni on July 19, 2018, 07:10:44 AM
Can it do anything more than compute the presumed heliocentric elements?

What else would it need to do?

If it computes the expected position of, say Mercury or Venus some 5 years 47 days in advance, and come that 47th day, you look up, and the predicted position of Mercury turns out to be its actual position, what does that tell you?
Title: Re: Round Earth Celestial Mechanics Cannot Predict the Solar System
Post by: totallackey on July 19, 2018, 10:46:41 AM
You ignoring something does not make it not exist.
There is no ignoring on my part.

You produced nothing of the sort and that is the truth.

Kepler and Newton are much easier to use and for most problems is sufficient
Kepler and Newton are insufficient to explain Mercury.

So when it comes to accurate modeling of what heliocentrists state is "easy," then they are insufficient.

So yeah, you did write something that is in total disagreement.
Title: Re: Round Earth Celestial Mechanics Cannot Predict the Solar System
Post by: Rama Set on July 19, 2018, 11:01:16 AM
You ignoring something does not make it not exist.
There is no ignoring on my part.

You produced nothing of the sort and that is the truth.

Nothing I said disagrees with this.  Did you understand what I said?  It doesn't seem like it.
Kepler and Newton are much easier to use and for most problems is sufficient
Kepler an Newton are insufficient to explain Mercury.

So when it comes to accurate modeling of what heliocentrists state is "easy," then they are insufficient.

So yeah, you did write something that is in total disagreement.
[/quote]

Sorry you don’t know what “for most problems” implies.
Title: Re: Round Earth Celestial Mechanics Cannot Predict the Solar System
Post by: totallackey on July 19, 2018, 11:10:19 AM
Sorry you don’t know what “for most problems” implies.
I most certainly know what it means in this case.

There is no computer model utilizing the math from Newton, Kepler, and Einstein, allowing a CGI representation of the supposed solar system.

The reason for this is it cannot be done, much like an accurate mathematical CGI representation of 9/11 events.
Title: Re: Round Earth Celestial Mechanics Cannot Predict the Solar System
Post by: Rama Set on July 19, 2018, 11:53:25 AM
Sorry you don’t know what “for most problems” implies.
I most certainly know what it means in this case.

Then why are you disputing that Kepler and Newton are sufficient for most problems?  The perihelion of mercury was the only phenomenon it couldn’t model in the solar system.

Quote
There is no computer model utilizing the math from Newton, Kepler, and Einstein, allowing a CGI representation of the supposed solar system.

It’s such a weird thing to say when Gary Green has been discussing just such a model interspersed with our little conversation.

Quote
The reason for this is it cannot be done, much like an accurate mathematical CGI representation of 9/11 events.

9/11 is a whole other topic. Let’s stick with Schpayzze.
Title: Re: Round Earth Celestial Mechanics Cannot Predict the Solar System
Post by: totallackey on July 19, 2018, 01:06:19 PM
Then why are you disputing that Kepler and Newton are sufficient for most problems?  The perihelion of mercury was the only phenomenon it couldn’t model in the solar system.
LOL!

Probably because Kepler and Newton are not sufficient at all.

I mean, they are for myth lovers but no one else.

Quote
There is no computer model utilizing the math from Newton, Kepler, and Einstein, allowing a CGI representation of the supposed solar system.
It’s such a weird thing to say when Gary Green has been discussing just such a model interspersed with our little conversation.
The fact you use the word "say," when it comes to writing explains quite a bit. You have an inability to distinguish between forms of communication, let alone understand.

The fact someone writes about something does not mean it exists and it appears what garygreen has presented is just another instance in a long line of myths.
9/11 is a whole other topic. Let’s stick with Schpayzze.
I can understand why you want the whole topic and relevant comparisons between the two issues placed in the gutter.

You are demonstrably in love with falsehoods.
Title: Re: Round Earth Celestial Mechanics Cannot Predict the Solar System
Post by: garygreen on July 19, 2018, 01:38:32 PM
I am writing an article about VSOP. I have found some further information on how it works and am putting together an analysis.

I looked at your "it uses kepler" reference and found that the article you quoted is saying that the authors are trying to use kepler equations based on the data from VSOP to do something. It is not saying that VSOP is using kepler equations.

http://articles.adsabs.harvard.edu/cgi-bin/nph-iarticle_query?db_key=AST&bibcode=1988A%26A...202..309B&letter=.&classic=YES&defaultprint=YES&whole_paper=YES&page=309&epage=309&send=Send+PDF&filetype=.pdf

p. 311

Quote
4. The solutions we are going to present in this paragraph have been built from VSOP87
....

4.1 VSOP87A solution

This solution is built for the bodies given in Table 1. It is represented with heliocentric rectangular variable X, Y, Z

We must first solve Kepler's equations in order to...

nice ellipses.  the quote is "We must first solve Kepler's equation in order to get the expressions of the variables X, Y, Z." 

as in "[VSOP87a] is represented with heliocentric rectangular variables X, Y, Z...We must first solve Kepler's equation in order to get the expressions of the variables X, Y, Z."  i dunno how the literature could be any more clear on this point.  they are saying that vsop87a outputs heliocentric rectangular coordinates for the positions of the planets.  these coordinates are given by solving kepler's laws.
Title: Re: Round Earth Celestial Mechanics Cannot Predict the Solar System
Post by: Tom Bishop on July 19, 2018, 02:19:12 PM
I am writing an article about VSOP. I have found some further information on how it works and am putting together an analysis.

I looked at your "it uses kepler" reference and found that the article you quoted is saying that the authors are trying to use kepler equations based on the data from VSOP to do something. It is not saying that VSOP is using kepler equations.

http://articles.adsabs.harvard.edu/cgi-bin/nph-iarticle_query?db_key=AST&bibcode=1988A%26A...202..309B&letter=.&classic=YES&defaultprint=YES&whole_paper=YES&page=309&epage=309&send=Send+PDF&filetype=.pdf

p. 311

Quote
4. The solutions we are going to present in this paragraph have been built from VSOP87
....

4.1 VSOP87A solution

This solution is built for the bodies given in Table 1. It is represented with heliocentric rectangular variable X, Y, Z

We must first solve Kepler's equations in order to...

nice ellipses.  the quote is "We must first solve Kepler's equation in order to get the expressions of the variables X, Y, Z." 

as in "[VSOP87a] is represented with heliocentric rectangular variables X, Y, Z...We must first solve Kepler's equation in order to get the expressions of the variables X, Y, Z."  i dunno how the literature could be any more clear on this point.  they are saying that vsop87a outputs heliocentric rectangular coordinates for the positions of the planets.  these coordinates are given by solving kepler's laws.

"The solutions we are going to present ... have been built from VSOP87"

Its not a description on how VSOP was programmed. The article is about taking data from VSOP and trying to manipulate it.

(https://i.imgur.com/7dl0vAI.png)
Title: Re: Round Earth Celestial Mechanics Cannot Predict the Solar System
Post by: garygreen on July 19, 2018, 07:16:26 PM
"The solutions we are going to present ... have been built from VSOP87"

Its not a description on how VSOP was programmed. The article is about taking data from VSOP and trying to manipulate it.

sort of, but not really.  read the abstract and section 3.  "solutions" means "the positions of the planets at a given time." 

vsop87 is a model of the solar system using only the elliptic parameters of each planetary orbit (semi-major axis, eccentricity, inclination, etc.).  but just knowing elliptic parameters isn't useful.  we want to know physical coordinates of the planets at a given time.  that's what vsop87a, vsop87b, etc, are for.  they're just outputting the positions of the planets in different coordinate/reference systems.

look at this table:
(https://i.imgur.com/9RvWy2g.png?1)

vsop87 describes each orbit in terms of the shape of the orbit (elliptic parameters).  vsop87a-e describe the positions of the planets in terms of different coordinate and reference systems (eg rectangular vs spherical).

all of these inputs and outputs are in terms of real, physical quantities: the shape of the orbit, the distance from the sun or barycenter, the mass of the object, etc.  i cannot find anything in these documents that describes the kind of pattern-counting that you say astronomy relies upon.
Title: Re: Round Earth Celestial Mechanics Cannot Predict the Solar System
Post by: Tom Bishop on July 20, 2018, 01:36:17 AM
I will come back to the VSOP program in a bit. I have added three new sections to the article:


https://wiki.tfes.org/Celestial_Mechanics_Cannot_Predict_The_Solar_System#The_Zeroth_Step
Title: Re: Round Earth Celestial Mechanics Cannot Predict the Solar System
Post by: Rama Set on July 20, 2018, 01:46:50 AM
Your writing is either incompetently misinterpreting the material you cite or painfully dishonest, Tom. You write that they found over 16,000,000 solutions to the 3-body problem and cherry-pick that a few thousand are exotic. And why is Newton’s personal belief in divinity at all relevant other than to serve your editorial mission? Is your argument so weak that you can’t make it cohesive without fallacies and dishonesty?
Title: Re: Round Earth Celestial Mechanics Cannot Predict the Solar System
Post by: Tom Bishop on July 20, 2018, 02:32:19 AM
I will come back to the VSOP program in a bit. I have added three new sections to the article:

    - The Zeroth Step
    - Very Sensitive Solutions
    - Official Explanation: Divine Intervention

https://wiki.tfes.org/Celestial_Mechanics_Cannot_Predict_The_Solar_System#The_Zeroth_Step
Your writing is either incompetently misinterpreting the material you cite or painfully dishonest, Tom. You write that they found over 16,000,000 solutions to the 3-body problem and cherry-pick that a few thousand are exotic. And why is Newton’s personal belief in divinity at all relevant other than to serve your editorial mission? Is your argument so weak that you can’t make it cohesive without fallacies and dishonesty?

The article says that they tested 16,000,000 orbits to find the stable ones. They did not find 15,998,777 stable heliocentric orbits. What happened to the other ones that were not the 1223 they found? They flew apart! There is no dishonesty about it. The article illustrates that Celestial Mechanics is on step zero. They can't model it. They can't even create a simple Sun-Earth-Moon system.

The model of astronomy is based on divine intervention, as stated by Newton himself. This is the only explanation that I have seen.
Title: Re: Round Earth Celestial Mechanics Cannot Predict the Solar System
Post by: Rama Set on July 20, 2018, 02:36:39 AM
I will come back to the VSOP program in a bit. I have added three new sections to the article:

    - The Zeroth Step
    - Very Sensitive Solutions
    - Official Explanation: Divine Intervention

https://wiki.tfes.org/Celestial_Mechanics_Cannot_Predict_The_Solar_System#The_Zeroth_Step
Your writing is either incompetently misinterpreting the material you cite or painfully dishonest, Tom. You write that they found over 16,000,000 solutions to the 3-body problem and cherry-pick that a few thousand are exotic. And why is Newton’s personal belief in divinity at all relevant other than to serve your editorial mission? Is your argument so weak that you can’t make it cohesive without fallacies and dishonesty?

The article says that they tested 16,000,000 orbits to find the stable ones. They did not find 15,998,777 stable heliocentric orbits. What happened to the other ones that were not the 1223 they found? They flew apart! There is no dishonesty about it. The article illustrates that Celestial Mechanics is on step zero. They can't model it. They can't even create a simple Sun-Earth-Moon system.

My mistake on the numbers, but your objection is still feeble. They can model it, they do model it. You constantly mistake, either willfully or out of plain inability to understand, why there is a limitation on calculating a three-body system.

Quote
The model of astronomy is based on divine intervention, as stated by Newton himself. This is the only explanation I have seen.

This is trash. It’s an argument from authority at the beginning and just a plain lie at the end. The model of astronomy, is based on observation, even if you roll it back to what the Babylonians were doing, and you are perfectly aware of that. Why do you feel the need to stoop down to the basest, most dishonest level?
Title: Re: Round Earth Celestial Mechanics Cannot Predict the Solar System
Post by: Tom Bishop on July 20, 2018, 06:15:24 AM
The heliocentric solar system cannot be modeled. That is the entire basis of the n-body problem. The greatest minds in human history have tried and tried and failed and failed.

The reason it cannot be modeled is because of Chaos Theory. I have a Three Body Problem Chaos Theory demo in the article. (https://wiki.tfes.org/Celestial_Mechanics_Cannot_Predict_The_Solar_System#Chaos_Theory:_A_Demo)

Newton's answer of divine intervention for this is a very important point. Newton is credited for bringing the laws of physics to the solar system.

http://earthguide.ucsd.edu/virtualmuseum/ita/05_1.shtml

Quote
Isaac Newton (1642-1727) who brought the laws of physics to the solar system. Isaac Newton explained why the planets move the way they do, by applying his laws of motion, and the force of gravitation between any two bodies, letting the force decrease with the square of the distance between the two bodies.

If this solar system of his does not work, and is, by his own words, based on divine intervention, then that is a fact that everyone should know.

Newton's physics of the solar system is "GOD DID IT!"
Title: Re: Round Earth Celestial Mechanics Cannot Predict the Solar System
Post by: Rama Set on July 20, 2018, 07:06:51 AM
Except that just a couple of paragraphs before you cite an article that gives thousands of stable solutions and as Gary has written ad nauseam that not having an analytical solution is not the same as not having any solution, nor is it the same as the solar system model, of which you have presented one in this very thread, doesn’t work. Just because Newton said something 300 hundred years ago, doesn’t mean that is the current state of affairs as well. Furthermore, Newton didn’t even say, “God did it!” He merely expressed his lack of knowledge of how it worked. He may have introduced some major physical laws, but that doesn’t mean he is meant to have all the answers.
Title: Re: Round Earth Celestial Mechanics Cannot Predict the Solar System
Post by: Tom Bishop on July 21, 2018, 04:06:57 AM
The Heliocentric Theory is a very sensitive theory that simply does not work. See these quotes Sandokan provided:

As Poincare experimented, he was relieved to discover that in most of
the situations, the possible orbits varied only slightly from the initial
2-body orbit, and were still stable, but what occurred during further
experimentation was a shock. Poincare discovered that even in some of the
smallest approximations some orbits behaved in an erratic unstable manner. His
calculations showed that even a minute gravitational pull from a third body
might cause a planet to wobble and fly out of orbit all together.

Here is Poincare describing his findings:

While Poincare did not succeed in giving a complete solution, his work was so impressive that he was awarded the prize anyway. The distinguished Weierstrass, who was one of the judges, said, 'this work cannot indeed be considered as furnishing the complete solution of the question proposed, but that it is nevertheless of such importance that its publication will inaugurate a new era in the history of celestial mechanics.' A lively account of this event is given in Newton's Clock: Chaos in the Solar System. To show how visionary Poincare was, it is perhaps best if he described the Hallmark of Chaos - sensitive dependence on initial conditions - in his own words:

'If we knew exactly the laws of nature and the situation of the universe at the initial moment, we could predict exactly the situation of that same universe at a succeeding moment. but even if it were the case that the natural laws had no longer any secret for us, we could still only know the initial situation approximately. If that enabled us to predict the succeeding situation with the same approximation, that is all we require, and we should say that the phenomenon had been predicted, that it is governed by laws. But it is not always so; it may happen that small differences in the initial conditions produce very great ones in the final phenomena. A small error in the former will produce an enormous error in the latter. Prediction becomes impossible, and we have the fortuitous phenomenon.' - in a 1903 essay 'Science and Method'

You can test it out for yourself in the simulation (https://wiki.tfes.org/Celestial_Mechanics_Cannot_Predict_The_Solar_System#Chaos_Theory:_A_Demo) I provided in my article. The slightest change causes the system to fall apart. You see it first hand for yourself.

The 'thousands of stable solutions' are scenarios where the bodies have the same mass or where one of the bodies is mass-less. All of these scenarios are incredibly sensitive, and none represent anything that looks like a heliocentric system. They are crazy loopy orbits, based on situations that would not happen in nature, and which fall apart with the slightest breeze.

Newton couldn't figure out how to make his system work. The greatest mathematicians couldn't figure out how to make it work. Supercomputers could not provide the solution. The problems are insoluble. The heliocentric systems of Copernicus, Newton, and Kepler do not work. The current state of affairs is that it doesn't work at all. It is a fantasy.
Title: Re: Round Earth Celestial Mechanics Cannot Predict the Solar System
Post by: sandokhan on July 21, 2018, 05:23:54 AM
There are two intractable problems with the 1,223 periodic solutions found by the team of Chinese mathematicians (no, not the fact that two of the three bodies have the same mass, or that they remain in the same plane).

The orbits will immediately exhibit homoclinic tangles at the slightest perturbation; that is, the orbital stability has not been tested/analyzed. The heliocentric planetary system features perturbations which are not slight; in fact, the perturbations in the solar system are far too large to apply KAM theory (which is valid for "sufficiently" small perturbations).

The second unsolvable problem concerns the fact that the initial conditions of the RE heliocentric planetary system are completely unknown: to show the importance and the dependence on the sensitivity of the initial conditions of the set of differential equations, an error as small as 15 meters in measuring the position of the Earth today would make it impossible to predict where the Earth would be in its orbit in just over 100 million years' time. A difference in the initial position of 1 cm grows to ∼1 AU (= 1.496 x 10^11 m) after 90–150 million years.

And the RE system of equations feature not three, but nine nonlinear ordinary differential equations.

The moral of the story is this: the RE heliocentric planetary orbits of motion cannot be described at all by the theory of nonlinear ordinary differential equations.
Title: Re: Round Earth Celestial Mechanics Cannot Predict the Solar System
Post by: SphericalEarther on July 21, 2018, 09:21:43 AM
3 body problem is completely unrelated to the heliocentric model.
And sure, we can't calculate precise orbits without precise measurements.
We can easily make the simulation though. Nothing wrong with the heliocentric model.
Title: Re: Round Earth Celestial Mechanics Cannot Predict the Solar System
Post by: BillO on July 21, 2018, 02:39:46 PM
Newton couldn't figure out how to make his system work. The greatest mathematicians couldn't figure out how to make it work. Supercomputers could not provide the solution. The problems are insoluble. The heliocentric systems of Copernicus, Newton, and Kepler do not work. The current state of affairs is that it doesn't work at all. It is a fantasy.
Is your issue with the model, or with the idea that things can orbit at all?
Title: Re: Round Earth Celestial Mechanics Cannot Predict the Solar System
Post by: Rama Set on July 21, 2018, 03:44:46 PM
No, Tom thinks that because there are no analytical solutions to the three-body problem that the heliocentric solar system is not a viable model. Despite having it explained to him numerous times, he still does not understand, or refuses to admit, that this is not a failure of the model but rather an inherent limitation of the math used to solve such problems.
Title: Re: Round Earth Celestial Mechanics Cannot Predict the Solar System
Post by: Tom Bishop on July 21, 2018, 05:21:57 PM
This has nothing to do with the analytical solution in specific.

The analytical solution is the ability to predict where bodies will be based on starting parameters, which is admitted to be impossible.

The numerical solution is essentially the ability to model or simulate where bodies will be, by accounting for the gravitational influences of the bodies at small incremental time steps.

In such simulations the orbits of three bodies with unequal masses will fly apart, and  the available solutions to the numerical three body problem assume unrealistic scenarios-- such as the Restricted Three Body Problem, which assumes that one of the bodies is mass-less.

From a Celestial Mechanics university course we read the following at http://farside.ph.utexas.edu/teaching/celestial/Celestial/node79.html

Quote
Three Body Problem

What about a system containing three gravitationally interacting point masses? Despite hundreds of years of research, no useful general solution of this famous problem--which is usually called the three-body problem--has ever been found. It is, however, possible to make some progress by severely restricting the problem's scope.

The following pages show what is meant by "restricting the problem's scope." If we assume that bodies are massless, or that bodies have identical masses, then sure, a "stable orbit" can be created (a 'stable orbit' that look nothing like a heliocentric orbit, mind you). These are unrealistic scenarios in astronomy, but it is the best that can be done to study the matter of basic orbits.

As Poincare discovered, and as was communicated in Sandokan's quote, any slight inequality causes the entire thing to fall apart.

The field of Celestial Mechanics is not even in its infancy. The field of Celestial Maechanics has shown that the basic ideas are invalid and the heliocentric solar system does not even work at all.

Despite having it explained to him numerous times, he still does not understand, or refuses to admit, that this is not a failure of the model but rather an inherent limitation of the math used to solve such problems.

The "limitation of the math" means that they cannot predict where bodies will be in the solar system.

The "limitations of the math" also means that they cannot even simulate a stable system  without assuming unrealistic scenarios.

"Limitations of the math" is an admission of defeat! The system cannot be modeled or predicted.
Title: Re: Round Earth Celestial Mechanics Cannot Predict the Solar System
Post by: Rama Set on July 21, 2018, 05:47:49 PM
As has been pointed out to you ad nauseam, n-body problems are routinely solved numerically. Analytically there is no way, currently to get a 100% accurate answer. The solar system has been modeled and even with the limitations of the three-body problem, scientists and mathematicians can get to greater than 99% accuracy. The remaining amount is filled in with constant observation. The model works.
Title: Re: Round Earth Celestial Mechanics Cannot Predict the Solar System
Post by: edby on July 21, 2018, 06:09:55 PM
As has been pointed out to you ad nauseam, n-body problems are routinely solved numerically. Analytically there is no way, currently to get a 100% accurate answer. The solar system has been modeled and even with the limitations of the three-body problem, scientists and mathematicians can get to greater than 99% accuracy. The remaining amount is filled in with constant observation. The model works.
This explains it well.

https://www.youtube.com/watch?v=97Dg2sWXUiM
Title: Re: Round Earth Celestial Mechanics Cannot Predict the Solar System
Post by: sandokhan on July 21, 2018, 06:36:49 PM
Orbital equations of motion = theory of nonlinear ordinary differential equations

The model works.

Let's see what the experts in the field have to say.

“Kolmogorov (1954) made the momentous discovery that catastrophes in planetary systems
are all too likely when he found that the minor terms in Newcomb’s series used to
solve the n-body problem for planetary motion in Newtonian mechanics were, in
fact, easily subject to random, unpredictable perturbations which could quickly lead
to turbulence; cf. Arnol’d (1997). In the meantime, the buzzword ‘chaos’ is applied
to such turbulence – also called ‘catastrophe,’ to the dismay of Shapley and many
other astronomers sharing Newton’s dream of an eternally stable solar system . . .
Subsequently, in the later fifties, Siegel & Moser (1971) and Moser (1973) further
extended mathematical knowledge of divergent [orbits of planets], Newcomb
series.

That ‘deterministic’ systems, like those obeying Newtonian mechanics, should
be subject to unpredictable catastrophes came as a real shock.

Next came pioneering computer simulations of planetary – system evolution
by Hills (1969). Wisdom (1981), finding even more advanced numerical methods
and using faster computers, achieved the breakthrough of repeatedly observing
chaos (or catastrophes) in computer-simulated planetary orbits, splendidly
confirming Kolmogorov and putting the dream of Newton  in an absolutely stable
and orderly cosmos to rest in a way most astronomers could not have previously
imagined. All experts now agree that even apparently stable [planetary] orbits will
occasionally experience unpredictable chaotic disturbances.”

E. Köhler, Induction and Deduction in Science

 “these [stability] models neglect solar mass loss and the effect of passing
stars . . . [whose perturbations] should shorten the lifetimes of the systems [change to instability]
by several orders of magnitude.”

J. J. Lissaur, Q. N. C. Lin, “Diversity of Planetary Systems and Unsolved Problems,” From Extrasolar
Planets to Cosmology: V. L. T. Opening Symposium

“Instead of using full equations of motion, Laskar focused on a special
formulation that spotlights gradual but cumulative changes in an orbit’s shape
[eccentricity] and orientation [inclination]. He worked with equations that smooth
out the recurring wiggles and wobbles in planetary orbits leaving only long term
trends . . .

“By applying a similar strategy to celestial curves [eccentricities], Laskar could
isolate these [non-gravitational] parts of a planet’s motion that correspond to lasting
changes in key characteristics of its orbit.”

Ivars Peterson, Newton’s Clock

“This differential system is a close approximation to the real solar system, and in
particular, the inner solar system . . . but the exact meaning of ‘close’ is still difficult to evaluate.”

J. Laskar

When one employs non-gravitational theory instead of “full equations of motion” to prove
stability, one has removed the solution of the problem from reality. But even believing in these
non-gravitational equations, Laskar cannot tell what a “close approximation” is. It is a theoretical
construct that comes out of non-gravitational math.

C. Ginenthal

Repeatedly the eccentricities and inclination of the positions of the orbits of the planets
in the solar system are relied on to determine whether or not there was stability in the past or
stability in the future. Because this tool, that is not based strictly on gravitational theory using
only masses and their separations, is the basis for all reckoning of stability, it is essentially a great
fudge factor that has been swallowed hook, line and sinker by astronomers, never noticing the
distinction between their heuristic mathematics and the underlying forces that should have been
employed in the first place.

C. Ginenthal

“Efforts to settle the question of the solar system’s stability face a serious,
perhaps insurmountable obstacle. As Scott Tremain has remarked: ‘In some sense,
you end up having to deal with probabilities. You can never rule anything out
completely. Even if a [planetary] system is well behaved, there’s always a small
chance of its wandering by some narrow path to just about any configuration.’ In
other words, with a mathematical model that automatically incorporates chaotic
behavior, there’s no way to prove, with absolute certainty, that something can’t ever
happen.”

Ivars Peterson, Newton’s Clock

“From a physical point of view, this model [of the solar system] is obviously
hard to accept, but cannot escape these conclusions, if one idealizes the problem
mathematically . . . In fact, the [mathematical] idealization goes further: We are not
talking about the motions of planets under realistic forces, but of the n-body
problem taking into account only Newton’s force laws and referring to mass points
with some smallness restriction on masses.”

Jurgen Moser (one of the giants in the field, proved the exceedingly difficult KAM theory)

J. Moser, “Stability in Celestial Mechanics,” The Stability of the Solar System and of Small Stellar
Systems Symposium

“The word ‘chaotic’ summarizes many fundamental concepts characterizing
a dynamical system such as complex predictability and stability. But above
all, it acts as a warming of the difficulties which are likely to arise when trying to
obtain a reliable picture of its past and future evolution. As an example, a
commonly accepted definition states that a system is ‘unstable’ if the trajectories of
two points that initially are arbitrarily close . . . diverge quickly in time. This has
strong implications, as small uncertainties in initial conditions . . . might [also] be
consistent with completely different future trajectories: The conclusion is that we
can exactly reproduce the motion of a chaotic system only if WE KNOW, WITH
ABSOLUTE PRECISION, THE INITIAL CONDITIONS – A STATEMENT
THAT, IN PRACTICE, CAN NEVER BE TRUE."


Alessandra Celletti, Ettore Perozzi, Celestial Mechanics: The Waltz of the Planets

Sussman and Wisdom's 1992 integration of the entire solar system displayed a disturbing dependence on the timestep of the integration (measurement of the Lyapunov time).

Thus, different researchers who draw their initial conditions from the same ephemeris at different times can find vastly different Lyapunov timescales.

Wayne Hayes, UC Irvine
Title: Re: Round Earth Celestial Mechanics Cannot Predict the Solar System
Post by: Tom Bishop on July 21, 2018, 06:43:22 PM
No one solved the n-body problem

Where is the Nobel Prize?

Just as the author of that video above says, the attempt to simulate the solar system, or predict it, was an utter failure in the history of science, so they gave up and instead are looking at the special perturbations of the bodies in the heavens (which the heliocentric fantasy is now calling "due to gravity").

This is the method of prediction that the Ancient Babylonians used: Patterns in the Sky.

Statistics... interpolation.. curve fitting.. "of chaos," etc.

They are inferring things about the patterns of the planets in the sky to determine whether they are stable or unstable, and to predict where they will go. They entirely gave up on the n-body problem. They admitted defeat!
Title: Re: Round Earth Celestial Mechanics Cannot Predict the Solar System
Post by: Tom Bishop on July 21, 2018, 06:55:35 PM
Words to the effect of they are now merely looking at perturbations are given in the video.

https://en.wikipedia.org/wiki/Perturbation_(astronomy)

Special Perturbations

In methods of special perturbations, numerical datasets, representing values for the positions, velocities and accelerative forces on the bodies of interest, are made the basis of numerical integration of the differential equations of motion.[6] In effect, the positions and velocities are perturbed directly, and no attempt is made to calculate the curves of the orbits or the orbital elements.[2] Special perturbations can be applied to any problem in celestial mechanics, as it is not limited to cases where the perturbing forces are small.[4] Once applied only to comets and minor planets, special perturbation methods are now the basis of the most accurate machine-generated planetary ephemerides of the great astronomical almanacs.[2][7] Special perturbations are also used for modeling an orbit with computers.
Title: Re: Round Earth Celestial Mechanics Cannot Predict the Solar System
Post by: Rama Set on July 21, 2018, 07:02:14 PM
That’s actually not what was said in the video Edby posted at all. It was stated quite clearly that our model of the solar system is accurate and runs how we have modeled it until you get 100,000,000s of years in to the future when something like the butterfly effect starts to be relevant. As such the model can only be expressed in statistical terms and that the solar system is 99% likely to be stable.
Title: Re: Round Earth Celestial Mechanics Cannot Predict the Solar System
Post by: Tom Bishop on July 21, 2018, 07:14:44 PM
That’s actually not what was said in the video Edby posted at all. It was stated quite clearly that our model of the solar system is accurate and runs how we have modeled it until you get 100,000,000s of years in to the future when something like the butterfly effect starts to be relevant. As such the model can only be expressed in statistical terms and that the solar system is 99% likely to be stable.

I watched the video. Where does he say that the n-body problem was solved?

After a history of the utter failure of science he does use the word perturbations at some point and starts saying they switched over to "following the paths" (let me help you: IN THE SKY) to see that the solar system is stable, and are using various methods to determine if there is chaotic activity that will tear the system apart.

Total fantasy. If you disbelieve, where is the flipping Nobel Prize for solving the greatest unsolved issue in science?
Title: Re: Round Earth Celestial Mechanics Cannot Predict the Solar System
Post by: Rama Set on July 21, 2018, 07:17:50 PM
As has been stated to you, and as he said in the video, they didn’t solve it analytically but numerically. Using the initial conditions and laws of motion. It’s been said to you enough that there currently is no analytical solution but numerical solutions are trivial and commonplace. Do you understand?

FYI Nobel prizes are not good metrics. Einstein didn’t get one for his most popular contribution: Relativity.
Title: Re: Round Earth Celestial Mechanics Cannot Predict the Solar System
Post by: Tom Bishop on July 21, 2018, 07:28:19 PM
Henri Poincare won a big prize for showing that the the basic orbits made by bodies are unstable, fragile, and are easily turned chaotic at a whiff.

This is not merely about trying to find an analytic or numeric solution. The basic orbits of the bodies is fragile, unstable, and unable to be modeled.

The author of the video admits as much, and transitions into them now following the paths (in the sky) and looking at chaos. The Solar System can't be modeled with orbital elements.

The above quote I gave shows that the astronomical almanacs are using special perturbations, which do not use orbital elements, and this is what the person in the video is talking about. None of it is based on the heliocentric model of orbits and gravity.
Title: Re: Round Earth Celestial Mechanics Cannot Predict the Solar System
Post by: sandokhan on July 21, 2018, 07:48:54 PM
This explains it well.

It does not, on the contrary.

Scott Tremaine's arguments rest totally on Jacques Laskar's numerical simulations.

(https://image.ibb.co/e2EYGJ/gil00_zpslwerxjhx.jpg)
(https://image.ibb.co/d4EawJ/gil01_zpsbzfkgbpu.jpg)
(https://image.ibb.co/mtf4qd/gil02_zpsodpyookj.jpg)
(https://image.ibb.co/gDuxAd/gil03_zpsibabxpcg.jpg)
(https://image.ibb.co/b8Giiy/gil04_zps4wcpajjf.jpg)
(https://image.ibb.co/nPa9Oy/gil05_zpsmmpvvhcu.jpg)
(https://image.ibb.co/dx9VVd/gil06_zpsljjqeiia.jpg)
(https://image.ibb.co/bys63y/gil07_zpsalpm8lqo.jpg)

Title: Re: Round Earth Celestial Mechanics Cannot Predict the Solar System
Post by: Rama Set on July 21, 2018, 07:53:54 PM
Their model is accurate out to 100,000,000 years and 99% accurate after that. This doesn’t seem to matter to you. All that matters to you is that it is 1% likely to be unstable after 100,000,000 years. It was found to be accurate using the laws of motion and observation of initial conditions. There isn’t much more to be said. Now, perhaps you should concentrate on developing a model that can show exactly the sun travels over a FE?
Title: Re: Round Earth Celestial Mechanics Cannot Predict the Solar System
Post by: Tom Bishop on July 21, 2018, 08:06:53 PM
Rama, look at the quotes Sandokan gave in response to edby about the millions of years stability stuff.

(https://i.imgur.com/tZdVR23.png)

Where did he get his data on "eccentricity" and "orbital shape," one may ask? From the place all astronomers get it from under in their fantasy conjecture: The sky!

Smooth out over "long term trends." This clearly indicates a statistical analysis.
Title: Re: Round Earth Celestial Mechanics Cannot Predict the Solar System
Post by: Rama Set on July 21, 2018, 08:09:34 PM
Rama, look at the quote Sandokan gave in response to edby.

(https://i.imgur.com/tZdVR23.png)

Where did he get his data on "eccentricity" and "orbital shape" one may as? From the place all astronomers get it from under in their fantasy conjecture: The sky!

Smooth out over "long term trends." This clearly indicates a statistical analysis.

I have no idea what Sandokhan is even sourcing.  Do you?
Title: Re: Round Earth Celestial Mechanics Cannot Predict the Solar System
Post by: Tom Bishop on July 21, 2018, 08:13:26 PM
He's sourcing the millions of years stability analysis:

(https://i.imgur.com/wqCEEKy.png)
Title: Re: Round Earth Celestial Mechanics Cannot Predict the Solar System
Post by: Rama Set on July 21, 2018, 08:14:33 PM
He's sourcing the millions of years stability analysis:

(https://i.imgur.com/wqCEEKy.png)

No, whose critique is it? Nature?  Cosmopolitan?
Title: Re: Round Earth Celestial Mechanics Cannot Predict the Solar System
Post by: sandokhan on July 21, 2018, 08:33:04 PM
KAM (Kolmogorov-Arnold-Moser) theory tells us that essentially all Hamiltonian systems which are not integrable are chaotic.

Since the solar system is not integrable, and experiences unpredictable small perturbations, it cannot lie permanently on a KAM torus, and is thus chaotic.

An initial condition not lying precisely on a KAM torus will eventually admit chaos, but with a time scale that depends critically on the initial condition.

The validity of symplectically-integrated numerical solutions also depends critically upon the integration time step h, with the longevity of the solution’s validity scaling as e^(a/h) (for some constant a).

Thus, different researchers who draw their initial coniditions from the same ephemeris at different times can find vastly different Lyapunov timescales.


As if this wasn't enough, astronomers have to deal with the phenomenal discovery made by Dr. Robert W. Bass.

Dr. Robert W. Bass

Ph.D. (Mathematics) Johns Hopkins University, 1955 [Wintner, Hartman]
A. Wintner, world's leading authority on celestial mechanics
Post-Doctoral Fellow Princeton University, 1955-56 [under S. Lefschetz]
Rhodes Scholar
Professor, Physics & Astronomy, Brigham Young University

"In a resonant, orbitally unstable or "wild" motion, the eccentricities of one or more of the terrestrial planets can increase in a century or two until a near collision occurs. Subsequently the Principle of Least Interaction Action predicts that the planets will rapidly "relax" into a configuration very near to a (presumably orbitally stable) resonant, Bode's-Law type of configuration. Near such a configuration, small, non-gravitational effects such as tidal friction can in a few centuries accumulate effectively to a discontinuous "jump" from the actual phase-space path to a nearby, truly orbitally stable, path. Subsequently, observations and theory would agree that the solar system is in a quasi-periodic motion stable in the sense of Laplace and orbitally stable. Also, numerical integrations backward in time would show that no near collision had ever occurred. Yet in actual fact this deduction would be false."

"I arrived independently at the preceding scenario before learning that dynamical astronomer, E. W. Brown, president of the American Astronomical Society, had already outlined the same possibility in 1931."

Dr. Robert Bass, Stability of the Solar System:

https://web.archive.org/web/20120916174745/http://www.innoventek.com:80/Bass1974PenseeAllegedProofsOfStabilityOfSolarSystemR.pdf

Dr. E.W. Brown

Fellowship, Royal Society
President of the American Mathematical Society
Professor of Mathematics, Yale University
President of the American Astronomical Society

What this means is that the interval of assured reliability for Newton's equations of gravitational motion is at most three hundred years.

Dr. W.M. Smart

Regius Professor of Astronomy at Glasgow University
President of the Royal Astronomical Society from 1949 to 1951

(https://image.ibb.co/dtvyuS/bass4.jpg)
(https://image.ibb.co/eRjPZS/bass5.jpg)
(https://image.ibb.co/ePj2M7/bass6.jpg)
(https://image.ibb.co/bPBMES/bass7.jpg)

Within this 300 year time interval, we again have the huge problem of the sensitive dependence on initial conditions.

To show the importance and the dependence on the sensitivity of the initial conditions of the set of differential equations, an error as small as 15 meters in measuring the position of the Earth today would make it impossible to predict where the Earth would be in its orbit in just over 100 million years' time.

“The word ‘chaotic’ summarizes many fundamental concepts characterizing
a dynamical system such as complex predictability and stability. But above
all, it acts as a warming of the difficulties which are likely to arise when trying to
obtain a reliable picture of its past and future evolution. As an example, a
commonly accepted definition states that a system is ‘unstable’ if the trajectories of
two points that initially are arbitrarily close . . . diverge quickly in time. This has
strong implications, as small uncertainties in initial conditions . . . might [also] be
consistent with completely different future trajectories: The conclusion is that we
can exactly reproduce the motion of a chaotic system only if WE KNOW, WITH
ABSOLUTE PRECISION, THE INITIAL CONDITIONS – A STATEMENT
THAT, IN PRACTICE, CAN NEVER BE TRUE."

Alessandra Celletti, Ettore Perozzi, Celestial Mechanics: The Waltz of the Planets

If any proofs can be provided that the solar system underwent cataclysmic planetary collisions in recent historical times, this fact would render any kind of heliocentric orbital calculations as completely useless.

https://www.theflatearthsociety.org/forum/index.php?topic=30499.msg1936055#msg1936055 (part I)

https://www.theflatearthsociety.org/forum/index.php?topic=30499.msg1938384#msg1938384 (part II)

https://www.theflatearthsociety.org/forum/index.php?topic=30499.msg1938393#msg1938393 (part III)

https://www.theflatearthsociety.org/forum/index.php?topic=30499.msg1938396#msg1938396 (part IV)

Title: Re: Round Earth Celestial Mechanics Cannot Predict the Solar System
Post by: edby on July 21, 2018, 09:01:00 PM
This explains it well.

It does not, on the contrary.

Scott Tremaine's arguments rest totally on Jacques Laskar's numerical simulations.

The passage you quote is from Newton, Einstein, and Velikovsky, is that correct? Ginenthal is the founder and principal contributor to an online journal The Velikovskian.

[edit] https://en.wikipedia.org/wiki/Immanuel_Velikovsky
Title: Re: Round Earth Celestial Mechanics Cannot Predict the Solar System
Post by: edby on July 21, 2018, 09:07:10 PM
Quote
In general, Velikovsky's theories have been ignored or vigorously rejected by the academic community. Nonetheless, his books often sold well and gained an enthusiastic support in lay circles, often fuelled by claims of unfair treatment for Velikovsky by orthodox academia. The controversy surrounding his work and its reception is often referred to as "the Velikovsky affair". Velikovsky's work is frequently cited as a canonical example of pseudoscience and has been used as an example of the demarcation problem.
https://en.wikipedia.org/wiki/Immanuel_Velikovsky

Quote
The demarcation problem in the philosophy of science is about how to distinguish between science and non-science, including between science, pseudoscience, and other products of human activity, like art and literature, and beliefs. The debate continues after over two millennia of dialogue among philosophers of science and scientists in various fields, and despite broad agreement on the basics of scientific method.
https://en.wikipedia.org/wiki/Demarcation_problem
The Demarcation Problem was what brought me to this site, as it happens.

Quote
Worlds in Collision is a book written by Immanuel Velikovsky and first published April 3, 1950. The book postulated that around the 15th century BC, Venus was ejected from Jupiter as a comet or comet-like object, and passed near Earth (an actual collision is not mentioned). The object changed Earth's orbit and axis, causing innumerable catastrophes that were mentioned in early mythologies and religions around the world. Many of the book's claims are completely rejected by the established scientific community as they are not supported by any available evidence.
https://en.wikipedia.org/wiki/Worlds_in_Collision

Quote
The plausibility of the theory was summarily rejected by the physics community, as the cosmic chain of events proposed by Velikovsky contradicts basic laws of physics.

Quote
As Friedlander recounts, "When I gave each example, [Velikovsky's] response was 'Where did I write that?'; when I showed a photo copy of the quoted pages, he simply switched to a different topic."
Title: Re: Round Earth Celestial Mechanics Cannot Predict the Solar System
Post by: sandokhan on July 21, 2018, 09:15:29 PM
The Demarcation Problem was what brought me to this site, as it happens.

Right.

Then, you'll happy to find out that Kepler fudged/faked/falsified the entire set of data in the New Astronomy.

This data in turn lead to the law of universal gravitation published by Newton.

This law was then used to derive the nonlinear differential equations approach to orbital mechanics.

Kepler published his first law of planetary motion based on the data gathered by Tycho
Brahe in 1609. The law states that planets orbit the sun in ellipses with the sun at one focus.


“Almost 400 years later, William H. Donohue undertook the task of translating
Kepler’s 1609 Astronomia Nova into the English New Astronomy (Donohue 1992)
when in the course of his work he redid many of Kepler’s calculations, he was
startled to find some fundamental inconsistencies with Kepler’s reporting of these
same calculations (Donohue 1988). Writing of Donohue’s pathbreaking work in
The New York Times, William Broad (1990) summarized Donahue’s findings
saying that although Kepler claimed to have confirmed the elliptical orbit by
independent observations and calculations of the position of Mars, in fact Kepler
derived the data from the theory instead of the other way around . . .

“But a close study of Kepler’s New Astronomy . . . shows that the plotted points
[he used] do not fall exactly on the ellipse (of course, measurements rarely fall
exactly on a theoretical curve because they usually have random error sources
incorporated into them.) Curtis Wilson (1968), however, carries error argument
further. The lack of precision inherent in the method . . . would have forced Kepler
to use the plotted points only as a guide to his theorizing . . .
“After detailed computational arguments Donahue concluded the results
reported by Kepler . . . were not at all based on Brahe’s observational data; rather
they were fabricated on the basis of Kepler’s determination that Mars’s orbit was
elliptical. Donahue reasons that Kepler must have gone back to revise his earlier
calculations that were made prior to his understanding that the orbit of Mars was
actually elliptical. Thus, anyone who cared to check Kepler’s tables would find
numbers that are consistent with the elliptical orbit [he] postulated for Mars and
would be inclined to believe that the numbers represented observational data. In
fact, they were computed from the hypothesis of an elliptical orbit and then
modified for measurement error; such data, if they were truly observations, would
be prime facie evidence of the theories’ correctness.

“So Donahue . . . realized that the theory was not obviously derivable from the
observations, . . . ‘Not only would the numbers be confused, but Kepler saw clearly
that no satisfactory theory could come from such a procedure. . . [Instead], he chose
a short cut.’ He became so convinced of what drove these physical processes that he subjectively projected his personal nonobservational-based belief onto the reporting scene to convince others in the scientific community of the validity of his theories.”

Thus, the very first law of planetary motion was built not on observation but on theory
and the mathematics was then employed to prove the theory not test it.


http://adsabs.harvard.edu/full/1988JHA....19..217D

Kepler's fabricated figures, by W.H. Donohue

The scholar, William H. Donahue, said the evidence of Kepler's scientific fakery is contained in an elaborate chart he presented to support his theory.

The discovery was made by Dr. Donahue, a science historian, while translating Kepler's master work, ''Astronomia Nova,'' or ''The New Astronomy,'' into English. Dr. Donahue, who lives in Sante Fe, N.M., described his discovery in a recent issue of The Journal of the History of Astronomy.

The fabricated data appear in calculated positions for the planet Mars, which Kepler used as a case study for all planetary motion. Kepler claimed the calculations gave his elliptical theory an independent check. But in fact they did nothing of the kind.

''He fudged things,'' Dr. Donahue said, adding that Kepler was never challenged by a contemporary. A pivotal presentation of data to support the elliptical theory was ''a fraud, a complete fabrication,'' Dr. Donahue wrote in his paper. ''It has nothing in common with the computations from which it was supposedly generated.''

But when Dr. Donahue started working through the method to make sure he understood the basis for Kepler's chart, he found his numbers disagreeing with those of the great astronomer. After repeatedly getting the wrong answers for the numbers displayed on Kepler's chart, Dr. Donahue started trying other methods. Finally, he realized that the numbers in the chart had been generated not by independent calculations based on triangulated planetary positions, but by calculations using the area law itself.

''He was claiming that those positions came from the earlier theory,'' Dr. Donahue said. ''But actually all of them were generated from the ellipse.''


Thus, the notion that a planet orbits the Sun in an elliptical orbit was a simple fabrication, based on fudged data.

In fact, a strong argument for the validity of Newton’s laws of motion and gravity was that they could be used to derive Kepler’s laws.

But the entire Nova Astronomia was faked/falsified, each and every entry.

(https://image.ibb.co/hKmYdn/fake1.jpg)
(https://image.ibb.co/hOnH4S/fake2.jpg)

Mathematics applied to deterministic problems in the natural sciences (C.C. Lin/L.A. Segel), chapter 2: Deterministic systems and ordinary differential equations (pg. 36-70)

To accomplish a mathematical formulation, we adopt a polar coordinate system (r, θ) with the sun as the origin.

The second law of Kepler then states that, following the orbit (r(t), θ(t)) of a planet,

r2dθ/dt = h

The first law of Kepler states that the orbit can be described by the simple formula,

r = p/(1 + ecosθ)

Then one can show that the acceleration in the radial direction is

ar = d2r/dt2 - r(dθ/dt)2 = -h2/pr2

Thus the acceleration is inversely proportional to the square of the radial distance.

Newton, by combining the above results with his second law of motion, was led to formulate the present form of the law of universal gravitation.

This, in turn, leads to a system of N particles in gravitational interaction; e.g., the solar system comprising the sun and the nine major planets.


Title: Re: Round Earth Celestial Mechanics Cannot Predict the Solar System
Post by: edby on July 21, 2018, 09:31:57 PM
Donahue's article here http://adsabs.harvard.edu/full/1988JHA....19..217D.
Title: Re: Round Earth Celestial Mechanics Cannot Predict the Solar System
Post by: sandokhan on July 21, 2018, 09:34:35 PM
Since you do not like Charles Ginenthal's formidable works, here is an analysis of J. Laskar's assumptions using only mainstream sources.

Laskar's entire theory rests on the hypothesis that the Milankovitch cycle is correct (expansion and contraction of ice caps).

“A team led by Jacques Laskar . . . has released new computational results for
the long-term evolution and rotational motion of the Earth. Following
Milankovitch’s theory of paleoclimate that describe how major, climatic changes
of the Earth are affected by astronomical events, these [astronomical] results have
been employed to provide a new calibration to the sedimentary records over the 0
to 23.03 Myr [million years] geological period (the so-called Neogene period). . . .
It is the first time that astronomical computations have been used to establish . . .
geological chronology over a full geological period.”

Astronomy & Astrophysics, Jacques Laskar, “New results refine the Geological Time Scale” (Oct 25, 2004)

However, the data concerning the Milankovitch cycle totally contradicts Laskar's astronomical analysis.

“We are puzzled by the table in the Scientific Correspondence by Emiliani.
He rejects the conventionally used (glacial, interglacial transitions) as time
markers and focuses on bathythermals (the coldest portions of glacial cycles),
which he deems to be sharper and, therefore, more precise time markers. He
claims that bathythermals in the Devils Hole . . . chronology occur at times when
the orbital parameters of [the Earth’s] obliquity [axial tilt] and eccentricity to the
sun] are both low . . . thereby supporting the Milankovitch mechanism . . .
“We show [in a table] . . . the seven astronomical ‘low’ events that Emiliani
gives . . . We are puzzled as to why Emiliani omitted [from his table] two welldefined
‘low’ events . . . and note that they do not correspond to bathythermals in
either the Devils Hole or the marine chronologies. Indeed, the ‘low’ [or coldest]
event occurs during a peak interglacial time [when it was warmest]. We also note
that Emiliani’s designation of a ‘low’ event [for two periods] does not fit the earlier
stated definition.
“Also show[n] in our table] . . . are eight major . . . minima denoting times of
full glacial climate, found in the Devils Hole chronology, and the subset of six
events that Emiliani gives . . . in his table . . . He does not mention the two Devils
Hole isotope minima which do not correspond to an astronomical ‘low’ event.
“In comparing the astronomical ‘low’ events predicted by the specific definition
with the minimal isotope events in the Devils Hole chronology, one sees that though
there are four ‘matches,’ there are six ‘non-matches,’ twice when a bathythermals
would be predicted but did not happen, and four times when one did occur but not
during an astronomical ‘low’ event.”

J. M. Landwehr, Isaac J. Winograd, T. B. Copen, “No Verification of Milankovitch,” Nature, Vol. 368 (Apr 14, 1994), p. 594

“One of the fundamental tenets of paleoclimate modeling, the Milankovitch
theory, is called into doubt by isotope analyses of a calcite vein [Devils hole] just
reported in Science, by Winograd and colleagues. The [Milankovitch] theory,
which is backed up by a compelling bank of evidence, suggests that the ice ages are
driven by periodic variations in the Earth’s orbit. But the timing of the ice ages
determined, with unprecedented accuracy in the new [Devils Hole] record, cannot
be reconciled with planetary cyclicity.”

Walter S. Broecker, “Upset for Milankovitch Theory,” Nature, Vol. 359 (Oct 29, 1992), p. 779

Richard A. Kerr, editor of Science, published the following:

“The Devils Hole Record traced climate swings of about the same length as the
marine record, but they were out of step with the variations of Earth’s orbit. Most
glaringly, the carbonates indicated a profound warming trend which appeared to
signal the end of the penultimate ice age thousands of years before orbital variations
could have begun to melt ice. If the Devils Hole chronology was a true record of
the world’s ice age, researchers would have to dump the astronomical mechanism
and look for something new.”

Richard A. Kerr, “Second Clock Supports Orbital Pacing of the Ice Ages,” Science, Vol. 276 (May 2, 1997), p. 680

K. R. Ludwig, et al., write: “The Devils Hole data . . . remains a challenge to the
Milankovitch hypothesis.”

K. R. Ludwig, et al., “Last Interglaciation in Devils Hole, Nature, Vol. 362 (Apr 15, 1993), p. 596

“Perhaps the most convincing evidence that neither local nor regional
hydrologic or synoptic climatological factors are the predominant ones influencing
the DH [Devils Hole] . . . signal is the strong linear correlation of this record with
SPECMAP VOSTOK . . . [climate archives].
“Local or regional hydrologic or synoptic – climatological factors are
improbable as the predominant causes . . .”
“In my estimation, the New Devils Hole Chronology is more firm than another
available isotopic age in this range. Nowhere else has such a high degree of
concordance between – 238U and 230 Th and 230 Th – 234U ages been achieved.
No other archive is better preserved. No other record has so many stratigraphically
ordered radiometric ages . . . [Although Broecker was a long-time supporter of
Milankovitch, he adds] one side will have to give – just to be safe – climate
modelers should start preparing themselves for a world without Milankovitch.”

Broecker, “Upset for Milankovitch Theory,” p. 780

“The study of sediment cores from the deep Atlantic and ice cores from
Greenland does not confirm the gradual transition from glacial to interglacial and
back again that is implied by the astronomical [Milankovitch] cycles and partly
documented by the oxygen isotope record. Instead, as on the land the response of
the North Atlantic Ocean – atmosphere system looks like a series of abrupt flipflops
from one stage to another.”

Tjeed H. Van Andel, New Views of an Old Planet, 2ed. (Cambridge, UK 1994), p. 97

“Then there is the question of whether waxing and waning of ice sheets in the
Northern Hemisphere could, as the Milankovitch theory assumes, drive all the
other climate changes that have accompanied ice ages – the cooling and glaciation
of the Southern hemisphere, for example, glaciers in the Andes and Antarctica
have advanced at roughly the same times as those in the north, that is, at times
when the [Milankovitch] orbital calculations should have been getting a lot of
summer sunlight. There is no generally accepted explanation of why ice ages
should be globally synchronized if driven by [Milankovitch] orbital
fluctuations.”

Robert Kunzig, “Ice Cycles,” Discover (May 1989), p. 78

“. . . Slight changes in the tilt of the Earth’s axis give only negligible solar
variations at equatorial latitudes; yet the last ice age produced the great glaciers on
Mauna Loa and Mauna Kea in Hawaii and Mount Elgon in Uganda. Obviously
something drastic happened in the tropics for which the Milankovitch theory cannot
account.”
Walter Broecker, an earlier proponent of Milankovitch, had this to say in 1997:
“An important piece of information in this regard is the state of Earth’s system
during the extreme cold millenniums of glacial times. At these times, all of Canada
and a major part of the northeastern and mid-western United States were covered
by ice sheets. The snow line descended about 1 km [3280 feet] on mountains
elsewhere on Earth. Geomorphologists have traversed the globe comparing the
elevation of the present-day mountain snowlines with those for the last glaciation .
. . Everywhere from 40° S to 40° N [latitude] snowlines descended . . . Thus, the
southern Andes and New Zealand’s South Island, which now have very small
glaciers, had quite large ones.
“What this tells us is that somehow, [the entire] Earth was in a much colder
condition during the glacial periods. To my way of thinking, no one [including
Milankovitch] has adequately explained how this could happen. We now have new
evidence from glacial-age corals and from glacial-age ground water . . . that the
tropics may have been as much as 5° C (9° F) colder during glacial times. How
could the climate of the Earth have changed so much . . . ?”

Sir Fred Hoyle, Ice (NY 1981), p. 70.
Walter S. Broecker, GSA Today (May 1997), pp. 4-5


Laskar's entire 20 million year stability analysis is only based on the deeply flawed Milankovitch theory.

Title: Re: Round Earth Celestial Mechanics Cannot Predict the Solar System
Post by: edby on July 21, 2018, 09:43:48 PM
Since you do not like Charles Ginenthal's formidable works ..
Let's stick with Donahue. Anyone who translates these post medieval Latin works into English has done a great service to scholarship. This review suggests Donahue is on the mark.

https://www.jstor.org/stable/4027649?seq=1#page_scan_tab_contents

It is not unknown for scientists to fudge the raw data to support a hunch. The question is (i) how much did Kepler fudge the data and (ii) was he right? Is the orbit of Mars in fact elliptical, or not?

[edit]

Quote
Dr. Donahue, like many other experts, feels the episode does little to diminish Kepler's reputation. ''He had a difficult job trying to convince people that the ellipse was correct,'' he said. ''So he fudged a little. This doesn't take him down a notch. It was a small point in the argument.'' https://www.nytimes.com/1990/01/23/science/after-400-years-a-challenge-to-kepler-he-fabricated-his-data-scholar-says.html 
Title: Re: Round Earth Celestial Mechanics Cannot Predict the Solar System
Post by: sandokhan on July 22, 2018, 05:11:17 AM
Let's put your word to the test.

(https://image.ibb.co/ndKidb/kpl1.jpg)
(https://image.ibb.co/kenTdb/kpl2.jpg)
(https://image.ibb.co/js8vPG/kpl3.jpg)
(https://image.ibb.co/ch4oBw/kpl4.jpg)
(https://image.ibb.co/fFVH4G/kpl5.jpg)
(https://image.ibb.co/jT9Ydb/kpl6.jpg)

Kepler faked/fudged/falsified the entire set of data obtained from Brahe, and used the ellipse to calculate the final entries for his tables.

Kepler FAKED THE ENTIRE SET OF DATA, and announced to the world he got it from a nonexistent elliptical orbit.

The elliptical orbit WAS NOT based on observational astronomical data.

It was simply written in by Kepler.

As such, his book is a work of FICTION.

No science involved.

The observational input is nil.

Kepler portrayed the source of the tables as other than it was, with the obvious goal of making the elliptical hypothesis look as if it had greater computational support than it actually had.

That is why Kepler's work is a total fraud.

Kepler used the elliptical hypothesis to calculate the tables.


That is not the same as computing the Mars-Sun distances from Brahe's observational data (directly from observations).

Moreover the longitudes in Kepler's tables were calculated with the aid of the area law of the ellipse AND NOT from direct observational values.


Since, according to his own words Kepler had no idea of the correct form of the orbital path, HOW COULD HE KNOW IN ADVANCE HOW TO CALCULATE THE TABLES WITH THE AID OF THE ELLIPTICAL HYPOTHESIS?


“Almost 400 years later, William H. Donohue undertook the task of translating
Kepler’s 1609 Astronomia Nova into the English New Astronomy (Donohue 1992)
when in the course of his work he redid many of Kepler’s calculations, he was
startled to find some fundamental inconsistencies with Kepler’s reporting of these
same calculations (Donohue 1988). Writing of Donohue’s pathbreaking work in
The New York Times, William Broad (1990) summarized Donahue’s findings
saying that although Kepler claimed to have confirmed the elliptical orbit by
independent observations and calculations of the position of Mars, in fact Kepler
derived the data from the theory instead of the other way around . . .

After detailed computational arguments Donahue concluded the results
reported by Kepler . . . were not at all based on Brahe’s observational data; rather
they were fabricated on the basis of Kepler’s determination that Mars’s orbit was
elliptical
."

Kepler faked his entire set of data to match the ellipse.

The fabricated data appear in calculated positions for the planet Mars, which Kepler used as a case study for all planetary motion. Kepler claimed the calculations gave his elliptical theory an independent check. But in fact they did nothing of the kind.

''He fudged things,'' Dr. Donahue said, adding that Kepler was never challenged by a contemporary. A pivotal presentation of data to support the elliptical theory was ''a fraud, a complete fabrication,'' Dr. Donahue wrote in his paper. ''It has nothing in common with the computations from which it was supposedly generated.''

''He was claiming that those positions came from the earlier theory,'' Dr. Donahue said. ''But actually all of them were generated from the ellipse.''


There is no such thing as an elliptical orbit.


(http://image.ibb.co/iQwvTm/kpl8.jpg)

DONAHUE'S CALCULATIONS ARE BASED UPON TYCHO BRAHE'S DATA.

KEPLER'S FAKE ENTRIES RELY ON THE ELLIPSE.


How in the world could Kepler know in advance which geometrical path to use?

Kepler portrayed the source of the tables as other than it was, with the obvious goal of making the elliptical hypothesis look as if it had greater computational support than it actually had.

The only thing Kepler knew in advance was the fact that the circles with epicycles WERE EQUIVALENT TO THE ELLIPSE, and all he had to do is FAKE THE ENTRIES.

He faked all of the entries.


(https://image.ibb.co/kenTdb/kpl2.jpg)

KEPLER MODIFIED THE ENTRIES IN THE FINAL TABLE FOR CHAPTER 53: HE SIMPLY ADJUSTED THEM TO FIT THE ELLIPTICAL HYPOTHESIS WITH NO OBSERVATIONAL INPUT WHATSOEVER.

(https://image.ibb.co/js8vPG/kpl3.jpg)

For the longitudes, Kepler claimed to have used the vicarious hypothesis: yet, the calculations show he used the area law for the ellipse.

http://adsbit.harvard.edu/cgi-bin/nph-iarticle_query?bibcode=1988JHA....19..217D&db_key=AST&page_ind=12&plate_select=NO&data_type=GIF&type=SCREEN_GIF&classic=YES

http://adsbit.harvard.edu/cgi-bin/nph-iarticle_query?bibcode=1988JHA....19..217D&db_key=AST&page_ind=16&plate_select=NO&data_type=GIF&type=SCREEN_GIF&classic=YES

The only sheer work involved was that of faking and replacing the correct entries by fudged entries.

Kepler simply replaced everything with data which suited his purpose.

Is this what you call science?

There was no observational input at all.

None whatsoever.

Kepler portrayed the source of the tables as other than it was, with the obvious goal of making the elliptical hypothesis look as if it had greater computational support than it actually had.

That is why Kepler's work is a total fraud.

(https://image.ibb.co/js8vPG/kpl3.jpg)



Title: Re: Round Earth Celestial Mechanics Cannot Predict the Solar System
Post by: edby on July 22, 2018, 06:02:02 AM
I asked (i) how much did Kepler fudge the data and (ii) was he right? Is the orbit of Mars in fact elliptical, or not?

From the passages quoted by Donahue, quite a lot. On the other hand we have this, which I quote again:
Quote
Dr. Donahue, like many other experts, feels the episode does little to diminish Kepler's reputation. ''He had a difficult job trying to convince people that the ellipse was correct,'' he said. ''So he fudged a little. This doesn't take him down a notch. It was a small point in the argument.'' https://www.nytimes.com/1990/01/23/science/after-400-years-a-challenge-to-kepler-he-fabricated-his-data-scholar-says.html 

Then there is my second question: Is the orbit of Mars in fact elliptical, or not?
Title: Re: Round Earth Celestial Mechanics Cannot Predict the Solar System
Post by: sandokhan on July 22, 2018, 06:40:10 AM
After detailed computational arguments Donahue concluded the results
reported by Kepler . . . were not at all based on Brahe’s observational data; rather
they were fabricated on the basis of Kepler’s determination that Mars’s orbit was
elliptical."

(http://image.ibb.co/iQwvTm/kpl8.jpg)

DONAHUE'S CALCULATIONS ARE BASED UPON TYCHO BRAHE'S DATA.

KEPLER'S FAKE ENTRIES RELY ON THE ELLIPSE.


How in the world could Kepler know in advance which geometrical path to use?

Kepler portrayed the source of the tables as other than it was, with the obvious goal of making the elliptical hypothesis look as if it had greater computational support than it actually had.

The only thing Kepler knew in advance was the fact that the circles with epicycles WERE EQUIVALENT TO THE ELLIPSE, and all he had to do is FAKE THE ENTRIES.

He faked all of the entries.


For the NYT article, Dr. Donahue tried his best to minimize the extent of Kepler's monumental fudging.

Kepler claimed the calculations gave his elliptical theory an independent check. But in fact they did nothing of the kind.

''He fudged things,'' Dr. Donahue said, adding that Kepler was never challenged by a contemporary. A pivotal presentation of data to support the elliptical theory was ''a fraud, a complete fabrication,'' Dr. Donahue wrote in his paper. ''It has nothing in common with the computations from which it was supposedly generated.''

But when Dr. Donahue started working through the method to make sure he understood the basis for Kepler's chart, he found his numbers disagreeing with those of the great astronomer. After repeatedly getting the wrong answers for the numbers displayed on Kepler's chart, Dr. Donahue started trying other methods. Finally, he realized that the numbers in the chart had been generated not by independent calculations based on triangulated planetary positions, but by calculations using the area law itself.

''He was claiming that those positions came from the earlier theory,'' Dr. Donahue said. ''But actually all of them were generated from the ellipse.''

(https://image.ibb.co/fFVH4G/kpl5.jpg)

Of course the orbit of Mars is not elliptical: the data obtained by Tycho Brahe (circles with epicycles) is correct.

Kepler faked the entire data to give the impression that the orbit is elliptical in shape, which it is not.

The demarcation problem in the philosophy of science is about how to distinguish between science and non-science, including between science, pseudoscience, and other products of human activity, like art and literature, and beliefs.

Do you understand the meaning of the words fake/fudge/falsify?

Does faking an entire set of data to suit one's own purposes count as NON-SCIENCE?

Kepler's entire Nova Astronomia is a work of fiction: it is based entirely on fraudulent data.

Just take a look at the fake data published by Kepler:

(http://image.ibb.co/iQwvTm/kpl8.jpg)

Since, according to his own words Kepler had no idea of the correct form of the orbital path, HOW COULD HE KNOW IN ADVANCE HOW TO CALCULATE THE TABLES WITH THE AID OF THE ELLIPTICAL HYPOTHESIS?
Title: Re: Round Earth Celestial Mechanics Cannot Predict the Solar System
Post by: edby on July 22, 2018, 07:06:38 AM
Then there is my second question: Is the orbit of Mars in fact elliptical, or not?

https://en.wikipedia.org/wiki/Orbit_of_Mars
Title: Re: Round Earth Celestial Mechanics Cannot Predict the Solar System
Post by: edby on July 22, 2018, 08:13:54 AM
Quote
Neptune is not visible to the unaided eye and is the only planet in the Solar System found by mathematical prediction rather than by empirical observation. Unexpected changes in the orbit of Uranus led Alexis Bouvard to deduce that its orbit was subject to gravitational perturbation by an unknown planet. Neptune was subsequently observed with a telescope on 23 September 1846 by Johann Galle within a degree of the position predicted by Urbain Le Verrier.
https://en.wikipedia.org/wiki/Neptune
Title: Re: Round Earth Celestial Mechanics Cannot Predict the Solar System
Post by: sandokhan on July 22, 2018, 08:26:59 AM
Of course the orbit of Mars is not elliptical: the data obtained by Tycho Brahe (circles with epicycles) is correct.

The wikipedia page on the orbit of Mars contains two blatant falsehoods.

A key discovery was that the motion of Mars followed an elliptical path.

Kepler faked/fugded/falsified the entire crucial set of data in Chapter 53 pertaining to the supposed orbital path of Mars.

His model with a circular orbit did not match the observations of Mars.

Tycho Brahe's circle/epicycle model matched very precisely the observations of Mars, that is why Kepler had to falsify them in order to introduce to the public the heliocentric version.

Neptune

"The greatest triumph of the theory of gravitation was the discovery of the planet Neptune, the position of which was calculated simultaneously by Adams and Leverrier from the perturbations experienced by Uranus. But in the controversy which ensued concerning the priority in announcing the existence of Neptune, it was stressed that neither of the two scholars was the real discoverer, as both of them calculated very erroneously the distance of Neptune from the orbit of Uranus. Yet, even if the computations were correct, there would be no proof that gravitation and not another energy acts between Uranus and Neptune. The gravitational pull decreases as the square of the distance. Electricity and magnetism act in the same way. Newton was mistaken when he ascribed to magnetism a decrease that follows the cube of the distance (Principia, Book III, Proposition V, Corr. V)."


Scott Tremaine's arguments rest totally on Jacques Laskar's numerical simulations.

As we have seen, Laskar committed some monumental errors, not the least of which is the hypothesis that the Milankovitch cycle is correct.

“The word ‘chaotic’ summarizes many fundamental concepts characterizing
a dynamical system such as complex predictability and stability. But above
all, it acts as a warming of the difficulties which are likely to arise when trying to
obtain a reliable picture of its past and future evolution. As an example, a
commonly accepted definition states that a system is ‘unstable’ if the trajectories of
two points that initially are arbitrarily close . . . diverge quickly in time. This has
strong implications, as small uncertainties in initial conditions . . . might [also] be
consistent with completely different future trajectories: The conclusion is that we
can exactly reproduce the motion of a chaotic system only if WE KNOW, WITH
ABSOLUTE PRECISION, THE INITIAL CONDITIONS – A STATEMENT
THAT, IN PRACTICE, CAN NEVER BE TRUE."

Alessandra Celletti, Ettore Perozzi, Celestial Mechanics: The Waltz of the Planets

Sussman and Wisdom's 1992 integration of the entire solar system displayed a disturbing dependence on the timestep of the integration (measurement of the Lyapunov time).

Thus, different researchers who draw their initial conditions from the same ephemeris at different times can find vastly different Lyapunov timescales.

Wayne Hayes, UC Irvine

“Instead of using full equations of motion, Laskar focused on a special
formulation that spotlights gradual but cumulative changes in an orbit’s shape
[eccentricity] and orientation [inclination]. He worked with equations that smooth
out the recurring wiggles and wobbles in planetary orbits leaving only long term
trends . . .

“By applying a similar strategy to celestial curves [eccentricities], Laskar could
isolate these [non-gravitational] parts of a planet’s motion that correspond to lasting
changes in key characteristics of its orbit.”

Ivars Peterson, Newton’s Clock

“This differential system is a close approximation to the real solar system, and in
particular, the inner solar system . . . but the exact meaning of ‘close’ is still difficult to evaluate.”

J. Laskar

When one employs non-gravitational theory instead of “full equations of motion” to prove
stability, one has removed the solution of the problem from reality. But even believing in these
non-gravitational equations, Laskar cannot tell what a “close approximation” is. It is a theoretical
construct that comes out of non-gravitational math.

C. Ginenthal
Title: Re: Round Earth Celestial Mechanics Cannot Predict the Solar System
Post by: edby on July 22, 2018, 08:59:52 AM
You are quoting Velikovsky without attribution. It would help if you provide clear citations for all your sources.

It's very hard to find any literature on Velikovsky except Velikovsky himself, or his close supporters.

Quote
Since Adams and Leverrier expected to find a planet of the size of Uranus ca. 1,750,000,000 miles beyond the orbit of Uranus, and it was found ca. 1,000,000,000 miles beyond Uranus, the mass of Neptune was overestimated by a factor of three.
https://www.varchive.org/ce/cosmos.htm#f_31
What is the source for this claim?

[edit]
Ah.

Quote
The criticism was soon afterwards made, that both Adams and Le Verrier had been over-optimistic in the precision they claimed for their calculations, and both had, by using Bode's law, greatly overestimated the planet's distance from the sun.
https://en.wikipedia.org/wiki/Discovery_of_Neptune

Quote
The hypothesis correctly anticipated the orbits of Ceres (in the asteroid belt) and Uranus, but failed as a predictor of Neptune's orbit and was eventually superseded as a theory of Solar System formation. It is named for Johann Daniel Titius and Johann Elert Bode.
https://en.wikipedia.org/wiki/Titius%E2%80%93Bode_law

However, the difference in distance given in the Wikipedia table does not match Velikovsky's claim. So what is the source for Velikovsky's claim?
Title: Re: Round Earth Celestial Mechanics Cannot Predict the Solar System
Post by: sandokhan on July 22, 2018, 09:54:49 AM
https://physics.stackexchange.com/questions/202342/was-leverrier-adams-prediction-of-neptune-a-lucky-coincidence

https://books.google.ro/books?id=kKQxhFSarkoC&pg=PT99&lpg=PT99&dq=adams+leverrier+neptune+overestimated+the+orbit+of+neptune&source=bl&ots=-gxgmzkLlV&sig=MMl6uJyuk9vPVorZNc3lBKz0W8Y&hl=ro&sa=X&ved=0ahUKEwj6iea3trLcAhUmQpoKHe6ICXgQ6AEIaDAH#v=onepage&q=adams%20leverrier%20neptune%20overestimated%20the%20orbit%20of%20neptune&f=false

Title: Re: Round Earth Celestial Mechanics Cannot Predict the Solar System
Post by: edby on July 22, 2018, 09:58:22 AM
https://physics.stackexchange.com/questions/202342/was-leverrier-adams-prediction-of-neptune-a-lucky-coincidence

https://books.google.ro/books?id=kKQxhFSarkoC&pg=PT99&lpg=PT99&dq=adams+leverrier+neptune+overestimated+the+orbit+of+neptune&source=bl&ots=-gxgmzkLlV&sig=MMl6uJyuk9vPVorZNc3lBKz0W8Y&hl=ro&sa=X&ved=0ahUKEwj6iea3trLcAhUmQpoKHe6ICXgQ6AEIaDAH#v=onepage&q=adams%20leverrier%20neptune%20overestimated%20the%20orbit%20of%20neptune&f=false
These were Velikovsky's sources? He died in 1979.
Title: Re: Round Earth Celestial Mechanics Cannot Predict the Solar System
Post by: sandokhan on July 22, 2018, 09:59:34 AM
Now, since you do not like quotes attributed to Velikovsky, here is a direct proof that Venus is a very young planet,  a fact which demolishes the claims made by modern astrophysics pertaining to the age of the solar system.

VENUS’ ARGON-36 AND ARGON-40 AGE

https://www.theflatearthsociety.org/forum/index.php?topic=30499.msg1938506#msg1938506

VENUS’ CARBON DIOXIDE AGE

https://www.theflatearthsociety.org/forum/index.php?topic=30499.msg1938793#msg1938793

VENUS’ NEON KRYPTON AGE

https://www.theflatearthsociety.org/forum/index.php?topic=30499.msg1938826#msg1938826

You did mention Mars before, so you are going to have to explain this also:

MARS’ NITROGEN-15 AGE

https://www.theflatearthsociety.org/forum/index.php?topic=30499.msg1938902#msg1938902


VENUS AND EARTH SPIN-ORBIT RESONANCE

P. Goldreich, CalTech

S.J. Peale, UCSB

https://www.researchgate.net/publication/232751781_Resonant_Rotation_for_Venus

http://adsabs.harvard.edu/full/1970AJ.....75..273G

Is there evidence, not probability mathematics but actual evidence that argues that Venus must have had a near collision with the Earth? Gravitational theory holds that when celestial bodies come close and interact, then there should remain some lingering remnant in some part of the orbital pattern of both bodies.

“…a discovery was announced by P. Goldreich of CalTech and S.J. Peale of the University of California, and reported at the annual meeting of the American Geophysical Union on April 23, 1966. The surprising discovery dealt with the axial rotation of Venus, already known to be slow and retrograde. Every time Venus passes between the sun and the Earth, it turns the same face to the earth. Gravitationally, this phenomenon cannot be explained even if Venus were lopsided, as some science writers have offered as the explanation, it would have been locked with the very same face toward the sun, whose gravitational pull on Venus is so much stronger than that of the earth; this ‘resonance’ as the discoverers of the phenomenon termed it, if confirmed, is a sure piece of evidence of close contact in the past between Earth and Venus, evidence not erased by the passage of time, in this case time measured in a mere few thousand years.”

An article titled “Venus and Earth: Engaged or Divorced?” in Astronomy (Vol. 7 for Oct. 1979), p. 58, discussed I.I. Shapiro and his colleague’s analysis of the Venus-Earth resonance. They note radar observations gathered over a 14 year period of time has permitted them to nail down Venus’ rotation period with high precision.

“They find it to be 243.01 +- 0.03 days. The 3 1/2 hour difference between this value and the resonance period of exactly 243.16 days; while very small, is statistically significant. On the other hand, the researchers point out that the probability of Venus’ rotation period falling by chance alone within one-fifth of a day of a resonance period is under 1%. Therefore, they suggest that Venus could either now be evolving toward such a resonance, or was once in resonance in the recent past.”

William R. Corliss who reported this article in The Moon and the Planets, (1985), p. 304, adds this remark,

“The possibility of a recent or imminent resonance is redolent of a recent solar system instability. It would be interesting if ‘recent’ means ‘within the time of man’ to that there would after all be astronomical explanations of many legends of celestial turmoil.”

Zdenek Kopal in The Realm of the Terrestrial Planets, (NY 1979) p. 180 informs us that:

“The remarkable resonance…between the synodic orbit of Venus and its axial rotation with respect to the Earth is certainly not accidental. It strongly suggests the existence of tidal coupling [Kopal’s emphasis] between the two neighboring planets, but the specific mechanism which could lead to its establishment is largely obscure…a…coupling between Venus and the Earth—a body much less massive [than the Sun]—constitutes a real challenge to our understanding.”

James Oberg further explains how difficult it is for scientists to account for this phenomenon,

“The best explanation for this close resonance (and for the fact that the Venusian year is within a few hours of being exactly 8/13 of Earth’s year), to appeal to coincidence—an unsatisfactory solution at best. Nagging doubts insist that something vital is missing from the logic involved. The best current theory [for Venus’ retrograde rotation] calls for a large off-centered asteroid impact late in Venus’ formation phase. This presents difficulties. Such an accident could reverse the spin but could not account for the spin axis being at near right angles to the plane of the orbit (an extremely unlikely result in a freak collision). If the spin reversing collision could set up nearly any new axis, but this axis would eventually wander back to its old position because of the planet’s oblateness. Such oblateness could have disappeared over millions of years that passed while the new slow rotation rate no longer provided sufficient centrifugal force. If this explanation sounds like magic its the best there is. Astronomers remain completely baffled.” [Oberg, “Venus” Astronomy (August 1976), p. 16].

Zdenek Kopal, above page 191, puts the problem this way,

“The first problem concerns the rotation of the planets. What made Venus rotate so slowly, and what tilted its axis of rotation almost upside down to give rise to its retrograde rotation. The only probable mechanism would be a very close encounter with another celestial body whose gravitational attraction played havoc with Venus and altered some of its kinematics [motions] and at the same time cause it to lock onto the Earth gravitationally?”

The answer is an interaction with the Earth. Here is what appears to be clear evidence based not on the probability theory, but on gravitational theory.

It indicates that Venus’ axial rotation is locked onto the Earth and not onto the Sun. Hoimar Von Ditfurth in Children of the Universe, (NY 1976), p. 115 remarks that, “the Earth must once have exerted a braking or decelerating effect on Venus until the two planets mutual gravitational attraction brought about the ‘coupling’ we observe today.” To do so, the Earth and Venus had to be quite close to each other for their gravitational fields to be effective in creating this couple effect. If the Earth and Venus never had a near encounter then any gravitational anomaly on Venus would cause it to lock onto the Sun. The Earth’s gravitational field is far too small compared with that of the Sun to nudge Venus into such a resonance.
Title: Re: Round Earth Celestial Mechanics Cannot Predict the Solar System
Post by: edby on July 22, 2018, 10:15:48 AM
Now, since you do not like quotes attributed to Velikovsky,
I didn't say that. I was wondering what his sources were.
Title: Re: Round Earth Celestial Mechanics Cannot Predict the Solar System
Post by: edby on July 22, 2018, 03:40:45 PM
An article titled “Venus and Earth: Engaged or Divorced?” in Astronomy (Vol. 7 for Oct. 1979), p. 58, discussed I.I. Shapiro and his colleague’s analysis of the Venus-Earth resonance. They note radar observations gathered over a 14 year period of time has permitted them to nail down Venus’ rotation period with high precision.

Doesn’t the radar technique work by focusing a radio beam onto the planet and working out the ‘echo time’?
http://pds-geosciences.wustl.edu/venus/arcb_nrao-v-rtls_gbt-3-delaydoppler-v1/vrm_90xx/document/venus_radar.pdf

Quote
Individual looks are limited to about 5 minutes duration by the round-trip light time for Venus near inferior conjunction.
How fast is the speed of light?
Title: Re: Round Earth Celestial Mechanics Cannot Predict the Solar System
Post by: sandokhan on July 22, 2018, 04:16:40 PM
The speed of light is variable: it varies with the density of various layers of aether/ether.

The martian faint young sun paradox defies Laskar's stability of the solar system calculations:

https://www.theflatearthsociety.org/forum/index.php?topic=30499.msg1775118#msg1775118

There is only one way out, for astronomers to accept that Mars was recently an inner planet (orbiting closer to the Sun); a massive collision lead to the loss of its atmosphere/oceans.
Title: Re: Round Earth Celestial Mechanics Cannot Predict the Solar System
Post by: BillO on July 23, 2018, 06:23:45 PM
The speed of light is variable: it varies with the density of various layers of aether/ether.
Okay, your belief in an aether/ether puts your entire discourse into deep doubt.

There is no evidence for an aether.  There is no test for an aether.  There is no way to observe and aether.

If it walks like nothing, quacks like nothing and smells like nothing - it's nothing.

Next thing is you'll be believing in a flat earth.  ::)
Title: Re: Round Earth Celestial Mechanics Cannot Predict the Solar System
Post by: BillO on July 25, 2018, 09:04:08 PM
Tom is so utterly worng on this, and it can easily be proven.

Back in 1980 when I had just wrapped up community college and got my fist job I rewarded myself with an Apple II+.  One of the fist things I did on that machine was write a celestial mechanics simulation based on an article in Popular Electronics, Byte or Scientific American (back when it was a decent magazine).  All it uses is Newton's law of universal gravitation.  When I saw this thread I remembered it but was sure it had been lost forever, however yesterday I found a few old floppy disks and sure enough my program was there and readable.  Now, this is a dreadfully simple program with no fancy GR or corrections for coarse step-wise calculation approximations, and is further limited by the dismal Applesoft floating point precision yet it will faithfully run a simulation of the Sun, Mercury, Venus and Earth for the equivalent of hundreds of years - without the orbits getting chaotic and looking like knots.  It can handle more than 4 bodies too, but that is stressing it's abilities.  Tom would have us believe that folks with access to the worlds fastest and most precise computers - people with PhDs and in astrophysics and years of experience with numerical methods can't get it to work.  I have no idea where Tom digs this BS up from.

I've been running the program on both real hardware (I have an old Apple //e) and on the AppleWin emulator under Windoze 10 for hours now and no issues whatsoever.  The instance running on the emulator simulates about 211 years per hour (~17 seconds for earth's orbit).  If anyone is interested I will post the code and a couple of samples as well as an explanation on how to run it.  It's pretty simple.  Perhaps someone could port it to C or Python to run natively on a PC so that we can even get more speed and precision.

Just let me know.
Title: Re: Round Earth Celestial Mechanics Cannot Predict the Solar System
Post by: Tom Bishop on July 25, 2018, 09:32:01 PM
You solved all of the the n-body problems? Amazing. Why not release your wonder to the world?
Title: Re: Round Earth Celestial Mechanics Cannot Predict the Solar System
Post by: markjo on July 25, 2018, 09:36:08 PM
You solved all of the the n-body problems? Amazing. Why not release your wonder to the world?
Tom, are you suggesting that a numerical solution to the n-body problem is not a valid solution?
Title: Re: Round Earth Celestial Mechanics Cannot Predict the Solar System
Post by: Tom Bishop on July 25, 2018, 10:21:30 PM
The numerical solutions are few and far between, are special cases that require bodies of equal mass or no mass, and cannot create heliocentric models, such as a stable heliocentric earth-moon system rotating around the sun.


Super Computing Challenge

Students participate in a programming challenge to simulate n-body orbits and the solar system:

http://www.supercomputingchallenge.org/14-15/finalreports/21.pdf

Quote
Simulation of Planetary Bodies in the Universe (N-Body)
New Mexico Supercomputing Challenge

Our solar system is an N-body system. N-body simulation is the simulation of astral bodies under gravity, using laws of classical mechanics to define how the astral bodies move. The goal of our project is to model the N-body problem in NetLogo. Code defining how astral bodies interact implements the inverse square law and a gravitational constant to calculate gravitational force between them.

Quote
Verification and Validation

Even though our model is not entirely accurate, it recreates with graphical simplicity and mathematical correctness of the N-body simulation. Through many many trials, we realize that normal orbits are incredibly complex and hard to obtain through any normal means, and causes us to conclude that our own solar system is an incredible anomaly of the universe

Who would have thought?

The paper discusses various other speculations surrounding their failure to even create basic orbits through "many many trials." What a special anomaly of the universe our solar system is! It is beyond the understanding of human science to create it! Surely its is the solar system that special, rather than something wrong with the ideas that are trying to be simulated.

The Heliocentric Solar System Cannot be Simulated
Title: Re: Round Earth Celestial Mechanics Cannot Predict the Solar System
Post by: markjo on July 25, 2018, 10:51:57 PM
The numerical solutions cannot create heliocentric models, such as a stable heliocentric earth-moon system rotating around the sun.
What degree of stability are you referring to over what time span?  You do realize that the solar system that we know and love today is far different from its younger, formative years.  It's fairly well accepted that the orbits of the various planets have changed quite a lot over the last few billion years, so your talk of stability is a bit of a red herring.
http://www2.ess.ucla.edu/~jewitt/kb/migrate.html
https://www.space.com/28901-wandering-jupiter-oddball-solar-system.html
Title: Re: Round Earth Celestial Mechanics Cannot Predict the Solar System
Post by: Tom Bishop on July 25, 2018, 10:58:37 PM
What degree of stability are you referring to over what time span?

Any time span. Heliocentric orbits cannot be created in n-body simulations at all.

Here is an N-Body Orbit Gallery, which showcases the limited orbits that can be made, and which must assume that the bodies are of equal mass or mass-less.

http://rectangleworld.com/demos/nBody/
Title: Re: Round Earth Celestial Mechanics Cannot Predict the Solar System
Post by: Rama Set on July 25, 2018, 11:24:01 PM
Quote
Verification and Validation

Even though our model is not entirely accurate, it recreates with graphical simplicity and mathematical correctness of the N-body simulation.

Hi Tom, why did you ignore the part where they said they recreated the solar system?  That would seem to contradict the thing you are saying. Yet you quoted it. Very, very disturbing b
Title: Re: Round Earth Celestial Mechanics Cannot Predict the Solar System
Post by: Tom Bishop on July 25, 2018, 11:34:29 PM
Quote
Verification and Validation

Even though our model is not entirely accurate, it recreates with graphical simplicity and mathematical correctness of the N-body simulation.

Hi Tom, why did you ignore the part where they said they recreated the solar system?  That would seem to contradict the thing you are saying. Yet you quoted it. Very, very disturbing b

FFS, read the paper.

Quote
As orbit hasn't been observed in the model

Quote
The next thing to do with the model is to further research how orbit fits into the n-body problem, and to look at other models in NetLogo that simulate these ideas so that we may identify what ours is missing and why we have not seen orbit.
Title: Re: Round Earth Celestial Mechanics Cannot Predict the Solar System
Post by: Rama Set on July 25, 2018, 11:39:34 PM
Your quote. Regardless, as us been mentioned, numerical solutions aplenty abound for n-body problems and they are solved this way routinely. Furthermore our solar system is modeled very accurately as an aggregate of 2-body systems. To say our solar system cannot be modeled is, at its most generous, an incredibly ignorant thing to say.
Title: Re: Round Earth Celestial Mechanics Cannot Predict the Solar System
Post by: Rama Set on July 25, 2018, 11:50:26 PM
Also, I am not sure we should be taking that supercomputing competition too seriously, at the very least with a grain of salt. I thought there was something odd about that report since it’s syntax reads like it was translated to English from another language by Google. Turns out it is a competition for New Mexico high school students to stimulate interest in STEM fields. Not saying they the report should be outright ignored, but there is good reason not to take it as representative of current scientific achievements, to say nothing of authoritative.
Title: Re: Round Earth Celestial Mechanics Cannot Predict the Solar System
Post by: markjo on July 26, 2018, 12:42:55 AM
What degree of stability are you referring to over what time span?

Any time span. Heliocentric orbits cannot be created in n-body simulations at all.

Here is an N-Body Orbit Gallery, which showcases the limited orbits that can be made, and which must assume that the bodies are of equal mass or mass-less.

http://rectangleworld.com/demos/nBody/
I'm sorry, but I thought you were talking about modeling our solar system.  You know, a solar system with a number of relatively tiny bodies orbiting one massive body. 

So tell me Tom, where does the n-body problem stipulate that all bodies must be of approximately the same mass?
Title: Re: Round Earth Celestial Mechanics Cannot Predict the Solar System
Post by: Tom Bishop on July 26, 2018, 01:15:32 AM
The n-body situations with more than two bodies need bodies of the same mass (or some of the bodies mass-less) because that is the only way to bring equilibrium to the system. Otherwise, with unequal masses, the system attempts to kick out the smallest body, and the system often falls apart entirely. Progress can only be made if we assume such wacky scenarios.

Look into George Hill's work on the Three Body Problem and heliocentric orbits. When you add in a third mass to a two body system, it just goes crazy as the system attempts to kick out the smallest body, often tearing apart the system or destroying it in the process. The only way Hill was able to make any progress at all was by using the Restricted Three Body Problem. The Restricted Three Body Problem assumes that the mass of the moon is zero, and even then, the mass-less moon still goes crazy in its orbits around the earth. The only benefit of the Restricted Three Body Problem and the Mass-less moon is that the moon is no longer ejected from the system. It is confined to what is known as "Hill's Region".

http://www.scholarpedia.org/article/Three_body_problem

(https://i.imgur.com/vgnLaWl.gif)

As seen above, the moon will go crazy and do random u-turns.

From the text that accompanies the image:

Quote
The simplest case:

It occurs when, the Jacobi constant being negative and big enough, the zero mass body (we shall still call it the Moon) moves in a component of the Hill region which is a disc around one of the massive bodies (the Earth). This fact already implies Hill's rigorous stability result: for all times such a Moon would not be able to escape from this disc. Nevertheless this does not prevent collisions with the Earth.

Other solutions to the Restricted Three Body Problem (which assumes a mass-less moon), as applied to the Sun-Earth-Moon system, are presented here in my article: https://wiki.tfes.org/Celestial_Mechanics_Cannot_Predict_The_Solar_System#The_Best_of_the_Best

(https://wiki.tfes.org/images/thumb/0/08/Three_Body_Problem.jpg/1050px-Three_Body_Problem.jpg)

Do these look like Heliocentric Orbits to you?
Title: Re: Round Earth Celestial Mechanics Cannot Predict the Solar System
Post by: BillO on July 26, 2018, 01:29:59 AM
You solved all of the the n-body problems? Amazing. Why not release your wonder to the world?
Typical FE'er exaggeration.  Where exactly did I say I solved all of the n-body problems?

Here is the Applesoft Basic code
Code: [Select]
20  INPUT "ENTER NUMBER OF OBJECTS";N
 30  DIM M(10),A(10,2),S(10,2),V(10,2),LP(10),LQ(10)
 50 G = 6.67E - 11
 60  FOR I = 1 TO N
 70  PRINT : PRINT "FOR OBJECT ";I;", ENTER:"
 90  INPUT "MASS : ";M(I)
 110  INPUT "X    : ";S(I,1)
 130  INPUT "Y    : ";S(I,2)
 150  INPUT "VX   : ";V(I,1)
 170  INPUT "VY   : ";V(I,2)
 180  NEXT I
 190  PRINT
 200  INPUT "SCALE FACTOR FOR X ";SX
 210 SY = SX * .85
 220  INPUT "TIME STEP DURATION ";DT
 240  HGR : POKE  - 16302,0
 250  GOSUB 440
 260  FOR I = 1 TO N
 270 V(I,1) = V(I,1) - .5 * A(I,1) * DT
 280 V(I,2) = V(I,2) - .5 * A(I,2) * DT
 290  NEXT I
 300  FOR I = 1 TO N
 310 CX = .5 * A(I,1) * DT
 320 CY = .5 * A(I,2) * DT
 330 VX = V(I,1) + CX
 340 VY = V(I,2) + CY
 350 S(I,1) = (CX + VX) * DT + S(I,1)
 360 S(I,2) = (CY + VY) * DT + S(I,2)
 370 V(I,1) = VX + CX
 380 V(I,2) = VY + CY
 390  NEXT I
 410  GOSUB 600
 420  GOSUB 440
 430  GOTO 300
 440  FOR I = 1 TO N
 450 A(I,1) = 0:A(I,2) = 0
 470  NEXT I
 480  FOR I = 1 TO N - 1
 490  FOR J = I + 1 TO N
 500 RD = ((S(I,1) - S(J,1)) ^ 2 + (S(I,2) - S(J,2)) ^ 2) ^ 1.5
 510 A1 = G * M(J) * ((S(J,1) - S(I,1)) / RD)
 520 A2 = G * M(J) * (S(J,2) - S(I,2)) / RD
 530 A(I,1) = A(I,1) + A1
 540 A(I,2) = A(I,2) + A2
 550 A(J,1) = A(J,1) - A1 * M(I) / M(J)
 560 A(J,2) = A(J,2) - A2 * M(I) / M(J)
 570  NEXT J
 580  NEXT I
 590  RETURN
 600  FOR I = 1 TO N
 610  IF  ABS (S(I,1)) > SX THEN 690
 620  IF  ABS (S(I,2)) > SY THEN 690
 630 P = 139 +  INT (.5 + 140 * S(I,1) / SX)
 640 Q = 95 +  INT (.5 + 96 * S(I,2) / SY)
 650  HCOLOR= 3: HPLOT P,Q
 660  IF P = LP(I) AND Q = LQ(I) THEN 690
 670  HCOLOR= 0: HPLOT LP(I),LQ(I)
 680 LP(I) = P:LQ(I) = Q
 690  NEXT I
 700  RETURN

Enter masses in kilograms (kg)
Distances in meters (m)
Velocities in meters per second (m/s)


The scale factor should be somewhat greater than the  radius of the largest orbit.
The time step duration is a balance between accuracy and speed the lower the number the more accurate, but the slower the simulation will run.  See the examples and play around with it.

When you enter a body, you need to enter the correct speed that goes with the distance you choose.  I usually enter perihelion and the minimum orbital speed.  Don't try to enter the average distance and average speed as those measurements usually do not align.  For simplicity sake I always line up the bodies along the center axis (Y=0)

Here are two simulations.  The first is the Earth and moon 2 body system, the 2nd is the Sun, Mercury, Venus and Earth 4 body system.
Code: [Select]
Earth-Moon (2 Objects)

Earth
M1 = 5.9723E24
X1 = 0
Y1 = 0
VX1 = 0
VY1 = -10

Moon
M2 = 7.346E22
X2 = 4.055E8
Y2 = 0
VX1 = 0
VY2 = 970

Scale = 1E9
Time Step = 50000


Sun-Mercury-Venus-Earth (4 Objects)

Sun
M1 = 1.9885E30
X1 = 0
Y1 = 0
VX1 = 0
VY1 = 0

Mercury
M2 = 3.3011E23
X2 = 6.983E10
Y2 = 0
VX = 0
VY = 38860

Venus
M3 = 4.8675E24
X3 = -1.0894E11
Y3 = 0
VX3 = 0
VY3 = -34790

Earth
M4 = 5.9724E24
X3 = 1.5210E11
Y3 = 0
VX3 = 0
VY3 = 29290

Scale = 2E11
Time Step = 50000

I've run the 4 body example for 10 hours.  That's 2110 years of time simulated.  I'm going to set it to run overnight.

AS I said, the will run unaltered on a real Apple II with Applesotf Basic, o ron the AppleWIN emulator.   However, feel free to port to something else if you wish.
Title: Re: Round Earth Celestial Mechanics Cannot Predict the Solar System
Post by: BillO on July 26, 2018, 01:48:54 AM

http://www.scholarpedia.org/article/Three_body_problem

https://wiki.tfes.org/Celestial_Mechanics_Cannot_Predict_The_Solar_System#The_Best_of_the_Best

Do these look like Heliocentric Orbits to you?
Neither of these are attempts at numerical solutions/simulations.  They are both the result of trying to find an analytic solution and run the solution on a computer to test it.  The two approaches use computers but are entirely different.  There is no analytic solution (yet), but numerical solutions exist and are deadly accurate.  You've been told this by many people in this thread.  You're mixing things up because you don't understand what your dealing with.

My butt simple numerical solution is there for you or anyone else to try.  I'll give you all the help you need to get it going and to understand it.
Title: Re: Round Earth Celestial Mechanics Cannot Predict the Solar System
Post by: Tom Bishop on July 26, 2018, 01:58:02 AM

http://www.scholarpedia.org/article/Three_body_problem

https://wiki.tfes.org/Celestial_Mechanics_Cannot_Predict_The_Solar_System#The_Best_of_the_Best

Do these look like Heliocentric Orbits to you?
Neither of these are attempts at numerical solutions/simulations.  They are both the result of trying to find an analytic solution and run the solution on a computer to test it.  The two approaches use computers but are entirely different.  There is no analytic solution (yet), but numerical solutions exist and are deadly accurate.  You've been told this by many people in this thread.  You're mixing things up because you don't understand what your dealing with.

Incorrect

These are numerical solutions. The Runge-Kutta method is demonstrated in the second image. The Rung-Kutta method is a numerical method.

https://en.wikipedia.org/wiki/Runge%E2%80%93Kutta_methods

Quote
In numerical analysis, the Runge–Kutta methods are a family of implicit and explicit iterative methods, which include the well-known routine called the Euler Method, used in temporal discretization for the approximate solutions of ordinary differential equations

https://www.intmath.com/differential-equations/12-runge-kutta-rk4-des.php

Quote
Runge-Kutta is a common method for solving differential equations numerically. It's used by computer algebra systems.


Quote from: BillO
My butt simple numerical solution is there for you or anyone else to try.  I'll give you all the help you need to get it going and to understand it.

I seriously doubt that your magazine-provided solar system sim does what you think it does. I will have a look when I get a chance.
Title: Re: Round Earth Celestial Mechanics Cannot Predict the Solar System
Post by: markjo on July 26, 2018, 03:10:20 AM
The n-body situations with more than two bodies need bodies of the same mass (or some of the bodies mass-less) because that is the only way to bring equilibrium to the system.
What makes you think that our solar system is in equilibrium? ???

Otherwise, with unequal masses, the system attempts to kick out the smallest body, and the system often falls apart entirely.
First of all, small bodies are booted out of our solar system fairly regularly.  Secondly, collisions among bodes happen quite often too (especially in the early part of the solar system's history).

Progress can only be made if we assume such wacky scenarios.
Progress?  ???
Title: Re: Round Earth Celestial Mechanics Cannot Predict the Solar System
Post by: totallackey on July 26, 2018, 12:18:57 PM
All it uses is Newton's law of universal gravitation.  When I saw this thread I remembered it but was sure it had been lost forever, however yesterday I found a few old floppy disks and sure enough my program was there and readable.  Now, this is a dreadfully simple program with no fancy GR or corrections for coarse step-wise calculation approximations, and is further limited by the dismal Applesoft floating point precision yet it will faithfully run a simulation of the Sun, Mercury, Venus and Earth for the equivalent of hundreds of years - without the orbits getting chaotic and looking like knots...
So, it is totally useless and crapola, uh..?
needless and mindless words snipped for brevity becasue this does not even have a moving sun included and actually demonstrates ZERO use of Newton's formulas.
Yep, absolute and verifiable bs...
Title: Re: Round Earth Celestial Mechanics Cannot Predict the Solar System
Post by: BillO on July 26, 2018, 02:28:59 PM
Incorrect

These are numerical solutions. The Runge-Kutta method is demonstrated in the second image. The Rung-Kutta method is a numerical method.

https://en.wikipedia.org/wiki/Runge%E2%80%93Kutta_methods

Quote
In numerical analysis, the Runge–Kutta methods are a family of implicit and explicit iterative methods, which include the well-known routine called the Euler Method, used in temporal discretization for the approximate solutions of ordinary differential equations

https://www.intmath.com/differential-equations/12-runge-kutta-rk4-des.php

Quote
Runge-Kutta is a common method for solving differential equations numerically. It's used by computer algebra systems.
Yeah, I know the Runge-Kutta methods and what they are for - you apparently don't.

Going back to the:http://www.scholarpedia.org/article/Three_body_problem#Equations (http://www.scholarpedia.org/article/Three_body_problem#Equations)  the entire preamble is using mathematical analysis to derive the Hamiltonians which they then use to create a system of differential equations which then need to be integrated - in their case by computer.

You do know what is meant by mathematical analysis, don't you?  Calculus is one of the main processes of mathematical analysis.  If a solution is arrived by trough doing calculus (especial integration) it is an analytic solution, whether or not that integration was done by a machine or not.  Your references are attempting to compute an analytic solution.  Whether or not they use Runge-Kutta methids or not to do the integration.

Quote
I seriously doubt that your magazine-provided solar system sim does what you think it does. I will have a look when I get a chance.
Well, this should be interesting and a test of you intellectual honesty.

Yes the magazine provided the algorithm, but it is simple to understand and I really don't need you to 'splain it to me.  For each object defined: use Newton's law of universal gravitation to determine the net force on it due to each of the other other bodies.  Use Newtons 2nd law of motion to calculate the change in velocity.  Move the object one step.  Repeat.

Title: Re: Round Earth Celestial Mechanics Cannot Predict the Solar System
Post by: BillO on July 26, 2018, 02:38:03 PM
All it uses is Newton's law of universal gravitation.  When I saw this thread I remembered it but was sure it had been lost forever, however yesterday I found a few old floppy disks and sure enough my program was there and readable.  Now, this is a dreadfully simple program with no fancy GR or corrections for coarse step-wise calculation approximations, and is further limited by the dismal Applesoft floating point precision yet it will faithfully run a simulation of the Sun, Mercury, Venus and Earth for the equivalent of hundreds of years - without the orbits getting chaotic and looking like knots...
So, it is totally useless and crapola, uh..?
This is really not worth replying to ... but  I never said it was going to be anything but a demonstration that purely numerical methods can produce stable systems of multiple bodies.  It does.  q.e.d.

needless and mindless words snipped for brevity becasue this does not even have a moving sun included and actually demonstrates ZERO use of Newton's formulas.
Ahhh, that's odd as there at least 2 of them in there.  My guess is you don't have much in the way of math or computer skills.  A bit of blustering then?  Would you care to demonstrate how the sun's motion would be significant to the orbital dynamics of the solar system?


Yep, absolute and verifiable blustering...
Title: Re: Round Earth Celestial Mechanics Cannot Predict the Solar System
Post by: Tom Bishop on July 26, 2018, 10:15:47 PM
Regarding the Pop-Science Magazine Solar System Sim written in Applesoft BASIC

I took a look at this one. It appears to merely be creating 2-body orbits around a static sun. It does not appear to be a three-body or n-body problem simulator.

I placed the code from the last page into an online Applesoft BASIC Emulator here: http://www.calormen.com/jsbasic/

Using the settings Bill gave, I made two Earths, rotating in opposite directions, in the same orbit.

(https://i.imgur.com/UkIimUH.png)

Full settings:

Code: [Select]
Sun-Earth1-Earth2 (3 Objects)

Sun
M1 = 1.9885E30
X1 = 0
Y1 = 0
VX1 = 0
VY1 = 0

Earth 1
M2 = 5.9724E24
X2 = 1.5210E11
Y2 = 0
VX2 = 0
VY2 = 29290

Earth 2
M3 = 5.9724E24
X3 = 0
Y3 = 1.5210E11
VX3 = 29290
VY3 = 0

Scale = 2E11
Time Step = 50000

Here were the results:

(https://i.imgur.com/XGHHpu1.gif)
Title: Re: Round Earth Celestial Mechanics Cannot Predict the Solar System
Post by: Tom Bishop on July 26, 2018, 11:14:46 PM
Yeah, I know the Runge-Kutta methods and what they are for - you apparently don't.

Going back to the:http://www.scholarpedia.org/article/Three_body_problem#Equations (http://www.scholarpedia.org/article/Three_body_problem#Equations)  the entire preamble is using mathematical analysis to derive the Hamiltonians which they then use to create a system of differential equations which then need to be integrated - in their case by computer.

You do know what is meant by mathematical analysis, don't you?  Calculus is one of the main processes of mathematical analysis.  If a solution is arrived by trough doing calculus (especial integration) it is an analytic solution, whether or not that integration was done by a machine or not.  Your references are attempting to compute an analytic solution.  Whether or not they use Runge-Kutta methids or not to do the integration.

Poliastro says that the image represents numerical methods for the Restricted Three Body Problem. Are you to assert that you know better than an astrodynamics software developer?

https://twitter.com/poliastro_py/status/993418078036873216

Quote from: poliastro
Look at this beautiful plot of several numerical methods for the restricted three body problem taken from Harier et al. "Solving Ordinary Differential Equations I". The use of high order Runge-Kutta methods is pervasive in Celestial Mechanics. Happy Monday!

https://pbs.twimg.com/media/DclUYMPXcAEOCUF.jpg


Progress can only be made if we assume such wacky scenarios.
Progress?  ???

Yes, progress towards creating the basic orbits seen in the heliocentric system with the ideas of Newton, Kepler, and Copernicus. Because so far, over hundreds of years and the efforts of the greatest minds on earth, it has not been done.
Title: Re: Round Earth Celestial Mechanics Cannot Predict the Solar System
Post by: markjo on July 26, 2018, 11:19:04 PM
Yes, progress towards creating the basic orbits seen in the heliocentric system with the ideas of Newton, Kepler, and Copernicus. Because so far, over hundreds of years and the efforts of the greatest minds on earth, it has not been done.
Of course it's been done.  How do you think that deep space probes are able to use gravity assists tool around the solar system?

BTW, what kind of progress have the greatest minds of the FE community made in modeling the FE solar system?
Title: Re: Round Earth Celestial Mechanics Cannot Predict the Solar System
Post by: Tom Bishop on July 27, 2018, 12:00:02 AM
Yes, progress towards creating the basic orbits seen in the heliocentric system with the ideas of Newton, Kepler, and Copernicus. Because so far, over hundreds of years and the efforts of the greatest minds on earth, it has not been done.
Of course it's been done.  How do you think that deep space probes are able to use gravity assists tool around the solar system?

BTW, what kind of progress have the greatest minds of the FE community made in modeling the FE solar system?

Now you are appealing to NASA and trying to move the conversation to look at something else? Have you gave up already, Markjo?

It is apparent that you are implicitly agreeing that your position is incredibly weak, are backed into a corner, and trying to do anything possible to avoid facing reality.

The Heliocentric System Cannot Be Predicted

The Heliocentric System Cannot Be Created

The Heliocentric System Cannot Even Exist
Title: Re: Round Earth Celestial Mechanics Cannot Predict the Solar System
Post by: BillO on July 27, 2018, 12:18:03 AM
Regarding the Pop-Science Magazine Solar System Sim written in Applesoft BASIC
Pop-Science?  Maybe, but I never had a subscription to them.  Just the ones I mentioned.

Quote
I took a look at this one. It appears to merely be creating 2-body orbits around a static sun. It does not appear to be a three-body or n-body problem simulator.
Bang, there it goes.  Either you are being intentionally intellectually dishonest or you don't know how to read this code.  Maybe get someone else to have a look at it if you have trouble with simple linearized math.  Look at the subroutine beginning at 440 and returning at 590.  You can see how every object entered into the system is taken into account and the force between each is calculated using Newton's law of Universal gravity.  Every single one.  I get the feeling that you think none of your FE followers will be able or inclined to read the code or do the simple math so that you can say whatever you wish.

Quote
I placed the code from the last page into an online Applesoft BASIC Emulator here: http://www.calormen.com/jsbasic/

Using the settings Bill gave, I made two Earths, rotating in opposite directions, in the same orbit.
Why?  Why did you not do the 4 body situation?

Quote
Here were the results:
Okay, so that's exactly what I would expect.    How long did you let it run?.  I wrote no collision routine, so bodies will just pass right through each other when in precisely the same orbit. They are basically dimensionless.  This situation may not create an instability for quite a while, if ever.  If you want to do that put one 'earth' on a slightly different orbit - for instance use the average distance and the average speed.  That way the small offset of gravitation will create an instability.  It should begin to visibly blow up in about 4 or 5 orbits.

Try this:
Code: [Select]
Sun-Earth1-Earth2 (3 Objects)

Sun
M1 = 1.9885E30
X1 = 0
Y1 = 0
VX1 = 0
VY1 = 0

Earth 1
M2 = 5.9724E24
X2 = 1.5210E11
Y2 = 0
VX2 = 0
VY2 = 29290

Earth 2
M3 = 5.9724E24
X3 = -1.496E11
Y3 = 0
VX3 = 0
VY3 = 29780

Scale = 2E11
Time Step = 50000
Title: Re: Round Earth Celestial Mechanics Cannot Predict the Solar System
Post by: BillO on July 27, 2018, 12:23:09 AM
Poliastro says that the image represents numerical methods for the Restricted Three Body Problem. Are you to assert that you know better than an astrodynamics software developer?
No, I certainly would not do that.  However, I can't see the whole thing here.  I have no idea of the work he is talking about and what the goals were.  Do you have a link handy to the original work?
Title: Re: Round Earth Celestial Mechanics Cannot Predict the Solar System
Post by: markjo on July 27, 2018, 12:58:47 AM
Yes, progress towards creating the basic orbits seen in the heliocentric system with the ideas of Newton, Kepler, and Copernicus. Because so far, over hundreds of years and the efforts of the greatest minds on earth, it has not been done.
Of course it's been done.  How do you think that deep space probes are able to use gravity assists tool around the solar system?

BTW, what kind of progress have the greatest minds of the FE community made in modeling the FE solar system?

Now you are appealing to NASA and trying to move the conversation to look at something else? Have you gave up already, Markjo?

It is apparent that you are implicitly agreeing that your position is incredibly weak, are backed into a corner, and trying to do anything possible to avoid facing reality.
No, I'm just agreeing that you are incredibly stubborn about accepting any evidence that proves that you're wrong.

The Heliocentric System Cannot Be Predicted

The Heliocentric System Cannot Be Created

The Heliocentric System Cannot Even Exist
Do you think that space probes could use gravitational assists to tool around the solar system if the relevant space agencies didn't have a very accurate models of the solar system so that they could predict where the relevant celestial bodies would be at a given time?

I'm sorry, how well did you say that your FE solar system model is coming along?
Title: Re: Round Earth Celestial Mechanics Cannot Predict the Solar System
Post by: totallackey on July 27, 2018, 10:51:33 AM
This is really not worth replying to ... but  I never said it was going to be anything but a demonstration that purely numerical methods can produce stable systems of multiple bodies.  It does.  q.e.d.
If your goal is producing utter fiction, then congratulations!

Quite a high bar you set for yourself!

needless and mindless words snipped for brevity becasue this does not even have a moving sun included and actually demonstrates ZERO use of Newton's formulas.
Ahhh, that's odd as there at least 2 of them in there.  My guess is you don't have much in the way of math or computer skills.  A bit of blustering then?  Would you care to demonstrate how the sun's motion would be significant to the orbital dynamics of the solar system?


Yep, absolute and verifiable blustering...
[/quote]
LMMFAO!!!

"Would you care to demonstrate how the sun's motion would be significant to the orbital dynamics of the solar system?"

You actually asked that question as if it was legitimate?

If you need that explained to you...omg...
Title: Re: Round Earth Celestial Mechanics Cannot Predict the Solar System
Post by: BillO on July 27, 2018, 01:43:40 PM
LMMFAO!!!

"Would you care to demonstrate how the sun's motion would be significant to the orbital dynamics of the solar system?"

You actually asked that question as if it was legitimate?

If you need that explained to you...omg...
Oh, yes.  I just remembered the problems you FE'ers have with understanding uniform motion, inertial frames of reference and Newton's first law of motion.  Pity.

So my guess is you won't demonstrate how the sun's motion would be significant to the orbital dynamics of the solar system, right?  Good choice as you can't, but I doubt you will ever understand why it is of no significance.
Title: Re: Round Earth Celestial Mechanics Cannot Predict the Solar System
Post by: AATW on July 27, 2018, 01:49:05 PM
The Heliocentric System Cannot Even Exist
That's an interesting claim. Care to back that up?
Title: Re: Round Earth Celestial Mechanics Cannot Predict the Solar System
Post by: Tom Bishop on July 27, 2018, 07:44:00 PM
The Heliocentric System Cannot Even Exist
That's an interesting claim. Care to back that up?

Look at the failure of the Three Body Problem efforts of astronomers and mathematicians to simulate the Sun-Earth-Moon system. The available solutions do not look like heliocentric systems at all. They are unable to create anything that looks like a Sun-Earth-Moon system. They just can't create it.

Others have pointed out the impossibility of Heliocentric motions in other ways:

https://malagabay.wordpress.com/2012/11/03/inventions-and-deceptions-hill-sphere/

Quote
The impossibility of the Moon’s orbit [according to Newtonian Gravity] was highlighted back in 1979 by Walter C Wright in his book “Gravity is a Push”:

“My conclusion is that the sun and the earth have no pull factor on the moon”

(https://malagabay.files.wordpress.com/2012/11/walter-c-wright-1979-gravity-is-a-push.gif)

Ralph Rene similarly concluded that Newtonian Gravity was “impossible” in his 1998 book “The Last Skeptic of Science”:

(https://malagabay.files.wordpress.com/2012/11/ralph-rene-1998-impossible_moon_orbit_1.gif?w=640&h=431)

(https://malagabay.files.wordpress.com/2012/11/ralph-rene-1998-impossible_moon_orbit_2.gif?w=640&h=475)

(https://malagabay.files.wordpress.com/2012/11/ralph-rene-1998-impossible_moon_orbit_3.gif?w=640&h=396)

Miles Mathis reviewed the work of Ralph Rene and concluded: “Mr. René is correct”.

    I think we must see that he is correct. We have a real problem here, and the standard-model answer is just one more pathetic dodge.
    … … … …
    Mr. René is correct. Even if the Moon and Earth could theoretically inhabit the same orbit, the Moon would have to be pulled lower at New Moon by the Sun. If it were, it could not re-establish its original distance. Once its distance from the Earth had been increased, its pull from the Earth would be lessened, due to the same equation. It would escape very quickly.

    Another Hole in Celestial Mechanics
    Miles Mathis
    http://milesmathis.com/cm2.html


It is apparent why, in the Restricted Three Body Problem simulations discussed earlier, the moon needs to be of zero mass in those models.
Title: Re: Round Earth Celestial Mechanics Cannot Predict the Solar System
Post by: Curious Squirrel on July 27, 2018, 08:13:31 PM
{snipped for brevity}
I would point out that at best all this shows is that Newtonian gravitation doesn't work to explain celestial motion. It does NOT however prove that "The Heliocentric System Cannot Even Exist" as you so boldly claim. Can you please present your evidence supporting the statement that a heliocentric solar system cannot exist, and not simply fringe sources proclaiming Newtonian mechanics are incorrect?
Title: Re: Round Earth Celestial Mechanics Cannot Predict the Solar System
Post by: Rama Set on July 27, 2018, 08:55:26 PM
Further to that, numerical solutions of the three-body problem are used regularly, and the heliocentric solar system can be modeled very accurately as an aggregate of two-body systems.  But Tom never acknowledges this.
Title: Re: Round Earth Celestial Mechanics Cannot Predict the Solar System
Post by: Tom Bishop on July 27, 2018, 09:37:30 PM
{snipped for brevity}
I would point out that at best all this shows is that Newtonian gravitation doesn't work to explain celestial motion.

I am glad to see agreement that the Heliocentric System cannot be created or exist under current theory. Celestial Mechanics is in the stone age, without even a working theory.

What else do you have besides Newton? The Solar System is based on Newtonian physics. Einstein's General Relativity isn't any different, and only makes very slight adjustments to Newton's gravity.

From http://milesmathis.com/cm2.html

Quote
Unfortunately, it is known by everyone, including them, that Einstein never claimed to have overthrown Newton. Einstein only claimed to have extended Newton's equations by importing time (and therefore mass) differentials into his field. That is, GR is Newton plus SR. And this means that modern physicists have nowhere to hide. Their new field equations do not solve this problem, since motions are still determined by mass interactions. No amount of new math can hide the fact that the Moon is out of balance at New Moon and Full Moon in the three-body problem, no matter whether you try to solve it with Newton's math, Laplace's math, or Einstein's math.

Einstein's gravity predicts essentially the same thing as Newton's gravity. I don't see where you are going with your assertion.

Quote
It does NOT however prove that "The Heliocentric System Cannot Even Exist" as you so boldly claim. Can you please present your evidence supporting the statement that a heliocentric solar system cannot exist, and not simply fringe sources proclaiming Newtonian mechanics are incorrect?

Ralph Rene's example with the moon is pretty clear. It's using Newton's own equation.

His reputation as a "crank" is because he says that Newton was wrong. Since you are apparently agreeing with that assessment, that makes you the "crank" as well.

If no one can answer these challenges, then we must conclude that under the current model the Heliocentric System cannot exist.
Title: Re: Round Earth Celestial Mechanics Cannot Predict the Solar System
Post by: BillO on July 27, 2018, 10:01:02 PM
I would point out that at best all this shows is that Newtonian gravitation doesn't work to explain celestial motion.
No it certainly does not, not in it's or René's wildest dreams.



Others have pointed out the impossibility of Heliocentric motions in other ways:
Ralph René is a well known nutbag, moon mission denier, conspiracy theorist and school drop-out.  https://en.wikipedia.org/wiki/Ralph_Ren%C3%A9

And for some reason you take his word over folks like Newton, Einstein, Neil Degrasse Tyson.  Of course you do!

Yeah, I also agree with René’s calculation of the forces on the moon from the earth and the sun.  He got that grade school math right, however his interpretation was hilarious.  What he forgets/misses/bumbles, and apparently you missed it too a, is that the moon is also orbiting the sun as it orbits the earth which completely satisfies the force on it from the sun.

Tom, you’re reading the wrong stuff, putting your lot in with the wrong people and you don’t have the math and/or physics skills to see how they are leading you astray, and you are also doing it willfully.  Why?

You made a claim about the simulator I provided:
Quote
I took a look at this one. It appears to merely be creating 2-body orbits around a static sun. It does not appear to be a three-body or n-body problem simulator.
I respectfully request that either you justify this baseless claim by reference to my code, or that you retract it here.
Title: Re: Round Earth Celestial Mechanics Cannot Predict the Solar System
Post by: Tom Bishop on July 27, 2018, 11:22:44 PM
And for some reason you take his word over folks like Newton, Einstein, Neil Degrasse Tyson.  Of course you do!

Newton? Newton invokes divine intervention to explain his solar system. He said it himself! (https://wiki.tfes.org/Celestial_Mechanics_Cannot_Predict_The_Solar_System#Official_Explanation:_Divine_Intervention)

Newton cannot explain the n-body issues with his model and tells us that god is keeping everything together.

Einstein's gravity is based on Newton's gravity and provides no further answers.

Neil Degrasse Tyson appears to be more of a TV celebrity than anything. I can't find that he solved any of the n-body issues.

Quote
Yeah, I also agree with René’s calculation of the forces on the moon from the earth and the sun.  He got that grade school math right, however his interpretation was hilarious.  What he forgets/misses/bumbles, and apparently you missed it too a, is that the moon is also orbiting the sun as it orbits the earth which completely satisfies the force on it from the sun.

Then, at the point in question, the moon will leave the earth and continue to orbit the sun, since the sun is applying the more powerful force at that time.

How does the moon get back to the earth?

Your "point" doesn't tell us anything.

Quote
Tom, you’re reading the wrong stuff, putting your lot in with the wrong people and you don’t have the math and/or physics skills to see how they are leading you astray, and you are also doing it willfully.  Why?

I am only interested in truth. As far as I can see, Bill, you are the main person here attempting to lead people astray.

The example is fairly clear to all. Your point about the moon orbiting the sun is rather stupid and only tells us that the moon would continue to orbit the sun and leave the earth behind.

Quote
You made a claim about the simulator I provided:
Quote
I took a look at this one. It appears to merely be creating 2-body orbits around a static sun. It does not appear to be a three-body or n-body problem simulator.
I respectfully request that either you justify this baseless claim by reference to my code, or that you retract it here.

The two earths in the simulation just passed right through each other without a change in momentum or orbital path. I don't know what more there is to show about about the matter. It is clearly not a three body or n-body simulator.

I ran the thing you followed up with yesterday. I got an error code after a few rotations, "Illegal quantity in line 650". All I can see from that is that you fed it bad two-body orbit values in your new settings and the two-body orbit either fell apart, grew too big for the screen, or there was some other underlying issue with the code. I have not seen anything that demonstrates that the planets are affected by each other. They always pass right through each other unperturbed.

No, I'm just agreeing that you are incredibly stubborn about accepting any evidence that proves that you're wrong.

Markjo, read the page I gave earlier: http://www.scholarpedia.org/article/Three_body_problem#The_astronomer.27s_three-body_problem:_ii.29_a_caricature_of_the_lunar_problem

If the Sun-Earth-Moon system could be described and understood with the Three Body Problem, we would have read that, rather than a demonstration of Hill's massless-moon that bounces around chaotically, makes mid-orbit u-turns, and collides with the earth. It does not say that the Three Body Solutions have created the Sun-Earth-Moon system, and none of the other Three Body Problem articles or books do either.

They have no idea how to handle the motions of three bodies under Newtonian physics in the heliocentric system. They cannot simulate or explain it. It cannot be done. Where are the examples of the Heliocentric Sun-Earth-Moon system modeled with the Three Body Solutions?

Instead of this simple thing you can point us to, you mentally cross out the sections which admit that three or more bodies is very problematic in everything you read, stubbornly refuse to face reality, and assume that "someone must understand it".

Show it. Show that the Three Body Solutions can simulate the Heliocentric Sun-Earth-Moon System.
Title: Re: Round Earth Celestial Mechanics Cannot Predict the Solar System
Post by: BillO on July 28, 2018, 12:20:01 AM
Newton? Newton invokes divine intervention to explain his solar system. He said it himself!

Newton cannot explain the n-body issues with his model and tells us that god is keeping everything together.
Perhaps a 17th century take on the failure to find a purely analytic solution.  I've not spoken with Newton to get his take on his comments.  Needless to say the computers of the time were not up to the task of providing a purely numerical proof.

Quote
Einstein's gravity is based on Newton's gravity and provides no further answers.
No it's not.  Where did you pull that rabbit from?  Yes it had to agree with Newton;s theory - that's the way scientists do things - they improve on older theories not toss them out and not replace them like FE'ers.  GR is not even linear and is based on entirely different concepts.

Quote
Neil Degrasse Tyson appears to be more of a TV celebrity than anything. I can't find that he solved any of the n-body issues.
Research his story and quit making assumptions- isn't that what a zetectic does?

Quote
Then, at the point in question, the moon will leave the earth and continue to orbit the sun, since the sun is applying the more powerful force at that time.

How does the moon get back to the earth?

Your "point" doesn't tell us anything.
It does, it's just that you don't understand.  It is the center of mass of the earth moon system that orbits the sun.  The moon isn't borrowed by the sun and then handed back to the earth.  What kind of thinking is that?  Rene thinking?  They both orbit the sun and also orbit around their common center of mass.  They can't move away from each other further than they are.

Quote
I am only interested in truth. As far as I can see, Bill, you are the main person here attempting to lead people astray.
Astray?  You make claims about things Tom, but you don't do the hard thinking yourself.  You quote things from google searches that 'look' like what you want to get across and you can't see they don't support you ideas.

Quote
The example is fairly clear to all. Your point about the moon orbiting the sun is rather stupid and only tells us that the moon would continue to orbit the sun and leave the earth behind.
How could it?  The Earth is also orbiting the sun at pretty much the same place as the moon, so they are forced to orbit each other as well.  If you think I'm a quack, that this up with a physicist at one of your local universities.

Quote
You made a claim about the simulator I provided:
Quote
I took a look at this one. It appears to merely be creating 2-body orbits around a static sun. It does not appear to be a three-body or n-body problem simulator.
I respectfully request that either you justify this baseless claim by reference to my code, or that you retract it here.

Quote
The two earths in the simulation just passed right through each other without a change in momentum or orbital path. I don't know what more there is to show about about the matter.
There is an obvious change in momentum in that video you provided and in my own simulator here at home.  Watch it again carefully.  You gave them exactly the same orbit.  They would necessarily meet exactly at the same point at the same time.  Without a collision (which I did not program) I would not expect any change to their orbits other than an acceleration when they get close and a slowing down as they move apart - which is witnessed.  Do you have something to quantify anything to the contrary?

Quote
I ran the thing you followed up with yesterday. I got an error code after a few rotations, "Illegal quantity in line 650". All I can see from that is that you fed it bad two-body orbit values in your new settings and the two-body orbit either fell apart, grew too big for the screen, or there was some other underlying issue with the code. I have not seen anything that demonstrates that the planets are affected by each other. They always pass right through each other unperturbed.
I gave one earth a circular orbit.  If you doubt it's validity, run them separately.

The error occurs when the program attempts to plot a point off the screen, so yeah, it is technically a bug, but it's not in the math routine.  Look at line 650.  I could trap that for you if you're interested.  Now that I re-discovered the code I intend to fancy it up a bit and make it more user friendly.  It is wanted by some others on vintage computing forums, so it would be no big deal.

Look Tom, either you prove your point that this program does not address the n-body (actually the program is limited to 10 bodies) by showing the error in my code, or have someone else do that for you, or retract your comments.  It's not right to cast aspersions one someone else like this.  I provided you with a numerical simulation and all you do is void all over it without any compunction or attempt to validate your 'reasons' with any rigor at all.  If you understood this stuff, it should be a simple matter for you.  I expect more from someone that professes to educate the world on new ideas.
Title: Re: Round Earth Celestial Mechanics Cannot Predict the Solar System
Post by: Curious Squirrel on July 28, 2018, 07:29:52 AM
{snipped for brevity}
I would point out that at best all this shows is that Newtonian gravitation doesn't work to explain celestial motion.

I am glad to see agreement that the Heliocentric System cannot be created or exist under current theory. Celestial Mechanics is in the stone age, without even a working theory.
There was no agreement with your statement. I was pointing out that you have not presented a case that the heliocentric solar system cannot work. Once again at best all that shows is that Newton gravity doesn't explain it. Please stop taking snippets of what I say and pretending it agrees with you.
Title: Re: Round Earth Celestial Mechanics Cannot Predict the Solar System
Post by: Rama Set on July 28, 2018, 01:19:48 PM
GL guys. Tom will not concede a point like this. In the Full Moon thread, icansciencethat provided a model with full mathematical rigour, exactly as Tom requested. Tom said that if said model were provided he would concede the point, but he has not. He isn’t interested in the truth, he is interested in arguing, so if you are game for that, then go to. I have been observing him in his natural habitat for almost ten years and it has always been so.

It doesn’t matter to him how many examples of heliocentric models he is shown, or that an analytic solution to the n-body problem is not the end of the heliocentric model. He literally could care less. He is honing something to write in the wiki, that is all.
Title: Re: Round Earth Celestial Mechanics Cannot Predict the Solar System
Post by: BillO on July 28, 2018, 02:12:10 PM
Yes, I think you are absolutely right.  I'm pretty well done on this one and more than likely will not put much more effort into it.

Has anyone else noticed a real lack of intellectual integrity amongst some of the FE'ers?
Title: Re: Round Earth Celestial Mechanics Cannot Predict the Solar System
Post by: Rama Set on July 28, 2018, 03:47:28 PM
Has anyone else noticed a real lack of intellectual integrity amongst some of the FE'ers?

Probably a topic for another thread.
Title: Re: Round Earth Celestial Mechanics Cannot Predict the Solar System
Post by: Tom Bishop on July 28, 2018, 07:52:53 PM
Newton? Newton invokes divine intervention to explain his solar system. He said it himself!

Newton cannot explain the n-body issues with his model and tells us that god is keeping everything together.
Perhaps a 17th century take on the failure to find a purely analytic solution.  I've not spoken with Newton to get his take on his comments.  Needless to say the computers of the time were not up to the task of providing a purely numerical proof.

I have shown the numerical three body methods as applied to the Sun-Earth-Moon system. We saw what the moon did. Where is this stable Three Body solution?

Quote
Quote
Neil Degrasse Tyson appears to be more of a TV celebrity than anything. I can't find that he solved any of the n-body issues.
Research his story and quit making assumptions- isn't that what a zetectic does?

What does his story have to do with this topic? Did he solve the three body problems? No. He did not. Get out of here with that.

Quote
Quote
Then, at the point in question, the moon will leave the earth and continue to orbit the sun, since the sun is applying the more powerful force at that time.

How does the moon get back to the earth?

Your "point" doesn't tell us anything.
It does, it's just that you don't understand.  It is the center of mass of the earth moon system that orbits the sun.  The moon isn't borrowed by the sun and then handed back to the earth.  What kind of thinking is that?  Rene thinking?  They both orbit the sun and also orbit around their common center of mass.  They can't move away from each other further than they are.

I fully understand. At the new moon the sun is applying more force on the moon than the earth. Over twice as much, in fact.

How does the moon get back behind the earth again, to where the earth is between the sun and the moon? Explain.

You said it yourself: "They can't move away from each other further than they are." Eat it. Explain it. How does the moon go around the earth to where the earth can be between it and the sun again?

Quote
Look Tom, either you prove your point that this program does not address the n-body (actually the program is limited to 10 bodies) by showing the error in my code, or have someone else do that for you, or retract your comments.

I've already shown that the bodies pass through each other unaffected. You have shown nothing contrary to this. I'm not going to decipher un-commented code from an ancient operating system. You need to demonstrate your case, and so far you have not done so.

GL guys. Tom will not concede a point like this. In the Full Moon thread, icansciencethat provided a model with full mathematical rigour, exactly as Tom requested. Tom said that if said model were provided he would concede the point, but he has not.

I said that if a Round Earth model were provided I would concede the point. Everyone saw the silly model that was presented. The last I checked, the Round Earth model doesn't have the bodies in space projected onto a screen close above the observer's heads. The model is totally invalid as a demonstration, just as Bill's magazine model is totally invalid.
Title: Re: Round Earth Celestial Mechanics Cannot Predict the Solar System
Post by: BillO on July 28, 2018, 08:19:03 PM

I have shown the numerical three body methods as applied to the Sun-Earth-Moon system.
You didn't


Quote
I fully understand. At the new moon the sun is applying more force on the moon than the earth. Over twice as much, in fact.

How does the moon get back behind the earth again, to where the earth is between the sun and the moon? Explain.
I did.  You didn't understand.  The moon is orbiting the Sun.  That's it- that's all.  If you take the earth out of it, the moon will happily orbit the sun at ~93M miles with a period of ~365 days.  Same can be said for the earth without the moon.  They happen to be close to each other so they orbit each other too.  It's butt simple Tom.  Children understand this with ease.

Quote
I've already shown that the bodies pass through each other unaffected.
I explained that.  That proves nothing as you did not frame a reason why it should create an instability.  You just did something you did not understand and said "See!"

Quote
You have shown nothing contrary to this.
I gave you another version of the model that showed the interaction between two earths.  Like everything else, you dismissed it even though you saw the results.  What more can I do?  What will you not dismiss?

Quote
I'm not going to decipher un-commented code from an ancient operating system. You need to demonstrate your case, and so far you have not done so.
The code is simple and does not require documentation to see what it's doing.  The language is BASIC.  I's still around and in use today.  You are not going to do it because you can't.

You have the program, If you doubt the last model I gave you, as I already said, you can run the two earths individually to see that they are both in stable orbits and then run them together to show they interact.

Tom, I'm out of this one.  You have the tools - do the zetetic thing.  I'll give you the last word...
Title: Re: Round Earth Celestial Mechanics Cannot Predict the Solar System
Post by: Tom Bishop on July 28, 2018, 10:19:36 PM
If Bill's only response to Rene's challenge (https://forum.tfes.org/index.php?topic=10175.msg160845#msg160845) is "the moon is orbiting the earth" then I suppose there is nothing left to discuss about the matter. Everyone can see it for what it is.

According to Bill, because the moon is orbiting the earth, it doesn't matter what kind of external forces are applied upon the moon. An external force can be twice as powerful, ten times as powerful, whatever, and the moon will just defy that force and move in an opposite direction from it as it continues to rotate around the earth. The moon is orbiting the earth and that's that!

The magic in this response is apparent and obvious.

Of the sun and moon, Bill says it himself: "They can't move away from each other further than they are."

A little honesty peaking trough! Bill admits that the Sun-Earth-Moon system, as it is described, is impossible. It is impossible for the moon to defy the stronger pull of the sun and get behind the earth again, so that the earth is between the sun and the moon.

Lets read it again: "They can't move away from each other further than they are."

Of course not. The moon isn't going to defy a stronger force in favor of a weaker force. In the followup Bill now seems to be spiraling out of control with excuses of "the moon is closer to the earth" or some such thing. As if physical laws were based on sentences that a "child could understand". Very amusing. These are weak arguments, and I agree with Bill's assessment that the best course of action is for you guys to just stop talking about it.
Title: Re: Round Earth Celestial Mechanics Cannot Predict the Solar System
Post by: inquisitive on July 28, 2018, 10:21:35 PM
Ending this gives Bill time to measure the angle of the sun at different times and locations to determine the shape of the earth.
Title: Re: Round Earth Celestial Mechanics Cannot Predict the Solar System
Post by: garygreen on July 29, 2018, 12:21:26 AM
Rene's challenge (https://forum.tfes.org/index.php?topic=10175.msg160845#msg160845).  Very amusing.

i couldn't agree more.  rene is conflating two different frames of reference (geocentric and heliocentric).

from the geocentric point of view, it doesn't matter that the relative forces are different.  acceleration due to gravity does not depend on the mass of the object being accelerated (eg everything falls at the same rate).  since your new thing is to ask people to do math for you:

(https://i.imgur.com/4DE6y3b.png)

since the earth-sun and moon-sun distances are basically identical, then the earth and moon are accelerated by the sun at the same rate and in the same direction.

from the heliocentric point of view the story is more interesting: the moon orbits the sun, and its orbit is perturbed by the earth.

https://www.scribd.com/document/384911423/The-Sun-the-Moon-and-Convexity
https://www.scribd.com/document/384912183/Why-the-Moon-s-Orbit-is-Convex

 
Title: Re: Round Earth Celestial Mechanics Cannot Predict the Solar System
Post by: Tom Bishop on July 29, 2018, 01:10:40 AM
Rene's challenge (https://forum.tfes.org/index.php?topic=10175.msg160845#msg160845).  Very amusing.

i couldn't agree more.  rene is conflating two different frames of reference (geocentric and heliocentric).

from the geocentric point of view, it doesn't matter that the relative forces are different.  acceleration due to gravity does not depend on the mass of the object being accelerated (eg everything falls at the same rate).  since your new thing is to ask people to do math for you:

(https://i.imgur.com/4DE6y3b.png)

since the earth-sun and moon-sun distances are basically identical, then the earth and moon are accelerated by the sun at the same rate and in the same direction.

from the heliocentric point of view the story is more interesting: the moon orbits the sun, and its orbit is perturbed by the earth.

https://www.scribd.com/document/384911423/The-Sun-the-Moon-and-Convexity
https://www.scribd.com/document/384912183/Why-the-Moon-s-Orbit-is-Convex

Your argument seem to be that "the moon and earth orbit so closely together so it doesn't matter".

And since they are "close together," you are crossing out various equations for gravity as applied to the earth and the moon?  ???

Is that a right assessment?

The moon's orbital path has a diameter of 768,000 km. That is almost one million miles. The moon can defy the force of the sun over a course of almost one million miles?

I don't see that either of those pages addresses this question.
Title: Re: Round Earth Celestial Mechanics Cannot Predict the Solar System
Post by: garygreen on July 29, 2018, 01:38:08 AM
Your argument seem to be that "the moon and earth orbit so closely together so it doesn't matter".

And since they are "close together," you are crossing out various equations for gravity as applied to the earth and the moon?  ???

Is that a right assessment?

The moon's orbital path has a diameter of 768,000 km. That is almost one million miles. The moon can defy the force of the sun over a course of almost one million miles?

yep, that's an accurate assessment.  the earth is ~149 million km from the sun.  adding 0.5% hardly changes the force.

I don't see that either of those pages addresses this question.

they describe the moon's orbit from a heliocentric point of view.  you've been asking: but wait isn't the sun acting on the moon, too?  answer: it is.  this is already well-understood.  from a heliocentric frame, the moon orbits the sun and is perturbed by the earth.

and in a geocentric frame, the influence of the sun doesn't matter because it curves the paths of the earth and moon in the same way.  the earth and moon are in the same reference frame centered on the system's barycenter.
Title: Re: Round Earth Celestial Mechanics Cannot Predict the Solar System
Post by: BillO on July 29, 2018, 03:13:29 PM
Rene's challenge (https://forum.tfes.org/index.php?topic=10175.msg160845#msg160845).  Very amusing.

i couldn't agree more.  rene is conflating two different frames of reference (geocentric and heliocentric).

from the geocentric point of view, it doesn't matter that the relative forces are different.  acceleration due to gravity does not depend on the mass of the object being accelerated (eg everything falls at the same rate).  since your new thing is to ask people to do math for you:

(https://i.imgur.com/4DE6y3b.png)

since the earth-sun and moon-sun distances are basically identical, then the earth and moon are accelerated by the sun at the same rate and in the same direction.

from the heliocentric point of view the story is more interesting: the moon orbits the sun, and its orbit is perturbed by the earth.

https://www.scribd.com/document/384911423/The-Sun-the-Moon-and-Convexity
https://www.scribd.com/document/384912183/Why-the-Moon-s-Orbit-is-Convex
Nicely and elegantly done! 

However, I hope you don't expect this to have any effect on the person you were responding to.  The reason I gave up discussing this with him was because he has no ability to understand it.  You can see his mathematics acumen from his response to you:
The moon's orbital path has a diameter of 768,000 km. That is almost one million miles.
::)

I doubt many on this board will understand, including the person you responded to (obviously) why you were able to make the cancellations you did.  The skills are just not there.

While you have proven your point with simple mathematics, it will be dismissed.

Kudos nonetheless.

Title: Re: Round Earth Celestial Mechanics Cannot Predict the Solar System
Post by: Tom Bishop on July 29, 2018, 05:34:31 PM
Great one Bill! Your main ammunition is an obvious typo, rather than any real or fundamental error. You even put it into your sig. Looks like a pretty desperate tactic.

We can all see how weak the argument is: Declaring that the moon is close to the earth and then crossing out the physics. High level stuff.
Title: Re: Round Earth Celestial Mechanics Cannot Predict the Solar System
Post by: Ofcourseitsnotflat on July 29, 2018, 05:35:33 PM
Great one Bill! Your main ammunition is an obvious typo, rather than any real or fundamental error. You even put it into your sig. Looks like a pretty desperate tactic

If it's a typo, what did you actually mean to say instead?
Title: Re: Round Earth Celestial Mechanics Cannot Predict the Solar System
Post by: BillO on July 29, 2018, 05:37:22 PM
My main ammo? "I doubt many on this board will understand, including the person you responded to (obviously) why you were able to make the cancellations you did."


Title: Re: Round Earth Celestial Mechanics Cannot Predict the Solar System
Post by: markjo on July 29, 2018, 05:45:46 PM
Tom, the moon's orbit around the sun looks something like this:
(https://qph.fs.quoracdn.net/main-qimg-c8ca8c109ecb9dacdaa8295c535d2bab)
Title: Re: Round Earth Celestial Mechanics Cannot Predict the Solar System
Post by: totallackey on July 30, 2018, 12:39:45 PM
They are basically dimensionless... 
Funny!

In response to Tome, you patently admit your whole routine is total shinola!

Dimensionless planets...as fictional as the heliocentric solar system...
LMMFAO!!!

"Would you care to demonstrate how the sun's motion would be significant to the orbital dynamics of the solar system?"

You actually asked that question as if it was legitimate?

If you need that explained to you...omg...
Oh, yes.  I just remembered the problems you FE'ers have with understanding uniform motion, inertial frames of reference and Newton's first law of motion.  Pity.

So my guess is you won't demonstrate how the sun's motion would be significant to the orbital dynamics of the solar system, right?  Good choice as you can't, but I doubt you will ever understand why it is of no significance.
And neither do you demonstrate as to how the Sun's motion is insignificant to the orbital dynamics of the planets around it while traipses around the fictional Milky Way!!!

The onus is on you there Copernicus-lover...

Make me a CGI model of the heliocentric solar system, utilizing Newton, Kepler, and Einstein...

Not excuses!
Title: Re: Round Earth Celestial Mechanics Cannot Predict the Solar System
Post by: BillO on July 30, 2018, 01:59:09 PM
They are basically dimensionless... 
Funny!

In response to Tome, you patently admit your whole routine is total shinola!

....

And neither do you demonstrate as to how the Sun's motion is insignificant to the orbital dynamics of the planets around it while traipses around the fictional Milky Way!!!
You have not a clue what you are talking about.

To your first ridiculous point: Newton's theory does not depend on the size of the objects, just their mass and the radius between them.  Even a complete idiot can see this:


f= Gm1m2/r2


That's the theory - where in there are the dimensions required?  They are not.  They have no bearing on the situation.

To your 2nd ridiculous point: The solar system is an inertial frame of reference WRT to the sun.  Uniform motion of the sun will not and can not affect the solar system as every body in it is also moving along with the sun at exactly the same speed in the direction of the sun's motion.  That's the explanation, there isn't any more one can say about it.  You don't understand that because you don't have the first clue about the mechanics of motion.  Why do you insist on displaying your lack of knowledge on this?

Quote
The onus is on you there Copernicus-lover...

Make me a CGI model of the heliocentric solar system, utilizing Newton, Kepler, and Einstein...

Not excuses!
Ahhh, I provided the model and it uses Newton, the onus is on you to prove it does not work.  I know you can't and I hate discussing things with people that proceed proudly from a position of profound ignorance, so I'll give you the last word ... our discussion is done until you provide some tangible proof.
Title: Re: Round Earth Celestial Mechanics Cannot Predict the Solar System
Post by: totallackey on July 30, 2018, 02:41:27 PM
They are basically dimensionless... 
Funny!

In response to Tome, you patently admit your whole routine is total shinola!

....

And neither do you demonstrate as to how the Sun's motion is insignificant to the orbital dynamics of the planets around it while traipses around the fictional Milky Way!!!
You have not a clue what you are talking about.

To your first ridiculous point: Newton's theory does not depend on the size of the objects, just their mass and the radius between them.  Even a complete idiot can see this:


f= Gm1m2/r2


That's the theory - where in there are the dimensions required?  They are not.  They have no bearing on the situation.

To your 2nd ridiculous point: The solar system is an inertial frame of reference WRT to the sun.  Uniform motion of the sun will not and can not affect the solar system as every body in it is also moving along with the sun at exactly the same speed in the direction of the sun's motion.  That's the explanation, there isn't any more one can say about it.  You don't understand that because you don't have the first clue about the mechanics of motion.  Why do you insist on displaying your lack of knowledge on this?

Quote
The onus is on you there Copernicus-lover...

Make me a CGI model of the heliocentric solar system, utilizing Newton, Kepler, and Einstein...

Not excuses!
Ahhh, I provided the model and it uses Newton, the onus is on you to prove it does not work.  I know you can't and I hate discussing things with people that proceed proudly from a position of profound ignorance, so I'll give you the last word ... our discussion is done until you provide some tangible proof.
So in your fictional construct, things that have mass must exist in a world without dimensions...

And you call my points silly?

WTF is the matter with you!?!?!
Title: Re: Round Earth Celestial Mechanics Cannot Predict the Solar System
Post by: AATW on July 30, 2018, 02:56:53 PM
Great one Bill! Your main ammunition is an obvious typo, rather than any real or fundamental error. You even put it into your sig. Looks like a pretty desperate tactic.
It's certainly no match for your main ammunition of not being able to understand anything and then declaring yourself right.
Quote
We can all see how weak the argument is: Declaring that the moon is close to the earth and then crossing out the physics. High level stuff.
Yes, look at all the FE people queuing up to agree with you.
It's not even an argument, he's literally done the maths for you. Rene is getting himself in a muddle because he hasn't factored in the pull of the sun on the earth.
Title: Re: Round Earth Celestial Mechanics Cannot Predict the Solar System
Post by: Tom Bishop on July 30, 2018, 09:08:55 PM
It's not even an argument, he's literally done the maths for you. Rene is getting himself in a muddle because he hasn't factored in the pull of the sun on the earth.

http://www.ralphrene.com/

Quote
My "Unproof of Newtonian Gravity" was one of my first. Using the Newtonian Formula, the listed gravitational constant and the accepted masses, and distances of Sun, Earth & Moon, I found that during New Moon, the Sun's attraction for the Moon Is almost 3 times that of the Earth. Therefore, once every month the Moon, obeying the stronger force, should cease orbiting the Earth and head toward the Sun. It doesn't!

Another physisist gas bag working for the National Science Foundation told me my Unproof of Gravity I was wrong because the Sun also pulls on the Earth. In other words you can no longer isolate forces in Physics. If this is the case every force table experiment must be discredited.

We can no longer isolate forces in physics?

Declaring that the moon and the earth are close, so it's practically the same thing, and then literally crossing out the physics equations that are working against you is not a valid solution to this.
Title: Re: Round Earth Celestial Mechanics Cannot Predict the Solar System
Post by: markjo on July 30, 2018, 09:29:13 PM
It's not even an argument, he's literally done the maths for you. Rene is getting himself in a muddle because he hasn't factored in the pull of the sun on the earth.

http://www.ralphrene.com/

Quote
My "Unproof of Newtonian Gravity" was one of my first. Using the Newtonian Formula, the listed gravitational constant and the accepted masses, and distances of Sun, Earth & Moon, I found that during New Moon, the Sun's attraction for the Moon Is almost 3 times that of the Earth. Therefore, once every month the Moon, obeying the stronger force, should cease orbiting the Earth and head toward the Sun. It doesn't!

Another physisist gas bag working for the National Science Foundation told me my Unproof of Gravity I was wrong because the Sun also pulls on the Earth. In other words you can no longer isolate forces in Physics. If this is the case every force table experiment must be discredited.

We can no longer isolate forces in physics?

Declaring that the moon and the earth are close, so it's practically the same thing, and then literally crossing out the physics equations that are working against you is not a valid solution to this.
Just out of curiosity, do you or Ralph Rene ever take the earth's and moon's velocity into consideration when calculating their expected orbits?

Also, do you consider an n-body solution to be the only valid way to model the solar system?  If so, then why?
Title: Re: Round Earth Celestial Mechanics Cannot Predict the Solar System
Post by: BillO on July 30, 2018, 09:29:59 PM
What did I tell you?  He has no understanding of even basic mathematical rules.

(A/A)*(B/C)=(B/C)*(D1/D2)


Since: A/A=1, therefore:

B/C=(B/C)*(D1/D2)

(B/C)*(C/B)=D1/D2


Since (B/C)*(C/B)=1, therefore:

D1/D2=1

D1=D2


Where A=GM/d2, B=m1, C=m2, D1=a1, D2=a2

I'm betting this won't help either.
Title: Re: Round Earth Celestial Mechanics Cannot Predict the Solar System
Post by: Tom Bishop on July 30, 2018, 09:51:02 PM
It's not even an argument, he's literally done the maths for you. Rene is getting himself in a muddle because he hasn't factored in the pull of the sun on the earth.

http://www.ralphrene.com/

Quote
My "Unproof of Newtonian Gravity" was one of my first. Using the Newtonian Formula, the listed gravitational constant and the accepted masses, and distances of Sun, Earth & Moon, I found that during New Moon, the Sun's attraction for the Moon Is almost 3 times that of the Earth. Therefore, once every month the Moon, obeying the stronger force, should cease orbiting the Earth and head toward the Sun. It doesn't!

Another physisist gas bag working for the National Science Foundation told me my Unproof of Gravity I was wrong because the Sun also pulls on the Earth. In other words you can no longer isolate forces in Physics. If this is the case every force table experiment must be discredited.

We can no longer isolate forces in physics?

Declaring that the moon and the earth are close, so it's practically the same thing, and then literally crossing out the physics equations that are working against you is not a valid solution to this.
Just out of curiosity, do you or Ralph Rene ever take the earth's and moon's velocity into consideration when calculating their expected orbits?

Also, do you consider an n-body solution to be the only valid way to model the solar system?  If so, then why?

As Rene says, at the point of New Moon the moon is experiencing the following forces:

Sun Force = 8.872 E13
Earth Force = 3.636 E13

Those were generated by Newton's Force = Mass / Distance^2. The distances between the earth and the moon, and the moon and the sun, are already accounted for in that equation. Declaring that the moon and earth is close, and are practically the same, so we should cross out the equations, is nonsensical. Newton's equation tells us how much force the moon is experiencing, with the distances already accounted for.

Yet we are supposed to believe that after the New Moon the moon will then move towards the earth?

I don't see how the speed of the moon has anything to do with it. A photon traveling at light speed would also move towards the greater force.

The n-body and three body solutions do isolate the forces; and this is why there are no Three Body Problem solutions that look anything like a heliocentric system. The forces are accounted for and the heliocentric system cannot exist.
Title: Re: Round Earth Celestial Mechanics Cannot Predict the Solar System
Post by: markjo on July 30, 2018, 10:40:18 PM
As Rene says, at the point of New Moon the moon is experiencing the following forces:

Sun Force = 8.872 E13
Earth Force = 3.636 E13

Those were generated by Newton's Force = Mass / Distance^2. The distances between the earth and the moon, and the moon and the sun, are already accounted for in that equation. Declaring that the moon and earth is close, and are practically the same, so we should cross out the equations, is nonsensical. Newton's equation tells us how much force the moon is experiencing, with the distances already accounted for.

Yet we are supposed to believe that after the New Moon the moon will then move towards the earth?
How much force does the sun exert on the earth and how does that compare with the force that the sun exerts on the moon?

I don't see how the speed of the moon has anything to do with it. A photon traveling at light speed would also move towards the greater force.
Are you saying that the velocity of an object has nothing to do with its orbit around another object? ???

The n-body and three body solutions do isolate the forces; and this is why there are no Three Body Problem solutions that look anything like a heliocentric system. The forces are accounted for and the heliocentric system cannot exist.
Tom, are you insisting on an analytical solution to an n-body solar system or will a numerical solution suffice?  Granted, an analytical solution isn't possible, but numerical solutions are possible.
Title: Re: Round Earth Celestial Mechanics Cannot Predict the Solar System
Post by: garygreen on July 30, 2018, 11:32:34 PM
As Rene says, at the point of New Moon the moon is experiencing the following forces:

Sun Force = 8.872 E13
Earth Force = 3.636 E13

Those were generated by Newton's Force = Mass / Distance^2. The distances between the earth and the moon, and the moon and the sun, are already accounted for in that equation. Declaring that the moon and earth is close, and are practically the same, so we should cross out the equations, is nonsensical. Newton's equation tells us how much force the moon is experiencing, with the distances already accounted for.

a 0.5% difference in distance is a very small difference.  1 and 1.005 are both basically 1.

if you don't like that i cancelled out the distance terms, then just plug in the real values and do the calculation.  it's just algebra.

acceleration of the earth due to the sun (http://www.wolframalpha.com/input/?i=G+*+(sun+mass)%2F(1.519%C3%9710%5E8+km)%5E2) = 0.0057 m/s^2

acceleration of the moon due to the sun (http://www.wolframalpha.com/input/?i=G+*+(sun+mass)%2F(1.519%C3%9710%5E8+km+-+800000+km)%5E2) = 0.0058 m/s^2
Title: Re: Round Earth Celestial Mechanics Cannot Predict the Solar System
Post by: BillO on July 31, 2018, 02:25:11 AM
Sun Force = 8.872 E13
Earth Force = 3.636 E13

I wonder if the person that posted this realizes that an orbiting body is in free-fall and experiences zero net force?  No, I guess not - won't work on a fake earth.

If I'm not banned for life by tomorrow or the next day, I'll 'do the math' on this.
Title: Re: Round Earth Celestial Mechanics Cannot Predict the Solar System
Post by: HorstFue on July 31, 2018, 11:17:14 PM
As Rene (http://www.ralphrene.com/) says, at the point of New Moon the moon is experiencing the following forces:

Sun Force = 8.872 E13
Earth Force = 3.636 E13

Those were generated by Newton's Force = Mass / Distance^2. The distances between the earth and the moon, and the moon and the sun, are already accounted for in that equation. Declaring that the moon and earth is close, and are practically the same, so we should cross out the equations, is nonsensical. Newton's equation tells us how much force the moon is experiencing, with the distances already accounted for.

Yet we are supposed to believe that after the New Moon the moon will then move towards the earth?
Gravity force of the sun doesn't matter. It's already canceled out by the centrifugal force.

Let's for a moment assume, earth does not attract the moon, e.g. assuming a moon being on the same orbit around the sun as earth, but a little behind, so that earth attraction is minimal. The moon would follow earth in a distance with exact the same speed. For the orbit mass does not matter, because both gravity and centrifugal force are proportional to the mass. So a small object can have the same orbit as a big object. So moon could have exact the same orbit as earth.

Now bring earth and moon back to their old "positions". At average both Earth and moon would still follow this orbit around the sun from above, but additionally orbit around themselves. The combined center of mass from earth and moon is still exactly on the orbit from above.
Title: Re: Round Earth Celestial Mechanics Cannot Predict the Solar System
Post by: BillO on August 01, 2018, 04:41:49 PM
I promised this, so I'll deliver - even though I know it will be dismissed.  However the math is here for you to try yourself.

Fist let me point out someone is not doing their math right, and someone using that math is not even bothering to check it.  Because these numbers:

Sun Force = 8.872 E13
Earth Force = 3.636 E13


Are pure hokum.  Zetetic indeed.  See ** at the end.

Now to get on with it...

So, an object in a nearly circular orbit about a mass is neither accelerating toward the mass or away from it, but instead is traveling in more or less uniform velocity around the mass.  In other words is exist in a state where at any point on its orbit the force, and therefore the accelerations’ on it are in equilibrium.  There are two sources of acceleration, those from gravity and those that arise centrifugally form the orbit itself.  Here the task is to show that the accelerations on the moon are in equilibrium.  They must all add up to 0.  Mathematically stated: ag-ac=0, for each orbital instance.

Let’s use Mr. Rene's facts and figures.  According to him we are looking at moon’s orbit around the sun of

r = 1.498E011 m, and a sun’s mass of m=1.991E30 kg

And for the moon’s orbit around earth he gives the following:

r = 4.055E8 m, and an earth’s mass of 5.979E24 kg

He then goes on to calculate something he calls ‘relative force’, there is no such thing, but if we look carefully at the equation he uses:

1)   RF = MASS/DISTANCE2, which we can see is similar to the equation for acceleration due to gravity:

2)   a = Gm/r2, where G is the gravitational constant (6.67E-11), m is the mass of the object in kg, r the radius of the object’s orbit in meters, and a is the acceleration in m/s2

We are going to need that acceleration.  We can get that we can correct Rene’s ‘work’ substituting RF for  m/r2 to get:

ag = G*RF

Further, in order to determine the centrifugal acceleration we are also going to need to calculate the orbital velocity of each of these orbits.

3)   v = sqrt(Gm/r)

And the centrifugal acceleration, which is given by

4)   ac = v2/r

So let’s begin.  We will first calculate the accelerations on the sun-moon instance using Rene’s numbers:

ag (sun-moon) = 6.67E-11*1.991E30/(1.498E11)2

ag (sun-moon) = 5.918E-3 m/s2

Now we’ll look at the centrifugal acceleration.  Using eq. 3) we get:

v(sun-moon) = sqrt(6.67E-11*1.991E30/1.498E11)
v(sun-moon) = 29774 m/s

Then from eq. 4) we get the centrifugal acceleration as:

ac (sun-moon) = 297742/1.498E11
ac (sun-moon) = 5.918E-3 m/s2

Such that:

ag (sun-moon)- ac (sun-moon) = 5.918E-3 m/s2- 5.918E-3 m/s2 = 0

Well, would you look at that!  No net acceleration and hence no net force.

Now let’s now look at the earth-moon instance:

ag (earth-moon) = 6.67E-11*5.979E24/(4.055E08)2

ag (earth-moon) = 2.425E-3 m/s2

Now we’ll look at the centrifugal acceleration as before:

v(earth-moon) = sqrt(6.67E-11*5.979E24/4.055E08)
v(earth-moon) = 991.7 m/s

And the centrifugal acceleration:

ac (earth-moon) = 991.72/4.055E08
ac (earth-moon) = 2.425E-3 m/s2

Such that:
ag (earth-moon)- ac (earth-moon) = 2.425E-3 m/s2- 2.425E-3 m/s2 = 0

My, oh my…

Just as a check on myself, let’s see if the ratio in the gravitational accelerations I calculated for the sun-moon and earth-moon match the ratio given by Rene.
5.918E-3 m/s2/ 2.425E-3 m/s2 = 2.440

Conclusion, there is no net acceleration or force acting on the moon due to the earth or sun.

q.e.d.


**Now just for fun let me note something for those of you that think of Rene as a minor prophet for your FE hypothesis.    AS I stated before there is no ‘relative mass’ thingy calculated as m/r2 and further,  he mixes his unit systems.  In the real world of physics, there are 3 unit systems usually used. cgs/Gaussian (centimeter-gram-second), mks/SI (meter-kilogram-second) and, rarely BE/fsg (foot-slug-second).  Your Mr. Rene used kg with km and ended up over estimating his ‘relative force’, whatever that is,  by a factor of 1 million!!!!  What a cac snámh!  And you trust this dude?  I will note that in the end he was taking a ratio and since this whole thing is linear in nature the ratio turned out to be correct and I admitted that before – however ignorance + 2 wrongs don’t make a right.  The real numbers:

Sun Force = 4.347E20 N
Earth Force = 1.781E20 N

Still a ratio of 2.440, but by doing the real math and producing the real numbers.  When you copy a cac snámh, you make yourself a cac snámh.  I thought zetetics were not supposed to copy anyone and do the research themselves?
Title: Re: Round Earth Celestial Mechanics Cannot Predict the Solar System
Post by: Tom Bishop on August 01, 2018, 05:47:49 PM
So, an object in a nearly circular orbit about a mass is neither accelerating toward the mass or away from it, but instead is traveling in more or less uniform velocity around the mass.  In other words is exist in a state where at any point on its orbit the force, and therefore the accelerations’ on it are in equilibrium.  There are two sources of acceleration, those from gravity and those that arise centrifugally form the orbit itself.

...

Such that:

ag (sun-moon)- ac (sun-moon) = 5.918E-3 m/s2- 5.918E-3 m/s2 = 0

...

Such that:
ag (earth-moon)- ac (earth-moon) = 2.425E-3 m/s2- 2.425E-3 m/s2 = 0

You are just saying that because a two-body orbit is possible, where the inward pull is balanced by the outwards centrifugal pull, that orbits can exist in any situation, and we can just stack them on top of each other.

If the moon were perfectly orbiting the sun, with just those two bodies alone, the moon would have a net acceleration of 0 towards the sun. That is what you are saying, correct? But this says nothing about the pull of the moon towards the earth in that description.

If the moon were perfectly orbiting the earth, with just those two bodies alone, the moon would have a net acceleration of 0 towards the earth. Again, that is what you are saying, correct? But this says nothing about the pull of the moon towards the sun in that description.

The pull of the moon towards the sun and the pull towards the earth is DIFFERENT.

We can't take those concepts of two-body orbits and combine them like magic to make a three body system. It doesn't work. The moon's pull from the sun is different than the moon's pull from the earth. That's the issue with the Three Body Problem: Two-Body orbits are possible, but stable orbits cannot be created with three or more bodies. It causes chaos.

If what you were saying were true, then it would be easy to predict the position of three bodies put into motion in that manner. They would have solved the Three Body Problem hundreds of years ago. They would have proclaimed "Stable orbits are easy! It's just two two-body orbits stacked on top of each other!" Your explanation is either an attempt at deception or a demonstration of ignoring the situation and searching really hard for excuses. The reader can decide which one to file you under.
Title: Re: Round Earth Celestial Mechanics Cannot Predict the Solar System
Post by: BillO on August 01, 2018, 05:47:57 PM
I give up.

The math here has been done n-ways to Sunday and you have been provided with a program that provides a 3 body pure numerical solution (it will actually do up to 10).

I was told once by a very smart person that, while simple ignorance can be defeated through education, if it is willful there is nothing that can be done.

https://en.wiktionary.org/wiki/willful_ignorance (https://en.wiktionary.org/wiki/willful_ignorance)

Quote
Noun

willful ignorance (uncountable)
1.   (idiomatic, law) A decision in bad faith to avoid becoming informed about something so as to avoid having to make undesirable decisions that such information might prompt.

Synonyms
•   (bad-faith decision to remain ignorant): vincible ignorance, willful blindness

Translations
 A decision in bad faith to avoid becoming informed

See also
•   Nelsonian knowledge
Title: Re: Round Earth Celestial Mechanics Cannot Predict the Solar System
Post by: markjo on August 01, 2018, 06:14:27 PM
If what you were saying were true, then it would be easy to predict the position of three bodies put into motion in that manner.
But they can predict the positions of the 3 bodies in motion.  That's what makes things like gravitational assists and trips to other bodies in the solar system possible.

They would have solved the Three Body Problem hundreds of years ago. They would have proclaimed "Stable orbits are easy! It's just two two-body orbits stacked on top of each other!"
The moon has been shown to be moving away from the earth at the rate of about one inch per year.  An argument could be made that the moon's orbit around the earth doesn't qualify as stable.  It's also known that the orbits of the outer gas giant planets perturb each other periodically to the point where Jupiter either absorbed or ejected a number of bodies out of the solar system and Neptune and Uranus swapped orbits about 4 billion years ago.  I don't think that the solar system is quite as stable as you think that it should be.
Title: Re: Round Earth Celestial Mechanics Cannot Predict the Solar System
Post by: HorstFue on August 01, 2018, 08:09:05 PM
...
You are just saying that because a two-body orbit is possible, where the inward pull is balanced by the outwards centrifugal pull, that orbits can exist in any situation, and we can just stack them on top of each other.
Ok, on a close inspection you are right, that's a 3 body problem, unsolvable.
But what would physicists do, when faced with an unsolvable problem? They make assumptions, approximations, that would transform the problem to a solvable one.
Distance to Sun is about 389 times the distance earth-moon. So the earth-moon combination is almost at the same distance to the Sun at any time. You could also say the combined earth-moon system is orbiting the Sun. 
I would assume, in this case you could "just stack them (the orbits) on top of each other".
At least for the very coarse question, if the moon will fall into the Sun or continue orbiting the earth.

The pull of the moon towards the sun and the pull towards the earth is DIFFERENT.
Yes,  I agree, but how DIFFERENT? My math. is not sufficient or this.
Can You, Tom give as a number? Or ask Ralph Renés for an answer.

Ah, sorry, later is not available right now. For not including centrifugal forces in his calculations, I just filed him under "nonsense".
Title: Re: Round Earth Celestial Mechanics Cannot Predict the Solar System
Post by: garygreen on August 01, 2018, 09:11:35 PM
The pull of the moon towards the sun and the pull towards the earth is DIFFERENT.

acceleration of the earth due to the sun = 0.0057 m/s^2

acceleration of the moon due to the sun = 0.0058 m/s^2

that's a difference of 0.0001 m/s^2 when the forces are maximally different.

i still don't get what your point is.  astronomers have known about this fact at least since newton (https://en.wikipedia.org/wiki/Lunar_theory#Solar_perturbation_of_lunar_motion).  the moon's orbit is dynamic.  no one but you says that it's supposed to be perfectly stable.